You are on page 1of 40

[G.R. No. 127882.

January 27, 2004] Petitioners thus filed the present petition for prohibition and mandamus, with a prayer for a temporary
LA BUGAL-BLAAN TRIBAL ASSOCIATION, INC., petitioners, vs. VICTOR O. RAMOS, SECRETARY, restraining order. They allege that at the time of the filing of the petition, 100 FTAA applications had already
DEPARTMENT OF ENVIRONMENT AND NATURAL RESOURCES (DENR), HORACIO RAMOS, been filed, covering an area of 8.4 million hectares,[38] 64 of which applications are by fully foreign-owned
DIRECTOR, MINES AND GEOSCIENCES BUREAU (MGB-DENR), RUBEN TORRES, corporations covering a total of 5.8 million hectares, and at least one by a fully foreign-owned mining
EXECUTIVE SECRETARY, and WMC (PHILIPPINES), INC. [4] respondents. company over offshore areas.[39]
DECISION Petitioners claim that the DENR Secretary acted without or in excess of jurisdiction:
CARPIO-MORALES, J.: I
The present petition for mandamus and prohibition assails the constitutionality of Republic Act No. x x x in signing and promulgating DENR Administrative Order No. 96-40 implementing Republic Act No.
7942,[5] otherwise known as the PHILIPPINE MINING ACT OF 1995, along with the Implementing Rules and 7942, the latter being unconstitutional in that it allows fully foreign owned corporations to explore, develop,
Regulations issued pursuant thereto, Department of Environment and Natural Resources (DENR) utilize and exploit mineral resources in a manner contrary to Section 2, paragraph 4, Article XII of the
Administrative Order 96-40, and of the Financial and Technical Assistance Agreement (FTAA) entered into Constitution;
on March 30, 1995 by the Republic of the Philippines and WMC (Philippines), Inc. (WMCP), a corporation II
organized under Philippine laws. x x x in signing and promulgating DENR Administrative Order No. 96-40 implementing Republic Act No.
On July 25, 1987, then President Corazon C. Aquino issued Executive Order (E.O.) No. 7942, the latter being unconstitutional in that it allows the taking of private property without the determination
279[6] authorizing the DENR Secretary to of public use and for just compensation;
accept, consider and evaluate proposals from foreign-owned corporations or foreign investors for contracts III
or agreements involving either technical or financial assistance for large-scale exploration, development, and x x x in signing and promulgating DENR Administrative Order No. 96-40 implementing Republic Act No.
utilization of minerals, which, upon appropriate recommendation of the Secretary, the President may execute 7942, the latter being unconstitutional in that it violates Sec. 1, Art. III of the Constitution;
with the foreign proponent. In entering into such proposals, the President shall consider the real IV
contributions to the economic growth and general welfare of the country that will be realized, as well as the x x x in signing and promulgating DENR Administrative Order No. 96-40 implementing Republic Act No.
development and use of local scientific and technical resources that will be promoted by the proposed 7942, the latter being unconstitutional in that it allows enjoyment by foreign citizens as well as fully foreign
contract or agreement. Until Congress shall determine otherwise, large-scale mining, for purpose of this owned corporations of the nations marine wealth contrary to Section 2, paragraph 2 of Article XII of the
Section, shall mean those proposals for contracts or agreements for mineral resources exploration, Constitution;
development, and utilization involving a committed capital investment in a single mining unit project of at V
least Fifty Million Dollars in United States Currency (US $50,000,000.00).[7] x x x in signing and promulgating DENR Administrative Order No. 96-40 implementing Republic Act No.
On March 3, 1995, then President Fidel V. Ramos approved R.A. No. 7942 to govern the exploration, 7942, the latter being unconstitutional in that it allows priority to foreign and fully foreign owned corporations
development, utilization and processing of all mineral resources. [8] R.A. No. 7942 defines the modes of in the exploration, development and utilization of mineral resources contrary to Article XII of the Constitution;
mineral agreements for mining operations,[9] outlines the procedure for their filing and VI
approval,[10] assignment/transfer[11] and withdrawal,[12] and fixes their terms.[13]Similar provisions govern x x x in signing and promulgating DENR Administrative Order No. 96-40 implementing Republic Act No.
financial or technical assistance agreements.[14] 7942, the latter being unconstitutional in that it allows the inequitable sharing of wealth contrary to Sections
The law prescribes the qualifications of contractors [15] and grants them certain rights, including [sic] 1, paragraph 1, and Section 2, paragraph 4[,] [Article XII] of the Constitution;
timber,[16] water[17] and easement[18] rights, and the right to possess explosives.[19] Surface owners, VII
occupants, or concessionaires are forbidden from preventing holders of mining rights from entering private x x x in recommending approval of and implementing the Financial and Technical Assistance Agreement
lands and concession areas.[20] A procedure for the settlement of conflicts is likewise provided for. [21] between the President of the Republic of the Philippines and Western Mining Corporation Philippines Inc.
The Act restricts the conditions for exploration, [22] quarry[23] and other[24] permits. It regulates the because the same is illegal and unconstitutional.[40]
transport, sale and processing of minerals,[25] and promotes the development of mining communities, They pray that the Court issue an order:
science and mining technology,[26] and safety and environmental protection.[27] (a) Permanently enjoining respondents from acting on any application for Financial or Technical Assistance
The governments share in the agreements is spelled out and allocated,[28] taxes and fees are Agreements;
imposed,[29] incentives granted.[30] Aside from penalizing certain acts,[31] the law likewise specifies grounds (b) Declaring the Philippine Mining Act of 1995 or Republic Act No. 7942 as unconstitutional and null and
for the cancellation, revocation and termination of agreements and permits. [32] void;
On April 9, 1995, 30 days following its publication on March 10, 1995 in Malaya and Manila Times, two (c) Declaring the Implementing Rules and Regulations of the Philippine Mining Act contained in DENR
newspapers of general circulation, R.A. No. 7942 took effect.[33] Administrative Order No. 96-40 and all other similar administrative issuances as unconstitutional and null
Shortly before the effectivity of R.A. No. 7942, however, or on March 30, 1995, the President entered and void; and
into an FTAA with WMCP covering 99,387 hectares of land in South Cotabato, Sultan Kudarat, Davao del (d) Cancelling the Financial and Technical Assistance Agreement issued to Western Mining Philippines, Inc.
Sur and North Cotabato.[34] as unconstitutional, illegal and null and void.[41]
On August 15, 1995, then DENR Secretary Victor O. Ramos issued DENR Administrative Order (DAO) Impleaded as public respondents are Ruben Torres, the then Executive Secretary, Victor O. Ramos,
No. 95-23, s. 1995, otherwise known as the Implementing Rules and Regulations of R.A. No. 7942. This was the then DENR Secretary, and Horacio Ramos, Director of the Mines and Geosciences Bureau of the
later repealed by DAO No. 96-40, s. 1996 which was adopted on December 20, 1996. DENR. Also impleaded is private respondent WMCP, which entered into the assailed FTAA with the
On January 10, 1997, counsels for petitioners sent a letter to the DENR Secretary demanding that the Philippine Government. WMCP is owned by WMC Resources International Pty., Ltd. (WMC), a wholly
DENR stop the implementation of R.A. No. 7942 and DAO No. 96-40,[35] giving the DENR fifteen days from owned subsidiary of Western Mining Corporation Holdings Limited, a publicly listed major Australian
receipt[36] to act thereon. The DENR, however, has yet to respond or act on petitioners letter.[37] mining and exploration company.[42] By WMCPs information, it is a 100% owned subsidiary of WMC
LIMITED.[43]
Respondents, aside from meeting petitioners contentions, argue that the requisites for judicial inquiry An actual case or controversy means an existing case or controversy that is appropriate or ripe for
have not been met and that the petition does not comply with the criteria for prohibition and determination, not conjectural or anticipatory, [60] lest the decision of the court would amount to an advisory
mandamus. Additionally, respondent WMCP argues that there has been a violation of the rule on hierarchy opinion.[61] The power does not extend to hypothetical questions [62] since any attempt at abstraction could
of courts. only lead to dialectics and barren legal questions and to sterile conclusions unrelated to actualities. [63]
After petitioners filed their reply, this Court granted due course to the petition. The parties have since Legal standing or locus standi has been defined as a personal and substantial interest in the case such
filed their respective memoranda. that the party has sustained or will sustain direct injury as a result of the governmental act that is being
WMCP subsequently filed a Manifestation dated September 25, 2002 alleging that on January 23, challenged,[64] alleging more than a generalized grievance. [65] The gist of the question of standing is whether
2001, WMC sold all its shares in WMCP to Sagittarius Mines, Inc. (Sagittarius), a corporation organized a party alleges such personal stake in the outcome of the controversy as to assure that concrete
under Philippine laws.[44] WMCP was subsequently renamed Tampakan Mineral Resources adverseness which sharpens the presentation of issues upon which the court depends for illumination of
Corporation.[45] WMCP claims that at least 60% of the equity of Sagittarius is owned by Filipinos and/or difficult constitutional questions.[66] Unless a person is injuriously affected in any of his constitutional rights by
Filipino-owned corporations while about 40% is owned by Indophil Resources NL, an Australian the operation of statute or ordinance, he has no standing. [67]
company.[46] It further claims that by such sale and transfer of shares, WMCP has ceased to be connected in Petitioners traverse a wide range of sectors. Among them are La Bugal Blaan Tribal Association, Inc.,
any way with WMC.[47] a farmers and indigenous peoples cooperative organized under Philippine laws representing a community
By virtue of such sale and transfer, the DENR Secretary, by Order of December 18, 2001, [48] approved actually affected by the mining activities of WMCP, members of said cooperative, [68] as well as other
the transfer and registration of the subject FTAA from WMCP to Sagittarius. Said Order, however, was residents of areas also affected by the mining activities of WMCP.[69] These petitioners have standing to
appealed by Lepanto Consolidated Mining Co. (Lepanto) to the Office of the President which upheld it by raise the constitutionality of the questioned FTAA as they allege a personal and substantial injury. They
Decision of July 23, 2002.[49] Its motion for reconsideration having been denied by the Office of the President claim that they would suffer irremediable displacement [70] as a result of the implementation of the FTAA
by Resolution of November 12, 2002,[50] Lepanto filed a petition for review[51] before the Court of allowing WMCP to conduct mining activities in their area of residence. They thus meet the appropriate case
Appeals. Incidentally, two other petitions for review related to the approval of the transfer and registration of requirement as they assert an interest adverse to that of respondents who, on the other hand, insist on the
the FTAA to Sagittarius were recently resolved by this Court. [52] FTAAs validity.
It bears stressing that this case has not been rendered moot either by the transfer and registration of In view of the alleged impending injury, petitioners also have standing to assail the validity of E.O. No.
the FTAA to a Filipino-owned corporation or by the non-issuance of a temporary restraining order or a 279, by authority of which the FTAA was executed.
preliminary injunction to stay the above-said July 23, 2002 decision of the Office of the President. [53] The Public respondents maintain that petitioners, being strangers to the FTAA, cannot sue either or both
validity of the transfer remains in dispute and awaits final judicial determination. This assumes, of course, contracting parties to annul it.[71] In other words, they contend that petitioners are not real parties in interest
that such transfer cures the FTAAs alleged unconstitutionality, on which question judgment is reserved. in an action for the annulment of contract.
WMCP also points out that the original claimowners of the major mineralized areas included in the Public respondents contention fails. The present action is not merely one for annulment of contract but
WMCP FTAA, namely, Sagittarius, Tampakan Mining Corporation, and Southcot Mining Corporation, are all for prohibition and mandamus. Petitioners allege that public respondents acted without or in excess of
Filipino-owned corporations,[54] each of which was a holder of an approved Mineral Production Sharing jurisdiction in implementing the FTAA, which they submit is unconstitutional. As the case involves
Agreement awarded in 1994, albeit their respective mineral claims were subsumed in the WMCP constitutional questions, this Court is not concerned with whether petitioners are real parties in interest, but
FTAA;[55] and that these three companies are the same companies that consolidated their interests in with whether they have legal standing. As held in Kilosbayan v. Morato:[72]
Sagittarius to whom WMC sold its 100% equity in WMCP. [56] WMCP concludes that in the event that the x x x. It is important to note . . . that standing because of its constitutional and public policy underpinnings, is
FTAA is invalidated, the MPSAs of the three corporations would be revived and the mineral claims would very different from questions relating to whether a particular plaintiff is the real party in interest or has
revert to their original claimants.[57] capacity to sue. Although all three requirements are directed towards ensuring that only certain parties can
These circumstances, while informative, are hardly significant in the resolution of this case, it involving maintain an action, standing restrictions require a partial consideration of the merits, as well as broader
the validity of the FTAA, not the possible consequences of its invalidation. policy concerns relating to the proper role of the judiciary in certain areas.[] (FRIEDENTHAL, KANE AND
Of the above-enumerated seven grounds cited by petitioners, as will be shown later, only the first and MILLER, CIVIL PROCEDURE 328 [1985])
the last need be delved into; in the latter, the discussion shall dwell only insofar as it questions the effectivity Standing is a special concern in constitutional law because in some cases suits are brought not by parties
of E. O. No. 279 by virtue of which order the questioned FTAA was forged. who have been personally injured by the operation of a law or by official action taken, but by concerned
I citizens, taxpayers or voters who actually sue in the public interest. Hence, the question in standing is
Before going into the substantive issues, the procedural questions posed by respondents shall first be whether such parties have alleged such a personal stake in the outcome of the controversy as to assure that
tackled. concrete adverseness which sharpens the presentation of issues upon which the court so largely depends
REQUISITES FOR JUDICIAL REVIEW for illumination of difficult constitutional questions. (Baker v. Carr, 369 U.S. 186, 7 L.Ed.2d 633 [1962].)
When an issue of constitutionality is raised, this Court can exercise its power of judicial review only if As earlier stated, petitioners meet this requirement.
the following requisites are present: The challenge against the constitutionality of R.A. No. 7942 and DAO No. 96-40 likewise fulfills the
(1) The existence of an actual and appropriate case; requisites of justiciability. Although these laws were not in force when the subject FTAA was entered into,
(2) A personal and substantial interest of the party raising the constitutional question; the question as to their validity is ripe for adjudication.
(3) The exercise of judicial review is pleaded at the earliest opportunity; and The WMCP FTAA provides:
(4) The constitutional question is the lis mota of the case. [58] 14.3 Future Legislation
Respondents claim that the first three requisites are not present. Any term and condition more favourable to Financial &Technical Assistance Agreement
Section 1, Article VIII of the Constitution states that (j)udicial power includes the duty of the courts of contractors resulting from repeal or amendment of any existing law or regulation or from
justice to settle actual controversies involving rights which are legally demandable and enforceable. The the enactment of a law, regulation or administrative order shall be considered a part of this
power of judicial review, therefore, is limited to the determination of actual cases and controversies. [59] Agreement.
It is undisputed that R.A. No. 7942 and DAO No. 96-40 contain provisions that are more favorable to WMCP, against the latter, with the Court of Appeals. A direct invocation of the Supreme Courts original
hence, these laws, to the extent that they are favorable to WMCP, govern the FTAA. jurisdiction to issue these writs should be allowed only where there are special and important
In addition, R.A. No. 7942 explicitly makes certain provisions apply to pre-existing agreements. reasons therefor, clearly and specifically set out in the petition. This is established policy. It is a policy
SEC. 112. Non-impairment of Existing Mining/Quarrying Rights. x x x That the provisions of Chapter XIV on necessary to prevent inordinate demands upon the Courts time and attention which are better devoted to
government share in mineral production-sharing agreement and of Chapter XVI on incentives of this Act those matters within its exclusive jurisdiction, and to prevent further over-crowding of the Courts docket x x
shall immediately govern and apply to a mining lessee or contractor unless the mining lessee or contractor x.[76] [Emphasis supplied.]
indicates his intention to the secretary, in writing, not to avail of said provisions x x xProvided, finally, That The repercussions of the issues in this case on the Philippine mining industry, if not the national
such leases, production-sharing agreements, financial or technical assistance agreements shall comply with economy, as well as the novelty thereof, constitute exceptional and compelling circumstances to justify
the applicable provisions of this Act and its implementing rules and regulations. resort to this Court in the first instance.
As there is no suggestion that WMCP has indicated its intention not to avail of the provisions of Chapter XVI In all events, this Court has the discretion to take cognizance of a suit which does not satisfy the
of R.A. No. 7942, it can safely be presumed that they apply to the WMCP FTAA. requirements of an actual case or legal standing when paramount public interest is involved. [77] When the
Misconstruing the application of the third requisite for judicial review that the exercise of the review is issues raised are of paramount importance to the public, this Court may brush aside technicalities of
pleaded at the earliest opportunity WMCP points out that the petition was filed only almost two years after procedure.[78]
the execution of the FTAA, hence, not raised at the earliest opportunity. II
The third requisite should not be taken to mean that the question of constitutionality must be raised Petitioners contend that E.O. No. 279 did not take effect because its supposed date of effectivity came
immediately after the execution of the state action complained of. That the question of constitutionality has after President Aquino had already lost her legislative powers under the Provisional Constitution.
not been raised before is not a valid reason for refusing to allow it to be raised later.[73] A contrary rule would And they likewise claim that the WMC FTAA, which was entered into pursuant to E.O. No. 279, violates
mean that a law, otherwise unconstitutional, would lapse into constitutionality by the mere failure of the Section 2, Article XII of the Constitution because, among other reasons:
proper party to promptly file a case to challenge the same. (1) It allows foreign-owned companies to extend more than mere financial or technical assistance to
PROPRIETY OF PROHIBITION the State in the exploitation, development, and utilization of minerals, petroleum, and other mineral oils, and
AND MANDAMUS even permits foreign owned companies to operate and manage mining activities.
Before the effectivity in July 1997 of the Revised Rules of Civil Procedure, Section 2 of Rule 65 read: (2) It allows foreign-owned companies to extend both technical and financial assistance, instead
SEC. 2. Petition for prohibition. When the proceedings of any tribunal, corporation, board, or person, of either technical or financial assistance.
whether exercising functions judicial or ministerial, are without or in excess of its or his jurisdiction, or with To appreciate the import of these issues, a visit to the history of the pertinent constitutional provision,
grave abuse of discretion, and there is no appeal or any other plain, speedy, and adequate remedy in the the concepts contained therein, and the laws enacted pursuant thereto, is in order.
ordinary course of law, a person aggrieved thereby may file a verified petition in the proper court alleging the Section 2, Article XII reads in full:
facts with certainty and praying that judgment be rendered commanding the defendant to desist from further Sec. 2. All lands of the public domain, waters, minerals, coal, petroleum, and other mineral oils, all forces of
proceeding in the action or matter specified therein. potential energy, fisheries, forests or timber, wildlife, flora and fauna, and other natural resources are owned
Prohibition is a preventive remedy.[74] It seeks a judgment ordering the defendant to desist from by the State. With the exception of agricultural lands, all other natural resources shall not be alienated. The
continuing with the commission of an act perceived to be illegal.[75] exploration, development, and utilization of natural resources shall be under the full control and supervision
The petition for prohibition at bar is thus an appropriate remedy. While the execution of the contract of the State. The State may directly undertake such activities or it may enter into co-production, joint venture,
itself may be fait accompli, its implementation is not. Public respondents, in behalf of the Government, have or production-sharing agreements with Filipino citizens, or corporations or associations at least sixty per
obligations to fulfill under said contract. Petitioners seek to prevent them from fulfilling such obligations on centum of whose capital is owned by such citizens. Such agreements may be for a period not exceeding
the theory that the contract is unconstitutional and, therefore, void. twenty-five years, renewable for not more than twenty-five years, and under such terms and conditions as
The propriety of a petition for prohibition being upheld, discussion of the propriety of the mandamus may be provided by law. In cases of water rights for irrigation, water supply, fisheries, or industrial uses other
aspect of the petition is rendered unnecessary. than the development of water power, beneficial use may be the measure and limit of the grant.
HIERARCHY OF COURTS The State shall protect the nations marine wealth in its archipelagic waters, territorial sea, and exclusive
The contention that the filing of this petition violated the rule on hierarchy of courts does not likewise economic zone, and reserve its use and enjoyment exclusively to Filipino citizens.
lie. The rule has been explained thus: The Congress may, by law, allow small-scale utilization of natural resources by Filipino citizens, as well as
Between two courts of concurrent original jurisdiction, it is the lower court that should initially pass upon the cooperative fish farming, with priority to subsistence fishermen and fish-workers in rivers, lakes, bays, and
issues of a case. That way, as a particular case goes through the hierarchy of courts, it is shorn of all but the lagoons.
important legal issues or those of first impression, which are the proper subject of attention of the appellate The President may enter into agreements with foreign-owned corporations involving either technical or
court. This is a procedural rule borne of experience and adopted to improve the administration of justice. financial assistance for large-scale exploration, development, and utilization of minerals, petroleum, and
This Court has consistently enjoined litigants to respect the hierarchy of courts. Although this Court has other mineral oils according to the general terms and conditions provided by law, based on real contributions
concurrent jurisdiction with the Regional Trial Courts and the Court of Appeals to issue writs of certiorari, to the economic growth and general welfare of the country. In such agreements, the State shall promote the
prohibition, mandamus, quo warranto, habeas corpus and injunction, such concurrence does not give a development and use of local scientific and technical resources.
party unrestricted freedom of choice of court forum. The resort to this Courts primary jurisdiction to issue The President shall notify the Congress of every contract entered into in accordance with this provision,
said writs shall be allowed only where the redress desired cannot be obtained in the appropriate courts or within thirty days from its execution.
where exceptional and compelling circumstances justify such invocation. We held in People v. THE SPANISH REGIME
Cuaresma that: AND THE REGALIAN DOCTRINE
A becoming regard for judicial hierarchy most certainly indicates that petitions for the issuance of
extraordinary writs against first level (inferior) courts should be filed with the Regional Trial Court, and those
The first sentence of Section 2 embodies the Regalian doctrine or jura regalia. Introduced by Spain into Section 21 also made possible the concession (frequently styled permit, license or
these Islands, this feudal concept is based on the States power of dominium, which is the capacity of the lease)[92] system.[93] This was the traditional regime imposed by the colonial administrators for the
State to own or acquire property.[79] exploitation of natural resources in the extractive sector (petroleum, hard minerals, timber, etc.). [94]
In its broad sense, the term jura regalia refers to royal rights, or those rights which the King has by virtue of Under the concession system, the concessionaire makes a direct equity investment for the purpose of
his prerogatives. In Spanish law, it refers to a right which the sovereign has over anything in which a subject exploiting a particular natural resource within a given area. [95] Thus, the concession amounts to complete
has a right of property or propriedad. These were rights enjoyed during feudal times by the king as the control by the concessionaire over the countrys natural resource, for it is given exclusive and plenary rights
sovereign. to exploit a particular resource at the point of extraction. [96] In consideration for the right to exploit a natural
The theory of the feudal system was that title to all lands was originally held by the King, and while the use resource, the concessionaire either pays rent or royalty, which is a fixed percentage of the gross
of lands was granted out to others who were permitted to hold them under certain conditions, the King proceeds.[97]
theoretically retained the title. By fiction of law, the King was regarded as the original proprietor of all lands, Later statutory enactments by the legislative bodies set up in the Philippines adopted the contractual
and the true and only source of title, and from him all lands were held. The theory of jura regalia was framework of the concession.[98] For instance, Act No. 2932,[99] approved on August 31, 1920, which
therefore nothing more than a natural fruit of conquest. [80] provided for the exploration, location, and lease of lands containing petroleum and other mineral oils and gas
The Philippines having passed to Spain by virtue of discovery and conquest, [81] earlier Spanish in the Philippines, and Act No. 2719,[100] approved on May 14, 1917, which provided for the leasing and
decrees declared that all lands were held from the Crown. [82] development of coal lands in the Philippines, both utilized the concession system.[101]
The Regalian doctrine extends not only to land but also to all natural wealth that may be found in the THE 1935 CONSTITUTION AND THE
bowels of the earth.[83] Spain, in particular, recognized the unique value of natural resources, viewing them, NATIONALIZATION OF NATURAL RESOURCES
especially minerals, as an abundant source of revenue to finance its wars against other nations. [84] Mining By the Act of United States Congress of March 24, 1934, popularly known as the Tydings-McDuffie
laws during the Spanish regime reflected this perspective.[85] Law, the People of the Philippine Islands were authorized to adopt a constitution. [102] On July 30, 1934, the
THE AMERICAN OCCUPATION AND Constitutional Convention met for the purpose of drafting a constitution, and the Constitution subsequently
THE CONCESSION REGIME drafted was approved by the Convention on February 8, 1935. [103] The Constitution was submitted to the
By the Treaty of Paris of December 10, 1898, Spain ceded the archipelago known as the Philippine President of the United States on March 18, 1935.[104] On March 23, 1935, the President of the United States
Islands to the United States. The Philippines was hence governed by means of organic acts that were in the certified that the Constitution conformed substantially with the provisions of the Act of Congress approved on
nature of charters serving as a Constitution of the occupied territory from 1900 to 1935.[86] Among the March 24, 1934.[105] On May 14, 1935, the Constitution was ratified by the Filipino people.[106]
principal organic acts of the Philippines was the Act of Congress of July 1, 1902, more commonly known as The 1935 Constitution adopted the Regalian doctrine, declaring all natural resources of the Philippines,
the Philippine Bill of 1902, through which the United States Congress assumed the administration of the including mineral lands and minerals, to be property belonging to the State.[107]As adopted in a republican
Philippine Islands.[87] Section 20 of said Bill reserved the disposition of mineral lands of the public domain system, the medieval concept of jura regalia is stripped of royal overtones and ownership of the land is
from sale. Section 21 thereof allowed the free and open exploration, occupation and purchase of mineral vested in the State.[108]
deposits not only to citizens of the Philippine Islands but to those of the United States as well: Section 1, Article XIII, on Conservation and Utilization of Natural Resources, of the 1935 Constitution
Sec. 21. That all valuable mineral deposits in public lands in the Philippine Islands, both surveyed and provided:
unsurveyed, are hereby declared to be free and open to exploration, occupation and purchase, and the land SECTION 1. All agricultural, timber, and mineral lands of the public domain, waters, minerals, coal,
in which they are found, to occupation and purchase, by citizens of the United States or of said petroleum, and other mineral oils, all forces of potential energy, and other natural resources of the
Islands: Provided, That when on any lands in said Islands entered and occupied as agricultural lands under Philippines belong to the State, and their disposition, exploitation, development, or utilization shall be limited
the provisions of this Act, but not patented, mineral deposits have been found, the working of such mineral to citizens of the Philippines, or to corporations or associations at least sixty per centum of the capital of
deposits is forbidden until the person, association, or corporation who or which has entered and is occupying which is owned by such citizens, subject to any existing right, grant, lease, or concession at the time of the
such lands shall have paid to the Government of said Islands such additional sum or sums as will make the inauguration of the Government established under this Constitution. Natural resources, with the exception of
total amount paid for the mineral claim or claims in which said deposits are located equal to the amount public agricultural land, shall not be alienated, and no license, concession, or lease for the exploitation,
charged by the Government for the same as mineral claims. development, or utilization of any of the natural resources shall be granted for a period exceeding twenty-five
Unlike Spain, the United States considered natural resources as a source of wealth for its nationals years, except as to water rights for irrigation, water supply, fisheries, or industrial uses other than the
and saw fit to allow both Filipino and American citizens to explore and exploit minerals in public lands, and to development of water power, in which cases beneficial use may be the measure and the limit of the grant.
grant patents to private mineral lands.[88] A person who acquired ownership over a parcel of private mineral The nationalization and conservation of the natural resources of the country was one of the fixed and
land pursuant to the laws then prevailing could exclude other persons, even the State, from exploiting dominating objectives of the 1935 Constitutional Convention.[109] One delegate relates:
minerals within his property.[89] Thus, earlier jurisprudence[90] held that: There was an overwhelming sentiment in the Convention in favor of the principle of state ownership of
A valid and subsisting location of mineral land, made and kept up in accordance with the provisions of the natural resources and the adoption of the Regalian doctrine. State ownership of natural resources was seen
statutes of the United States, has the effect of a grant by the United States of the present and exclusive as a necessary starting point to secure recognition of the states power to control their disposition,
possession of the lands located, and this exclusive right of possession and enjoyment continues during the exploitation, development, or utilization. The delegates of the Constitutional Convention very well knew that
entire life of the location. x x x. the concept of State ownership of land and natural resources was introduced by the Spaniards, however,
x x x. they were not certain whether it was continued and applied by the Americans. To remove all doubts, the
The discovery of minerals in the ground by one who has a valid mineral location perfects his claim and his Convention approved the provision in the Constitution affirming the Regalian doctrine.
location not only against third persons, but also against the Government. x x x. [Italics in the original.] The adoption of the principle of state ownership of the natural resources and of the Regalian doctrine was
The Regalian doctrine and the American system, therefore, differ in one essential respect. Under the considered to be a necessary starting point for the plan of nationalizing and conserving the natural resources
Regalian theory, mineral rights are not included in a grant of land by the state; under the American doctrine, of the country. For with the establishment of the principle of state ownership of the natural resources, it
mineral rights are included in a grant of land by the government. [91]
would not be hard to secure the recognition of the power of the State to control their disposition, exploitation, Exploitation concessionaires, in particular, were obliged to pay an annual exploitation tax, [128] the object
development or utilization.[110] of which is to induce the concessionaire to actually produce petroleum, and not simply to sit on the
The nationalization of the natural resources was intended (1) to insure their conservation for Filipino concession without developing or exploiting it. [129] These concessionaires were also bound to pay the
posterity; (2) to serve as an instrument of national defense, helping prevent the extension to the country of Government royalty, which was not less than 12% of the petroleum produced and saved, less that
foreign control through peaceful economic penetration; and (3) to avoid making the Philippines a source of consumed in the operations of the concessionaire. [130] Under Article 66, R.A. No. 387, the exploitation tax
international conflicts with the consequent danger to its internal security and independence. [111] may be credited against the royalties so that if the concessionaire shall be actually producing enough oil, it
The same Section 1, Article XIII also adopted the concession system, expressly permitting the State to would not actually be paying the exploitation tax.[131]
grant licenses, concessions, or leases for the exploitation, development, or utilization of any of the natural Failure to pay the annual exploitation tax for two consecutive years, [132] or the royalty due to the
resources. Grants, however, were limited to Filipinos or entities at least 60% of the capital of which is owned Government within one year from the date it becomes due, [133] constituted grounds for the cancellation of the
by Filipinos. concession. In case of delay in the payment of the taxes or royalty imposed by the law or by the concession,
The swell of nationalism that suffused the 1935 Constitution was radically diluted when on November a surcharge of 1% per month is exacted until the same are paid. [134]
1946, the Parity Amendment, which came in the form of an Ordinance Appended to the Constitution, was As a rule, title rights to all equipment and structures that the concessionaire placed on the land belong
ratified in a plebiscite.[112] The Amendment extended, from July 4, 1946 to July 3, 1974, the right to utilize to the exploration or exploitation concessionaire. [135] Upon termination of such concession, the
and exploit our natural resources to citizens of the United States and business enterprises owned or concessionaire had a right to remove the same.[136]
controlled, directly or indirectly, by citizens of the United States: [113] The Secretary of Agriculture and Natural Resources was tasked with carrying out the provisions of the
Notwithstanding the provision of section one, Article Thirteen, and section eight, Article Fourteen, of the law, through the Director of Mines, who acted under the Secretarys immediate supervision and
foregoing Constitution, during the effectivity of the Executive Agreement entered into by the President of the control.[137] The Act granted the Secretary the authority to inspect any operation of the concessionaire and to
Philippines with the President of the United States on the fourth of July, nineteen hundred and forty-six, examine all the books and accounts pertaining to operations or conditions related to payment of taxes and
pursuant to the provisions of Commonwealth Act Numbered Seven hundred and thirty-three, but in no case royalties.[138]
to extend beyond the third of July, nineteen hundred and seventy-four, the disposition, exploitation, The same law authorized the Secretary to create an Administration Unit and a Technical
development, and utilization of all agricultural, timber, and mineral lands of the public domain, waters, Board.[139] The Administration Unit was charged, inter alia, with the enforcement of the provisions of the
minerals, coals, petroleum, and other mineral oils, all forces and sources of potential energy, and other law.[140] The Technical Board had, among other functions, the duty to check on the performance of
natural resources of the Philippines, and the operation of public utilities, shall, if open to any person, be open concessionaires and to determine whether the obligations imposed by the Act and its implementing
to citizens of the United States and to all forms of business enterprise owned or controlled, directly or regulations were being complied with.[141]
indirectly, by citizens of the United States in the same manner as to, and under the same conditions Victorio Mario A. Dimagiba, Chief Legal Officer of the Bureau of Energy Development, analyzed the
imposed upon, citizens of the Philippines or corporations or associations owned or controlled by citizens of benefits and drawbacks of the concession system insofar as it applied to the petroleum industry:
the Philippines. Advantages of Concession. Whether it emphasizes income tax or royalty, the most positive aspect of the
The Parity Amendment was subsequently modified by the 1954 Revised Trade Agreement, also known concession system is that the States financial involvement is virtually risk free and administration is simple
as the Laurel-Langley Agreement, embodied in Republic Act No. 1355.[114] and comparatively low in cost. Furthermore, if there is a competitive allocation of the resource leading to
THE PETROLEUM ACT OF 1949 substantial bonuses and/or greater royalty coupled with a relatively high level of taxation, revenue accruing
AND THE CONCESSION SYSTEM to the State under the concession system may compare favorably with other financial arrangements.
In the meantime, Republic Act No. 387,[115] also known as the Petroleum Act of 1949, was approved on Disadvantages of Concession. There are, however, major negative aspects to this system. Because the
June 18, 1949. Governments role in the traditional concession is passive, it is at a distinct disadvantage in managing and
The Petroleum Act of 1949 employed the concession system for the exploitation of the nations developing policy for the nations petroleum resource. This is true for several reasons. First, even though
petroleum resources. Among the kinds of concessions it sanctioned were exploration and exploitation most concession agreements contain covenants requiring diligence in operations and production, this
concessions, which respectively granted to the concessionaire the exclusive right to explore for[116] or establishes only an indirect and passive control of the host country in resource development. Second, and
develop[117] petroleum within specified areas. more importantly, the fact that the host country does not directly participate in resource management
Concessions may be granted only to duly qualified persons [118] who have sufficient finances, decisions inhibits its ability to train and employ its nationals in petroleum development. This factor could
organization, resources, technical competence, and skills necessary to conduct the operations to be delay or prevent the country from effectively engaging in the development of its resources. Lastly, a direct
undertaken.[119] role in management is usually necessary in order to obtain a knowledge of the international petroleum
Nevertheless, the Government reserved the right to undertake such work itself.[120] This proceeded industry which is important to an appreciation of the host countrys resources in relation to those of other
from the theory that all natural deposits or occurrences of petroleum or natural gas in public and/or private countries.[142]
lands in the Philippines belong to the State.[121] Exploration and exploitation concessions did not confer upon Other liabilities of the system have also been noted:
the concessionaire ownership over the petroleum lands and petroleum deposits.[122] However, they did grant x x x there are functional implications which give the concessionaire great economic power arising from its
concessionaires the right to explore, develop, exploit, and utilize them for the period and under the exclusive equity holding. This includes, first, appropriation of the returns of the undertaking, subject to a
conditions determined by the law.[123] modest royalty; second, exclusive management of the project; third, control of production of the natural
Concessions were granted at the complete risk of the concessionaire; the Government did not resource, such as volume of production, expansion, research and development; and fourth, exclusive
guarantee the existence of petroleum or undertake, in any case, title warranty.[124] responsibility for downstream operations, like processing, marketing, and distribution. In short, even if
Concessionaires were required to submit information as maybe required by the Secretary of Agriculture nominally, the state is the sovereign and owner of the natural resource being exploited, it has been shorn of
and Natural Resources, including reports of geological and geophysical examinations, as well as production all elements of control over such natural resource because of the exclusive nature of the contractual regime
reports.[125] Exploration[126] and exploitation[127] concessionaires were also required to submit work programs. of the concession. The concession system, investing as it does ownership of natural resources, constitutes a
consistent inconsistency with the principle embodied in our Constitution that natural resources belong to the
state and shall not be alienated, not to mention the fact that the concession was the bedrock of the colonial the exception of agricultural, industrial or commercial, residential and resettlement lands of the public
system in the exploitation of natural resources.[143] domain, natural resources shall not be alienated, and no license, concession, or lease for the exploration,
Eventually, the concession system failed for reasons explained by Dimagiba: development, exploitation, or utilization of any of the natural resources shall be granted for a period
Notwithstanding the good intentions of the Petroleum Act of 1949, the concession system could not have exceeding twenty-five years, renewable for not more than twenty-five years, except as to water rights for
properly spurred sustained oil exploration activities in the country, since it assumed that such a capital- irrigation, water supply, fisheries, or industrial uses other than the development of water power, in which
intensive, high risk venture could be successfully undertaken by a single individual or a small company. In cases beneficial use may be the measure and the limit of the grant.
effect, concessionaires funds were easily exhausted. Moreover, since the concession system practically While Section 9 of the same Article maintained the Filipino-only policy in the enjoyment of natural
closed its doors to interested foreign investors, local capital was stretched to the limits. The old system also resources, it also allowed Filipinos, upon authority of the Batasang Pambansa, to enter into service contracts
failed to consider the highly sophisticated technology and expertise required, which would be available only with any person or entity for the exploration or utilization of natural resources.
to multinational companies.[144] SEC. 9. The disposition, exploration, development, exploitation, or utilization of any of the natural resources
A shift to a new regime for the development of natural resources thus seemed imminent. of the Philippines shall be limited to citizens, or to corporations or associations at least sixty per centum of
PRESIDENTIAL DECREE NO. 87, THE 1973 which is owned by such citizens. The Batasang Pambansa, in the national interest, may allow such
CONSTITUTION AND THE SERVICE CONTRACT SYSTEM citizens, corporations or associations to enter into service contracts for financial, technical,
The promulgation on December 31, 1972 of Presidential Decree No. 87, [145] otherwise known as THE management, or other forms of assistance with any person or entity for the exploration, or utilization
OIL EXPLORATION AND DEVELOPMENT ACT OF 1972 signaled such a transformation. P.D. No. 87 of any of the natural resources. Existing valid and binding service contracts for financial, technical,
permitted the government to explore for and produce indigenous petroleum through service contracts.[146] management, or other forms of assistance are hereby recognized as such. [Emphasis supplied.]
Service contracts is a term that assumes varying meanings to different people, and it has carried many The concept of service contracts, according to one delegate, was borrowed from the methods followed
names in different countries, like work contracts in Indonesia, concession agreements in Africa, production- by India, Pakistan and especially Indonesia in the exploration of petroleum and mineral oils. [162] The
sharing agreements in the Middle East, and participation agreements in Latin America. [147] A functional provision allowing such contracts, according to another, was intended to enhance the proper development of
definition of service contracts in the Philippines is provided as follows: our natural resources since Filipino citizens lack the needed capital and technical know-how which are
A service contract is a contractual arrangement for engaging in the exploitation and development of essential in the proper exploration, development and exploitation of the natural resources of the country. [163]
petroleum, mineral, energy, land and other natural resources by which a government or its agency, or a The original idea was to authorize the government, not private entities, to enter into service contracts
private person granted a right or privilege by the government authorizes the other party (service contractor) with foreign entities.[164] As finally approved, however, a citizen or private entity could be allowed by the
to engage or participate in the exercise of such right or the enjoyment of the privilege, in that the latter National Assembly to enter into such service contract.[165] The prior approval of the National Assembly was
provides financial or technical resources, undertakes the exploitation or production of a given resource, or deemed sufficient to protect the national interest. [166]Notably, none of the laws allowing service contracts
directly manages the productive enterprise, operations of the exploration and exploitation of the resources or were passed by the Batasang Pambansa. Indeed, all of them were enacted by presidential decree.
the disposition of marketing or resources.[148] On March 13, 1973, shortly after the ratification of the new Constitution, the President promulgated
In a service contract under P.D. No. 87, service and technology are furnished by the service contractor Presidential Decree No. 151.[167] The law allowed Filipino citizens or entities which have acquired lands of
for which it shall be entitled to the stipulated service fee. [149] The contractor must be technically competent the public domain or which own, hold or control such lands to enter into service contracts for financial,
and financially capable to undertake the operations required in the contract. [150] technical, management or other forms of assistance with any foreign persons or entity for the exploration,
Financing is supposed to be provided by the Government to which all petroleum produced development, exploitation or utilization of said lands. [168]
belongs.[151] In case the Government is unable to finance petroleum exploration operations, the contractor Presidential Decree No. 463,[169] also known as THE MINERAL RESOURCES DEVELOPMENT
may furnish services, technology and financing, and the proceeds of sale of the petroleum produced under DECREE OF 1974, was enacted on May 17, 1974. Section 44 of the decree, as amended, provided that a
the contract shall be the source of funds for payment of the service fee and the operating expenses due the lessee of a mining claim may enter into a service contract with a qualified domestic or foreign contractor for
contractor.[152] The contractor shall undertake, manage and execute petroleum operations, subject to the the exploration, development and exploitation of his claims and the processing and marketing of the product
government overseeing the management of the operations. [153] The contractor provides all necessary thereof.
services and technology and the requisite financing, performs the exploration work obligations, and assumes Presidential Decree No. 704[170] (THE FISHERIES DECREE OF 1975), approved on May 16, 1975,
all exploration risks such that if no petroleum is produced, it will not be entitled to reimbursement. [154] Once allowed Filipinos engaged in commercial fishing to enter into contracts for financial, technical or other forms
petroleum in commercial quantity is discovered, the contractor shall operate the field on behalf of the of assistance with any foreign person, corporation or entity for the production, storage, marketing and
government.[155] processing of fish and fishery/aquatic products.[171]
P.D. No. 87 prescribed minimum terms and conditions for every service contract. [156] It also granted the Presidential Decree No. 705[172] (THE REVISED FORESTRY CODE OF THE PHILIPPINES), approved
contractor certain privileges, including exemption from taxes and payment of tariff duties, [157] and permitted on May 19, 1975, allowed forest products licensees, lessees, or permitees to enter into service contracts for
the repatriation of capital and retention of profits abroad.[158] financial, technical, management, or other forms of assistance . . . with any foreign person or entity for the
Ostensibly, the service contract system had certain advantages over the concession regime. [159] It has exploration, development, exploitation or utilization of the forest resources. [173]
been opined, though, that, in the Philippines, our concept of a service contract, at least in the petroleum Yet another law allowing service contracts, this time for geothermal resources, was Presidential Decree
industry, was basically a concession regime with a production-sharing element.[160] No. 1442,[174] which was signed into law on June 11, 1978. Section 1 thereof authorized the Government to
On January 17, 1973, then President Ferdinand E. Marcos proclaimed the ratification of a new enter into service contracts for the exploration, exploitation and development of geothermal resources with a
Constitution.[161] Article XIV on the National Economy and Patrimony contained provisions similar to the 1935 foreign contractor who must be technically and financially capable of undertaking the operations required in
Constitution with regard to Filipino participation in the nations natural resources. Section 8, Article XIV the service contract.
thereof provides: Thus, virtually the entire range of the countrys natural resources from petroleum and minerals to
SEC. 8. All lands of the public domain, waters, minerals, coal, petroleum and other mineral oils, all forces of geothermal energy, from public lands and forest resources to fishery products was well covered by apparent
potential energy, fisheries, wildlife, and other natural resources of the Philippines belong to the State. With
legal authority to engage in the direct participation or involvement of foreign persons or corporations First, the parties to FTAAs. Only the President, in behalf of the State, may enter into these
(otherwise disqualified) in the exploration and utilization of natural resources through service contracts.[175] agreements, and only with corporations. By contrast, under the 1973 Constitution, a Filipino citizen,
THE 1987 CONSTITUTION AND TECHNICAL corporation or association may enter into a service contract with a foreign person or entity.
OR FINANCIAL ASSISTANCE AGREEMENTS Second, the size of the activities: only large-scale exploration, development, and utilization is allowed.
After the February 1986 Edsa Revolution, Corazon C. Aquino took the reins of power under a The term large-scale usually refers to very capital-intensive activities.[183]
revolutionary government. On March 25, 1986, President Aquino issued Proclamation No. Third, the natural resources subject of the activities is restricted to minerals, petroleum and other
3,[176] promulgating the Provisional Constitution, more popularly referred to as the Freedom Constitution. By mineral oils, the intent being to limit service contracts to those areas where Filipino capital may not be
authority of the same Proclamation, the President created a Constitutional Commission (CONCOM) to draft sufficient.[184]
a new constitution, which took effect on the date of its ratification on February 2, 1987.[177] Fourth, consistency with the provisions of statute. The agreements must be in accordance with the
The 1987 Constitution retained the Regalian doctrine. The first sentence of Section 2, Article XII terms and conditions provided by law.
states: All lands of the public domain, waters, minerals, coal, petroleum, and other mineral oils, all forces of Fifth, Section 2 prescribes certain standards for entering into such agreements. The agreements must
potential energy, fisheries, forests or timber, wildlife, flora and fauna, and other natural resources are owned be based on real contributions to economic growth and general welfare of the country.
by the State. Sixth, the agreements must contain rudimentary stipulations for the promotion of the development
Like the 1935 and 1973 Constitutions before it, the 1987 Constitution, in the second sentence of the and use of local scientific and technical resources.
same provision, prohibits the alienation of natural resources, except agricultural lands. Seventh, the notification requirement. The President shall notify Congress of every financial or
The third sentence of the same paragraph is new: The exploration, development and utilization of technical assistance agreement entered into within thirty days from its execution.
natural resources shall be under the full control and supervision of the State. The constitutional policy of Finally, the scope of the agreements. While the 1973 Constitution referred to service contracts for
the States full control and supervision over natural resources proceeds from the concept of jura regalia, as financial, technical, management, or other forms of assistance the 1987 Constitution provides for
well as the recognition of the importance of the countrys natural resources, not only for national economic agreements. . . involving either financial or technical assistance. It bears noting that the phrases service
development, but also for its security and national defense. [178] Under this provision, the State assumes a contracts and management or other forms of assistance in the earlier constitution have been omitted.
more dynamic role in the exploration, development and utilization of natural resources. [179] By virtue of her legislative powers under the Provisional Constitution, [185] President Aquino, on July 10,
Conspicuously absent in Section 2 is the provision in the 1935 and 1973 Constitutions authorizing the 1987, signed into law E.O. No. 211 prescribing the interim procedures in the processing and approval of
State to grant licenses, concessions, or leases for the exploration, exploitation, development, or utilization of applications for the exploration, development and utilization of minerals. The omission in the 1987
natural resources. By such omission, the utilization of inalienable lands of public domain through license, Constitution of the term service contracts notwithstanding, the said E.O. still referred to them in Section 2
concession or lease is no longer allowed under the 1987 Constitution. [180] thereof:
Having omitted the provision on the concession system, Section 2 proceeded to introduce unfamiliar SEC. 2. Applications for the exploration, development and utilization of mineral resources, including renewal
language:[181] applications and applications for approval of operating agreements and mining service contracts, shall be
The State may directly undertake such activities or it may enter into co-production, joint venture, or accepted and processed and may be approved x x x. [Emphasis supplied.]
production-sharing agreements with Filipino citizens, or corporations or associations at least sixty per The same law provided in its Section 3 that the processing, evaluation and approval of all mining
centum of whose capital is owned by such citizens. applications . . . operating agreements and service contracts . . . shall be governed by Presidential Decree
Consonant with the States full supervision and control over natural resources, Section 2 offers the No. 463, as amended, other existing mining laws, and their implementing rules and regulations. . . .
State two options.[182] One, the State may directly undertake these activities itself; or two, it may enter into As earlier stated, on the 25th also of July 1987, the President issued E.O. No. 279 by authority of which
co-production, joint venture, or production-sharing agreements with Filipino citizens, or entities at least 60% the subject WMCP FTAA was executed on March 30, 1995.
of whose capital is owned by such citizens. On March 3, 1995, President Ramos signed into law R.A. No. 7942. Section 15 thereof declares that
A third option is found in the third paragraph of the same section: the Act shall govern the exploration, development, utilization, and processing of all mineral resources. Such
The Congress may, by law, allow small-scale utilization of natural resources by Filipino citizens, as well as declaration notwithstanding, R.A. No. 7942 does not actually cover all the modes through which the State
cooperative fish farming, with priority to subsistence fishermen and fish-workers in rivers, lakes, bays, and may undertake the exploration, development, and utilization of natural resources.
lagoons. The State, being the owner of the natural resources, is accorded the primary power and responsibility
While the second and third options are limited only to Filipino citizens or, in the case of the former, to in the exploration, development and utilization thereof. As such, it may undertake these activities through
corporations or associations at least 60% of the capital of which is owned by Filipinos, a fourth allows the four modes:
participation of foreign-owned corporations. The fourth and fifth paragraphs of Section 2 provide: The State may directly undertake such activities.
The President may enter into agreements with foreign-owned corporations involving either technical or (2) The State may enter into co-production, joint venture or production-sharing agreements with Filipino
financial assistance for large-scale exploration, development, and utilization of minerals, petroleum, and citizens or qualified corporations.
other mineral oils according to the general terms and conditions provided by law, based on real contributions (3) Congress may, by law, allow small-scale utilization of natural resources by Filipino citizens.
to the economic growth and general welfare of the country. In such agreements, the State shall promote the (4) For the large-scale exploration, development and utilization of minerals, petroleum and other
development and use of local scientific and technical resources. mineral oils, the President may enter into agreements with foreign-owned corporations involving technical or
The President shall notify the Congress of every contract entered into in accordance with this provision, financial assistance.[186]
within thirty days from its execution. Except to charge the Mines and Geosciences Bureau of the DENR with performing researches and
Although Section 2 sanctions the participation of foreign-owned corporations in the exploration, surveys,[187] and a passing mention of government-owned or controlled corporations,[188] R.A. No. 7942 does
development, and utilization of natural resources, it imposes certain limitations or conditions to agreements not specify how the State should go about the first mode. The third mode, on the other hand, is governed by
with such corporations. Republic Act No. 7076[189] (the Peoples Small-Scale Mining Act of 1991) and other pertinent laws. [190] R.A.
No. 7942 primarily concerns itself with the second and fourth modes.
Mineral production sharing, co-production and joint venture agreements are collectively classified by Petitioners argue that E.O. No. 279, the law in force when the WMC FTAA was executed, did not come
R.A. No. 7942 as mineral agreements.[191] The Government participates the least in a mineral production into effect.
sharing agreement (MPSA). In an MPSA, the Government grants the contractor [192] the exclusive right to E.O. No. 279 was signed into law by then President Aquino on July 25, 1987, two days before the
conduct mining operations within a contract area [193] and shares in the gross output.[194] The MPSA opening of Congress on July 27, 1987.[214] Section 8 of the E.O. states that the same shall take
contractor provides the financing, technology, management and personnel necessary for the agreements effect immediately. This provision, according to petitioners, runs counter to Section 1 of E.O. No.
implementation.[195] The total government share in an MPSA is the excise tax on mineral products under 200,[215] which provides:
Republic Act No. 7729,[196] amending Section 151(a) of the National Internal Revenue Code, as SECTION 1. Laws shall take effect after fifteen days following the completion of their publication either
amended.[197] in the Official Gazette or in a newspaper of general circulation in the Philippines, unless it is otherwise
In a co-production agreement (CA),[198] the Government provides inputs to the mining operations other provided.[216] [Emphasis supplied.]
than the mineral resource,[199] while in a joint venture agreement (JVA), where the Government enjoys the On that premise, petitioners contend that E.O. No. 279 could have only taken effect fifteen days after
greatest participation, the Government and the JVA contractor organize a company with both parties having its publication at which time Congress had already convened and the Presidents power to legislate had
equity shares.[200] Aside from earnings in equity, the Government in a JVA is also entitled to a share in the ceased.
gross output.[201] The Government may enter into a CA[202] or JVA[203] with one or more contractors. The Respondents, on the other hand, counter that the validity of E.O. No. 279 was settled in Miners
Governments share in a CA or JVA is set out in Section 81 of the law: Association of the Philippines v. Factoran, supra. This is of course incorrect for the issue in Miners
The share of the Government in co-production and joint venture agreements shall be negotiated by the Association was not the validity of E.O. No. 279 but that of DAO Nos. 57 and 82 which were issued pursuant
Government and the contractor taking into consideration the: (a) capital investment of the project, (b) the thereto.
risks involved, (c) contribution of the project to the economy, and (d) other factors that will provide for a fair Nevertheless, petitioners contentions have no merit.
and equitable sharing between the Government and the contractor. The Government shall also be entitled to It bears noting that there is nothing in E.O. No. 200 that prevents a law from taking effect on a date
compensations for its other contributions which shall be agreed upon by the parties, and shall consist, other than even before the 15-day period after its publication. Where a law provides for its own date of
among other things, the contractors income tax, excise tax, special allowance, withholding tax due from the effectivity, such date prevails over that prescribed by E.O. No. 200. Indeed, this is the very essence of the
contractors foreign stockholders arising from dividend or interest payments to the said foreign stockholders, phrase unless it is otherwise provided in Section 1 thereof.Section 1, E.O. No. 200, therefore, applies only
in case of a foreign national and all such other taxes, duties and fees as provided for under existing laws. when a statute does not provide for its own date of effectivity.
All mineral agreements grant the respective contractors the exclusive right to conduct mining What is mandatory under E.O. No. 200, and what due process requires, as this Court held in Taada v.
operations and to extract all mineral resources found in the contract area. [204] A qualified person may enter Tuvera,[217] is the publication of the law for
into any of the mineral agreements with the Government.[205] A qualified person is without such notice and publication, there would be no basis for the application of the maxim ignorantia legis
any citizen of the Philippines with capacity to contract, or a corporation, partnership, association, or n[eminem] excusat. It would be the height of injustice to punish or otherwise burden a citizen for the
cooperative organized or authorized for the purpose of engaging in mining, with technical and financial transgression of a law of which he had no notice whatsoever, not even a constructive one.
capability to undertake mineral resources development and duly registered in accordance with law at least While the effectivity clause of E.O. No. 279 does not require its publication, it is not a ground for its
sixty per centum (60%) of the capital of which is owned by citizens of the Philippines x x x. [206] invalidation since the Constitution, being the fundamental, paramount and supreme law of the nation, is
The fourth mode involves financial or technical assistance agreements. An FTAA is defined as a deemed written in the law.[218] Hence, the due process clause,[219] which, so Taada held, mandates the
contract involving financial or technical assistance for large-scale exploration, development, and utilization of publication of statutes, is read into Section 8 of E.O. No. 279.Additionally, Section 1 of E.O. No. 200 which
natural resources.[207] Any qualified person with technical and financial capability to undertake large-scale provides for publication either in the Official Gazette or in a newspaper of general circulation in the
exploration, development, and utilization of natural resources in the Philippines may enter into such Philippines, finds suppletory application. It is significant to note that E.O. No. 279 was actually published in
agreement directly with the Government through the DENR. [208] For the purpose of granting an FTAA, a the Official Gazette[220] on August 3, 1987.
legally organized foreign-owned corporation (any corporation, partnership, association, or cooperative duly From a reading then of Section 8 of E.O. No. 279, Section 1 of E.O. No. 200, and Taada v. Tuvera, this
registered in accordance with law in which less than 50% of the capital is owned by Filipino citizens) [209] is Court holds that E.O. No. 279 became effective immediately upon its publication in the Official Gazette on
deemed a qualified person.[210] August 3, 1987.
Other than the difference in contractors qualifications, the principal distinction between mineral That such effectivity took place after the convening of the first Congress is irrelevant. At the time
agreements and FTAAs is the maximum contract area to which a qualified person may hold or be President Aquino issued E.O. No. 279 on July 25, 1987, she was still validly exercising legislative powers
granted.[211] Large-scale under R.A. No. 7942 is determined by the size of the contract area, as opposed to under the Provisional Constitution.[221] Article XVIII (Transitory Provisions) of the 1987 Constitution explicitly
the amount invested (US $50,000,000.00), which was the standard under E.O. 279. states:
Like a CA or a JVA, an FTAA is subject to negotiation. [212] The Governments contributions, in the form SEC. 6. The incumbent President shall continue to exercise legislative powers until the first Congress is
of taxes, in an FTAA is identical to its contributions in the two mineral agreements, save that in an FTAA: convened.
The collection of Government share in financial or technical assistance agreement shall commence after the The convening of the first Congress merely precluded the exercise of legislative powers by President
financial or technical assistance agreement contractor has fully recovered its pre-operating expenses, Aquino; it did not prevent the effectivity of laws she had previously enacted.
exploration, and development expenditures, inclusive. [213] There can be no question, therefore, that E.O. No. 279 is an effective, and a validly enacted,
III statute.
Having examined the history of the constitutional provision and statutes enacted pursuant thereto, a THE CONSTITUTIONALITY
consideration of the substantive issues presented by the petition is now in order. OF THE WMCP FTAA
THE EFFECTIVITY OF Petitioners submit that, in accordance with the text of Section 2, Article XII of the Constitution, FTAAs
EXECUTIVE ORDER NO. 279 should be limited to technical or financial assistance only. They observe, however, that, contrary to the
language of the Constitution, the WMCP FTAA allows WMCP, a fully foreign-owned mining corporation, to
extend more than mere financial or technical assistance to the State, for it permits WMCP to manage and taking into account that Filipinos should have real control over our economy and patrimony, and if foreign
operate every aspect of the mining activity. [222] equity is permitted, the same must be subordinated to the imperative demands of the national interest.
Petitioners submission is well-taken. It is a cardinal rule in the interpretation of constitutions that the x x x.
instrument must be so construed as to give effect to the intention of the people who adopted it. [223] This It is also my understanding that service contracts involving foreign corporations or entities are
intention is to be sought in the constitution itself, and the apparent meaning of the words is to be taken as resorted to only when no Filipino enterprise or Filipino-controlled enterprise could possibly
expressing it, except in cases where that assumption would lead to absurdity, ambiguity, or undertake the exploration or exploitation of our natural resources and that compensation under
contradiction.[224] What the Constitution says according to the text of the provision, therefore, compels such contracts cannot and should not equal what should pertain to ownership of capital. In other
acceptance and negates the power of the courts to alter it, based on the postulate that the framers and the words, the service contract should not be an instrument to circumvent the basic provision, that the
people mean what they say.[225] Accordingly, following the literal text of the Constitution, assistance exploration and exploitation of natural resources should be truly for the benefit of Filipinos.
accorded by foreign-owned corporations in the large-scale exploration, development, and utilization of Thank you, and I vote yes.[233] [Emphasis supplied.]
petroleum, minerals and mineral oils should be limited to technical or financial assistance only. x x x.
WMCP nevertheless submits that the word technical in the fourth paragraph of Section 2 of E.O. No. MR. TADEO. Nais ko lamang ipaliwanag ang aking boto.
279 encompasses a broad number of possible services, perhaps, scientific and/or technological in Matapos suriin ang kalagayan ng Pilipinas, ang saligang suliranin, pangunahin ang salitang
basis.[226] It thus posits that it may also well include the area of management or operations . . . so long as imperyalismo. Ang ibig sabihin nito ay ang sistema ng lipunang pinaghaharian ng iilang monopolyong
such assistance requires specialized knowledge or skills, and are related to the exploration, development kapitalista at ang salitang imperyalismo ay buhay na buhay sa National Economy and Patrimony na nating
and utilization of mineral resources.[227] ginawa. Sa pamamagitan ng salitang based on, naroroon na ang free trade sapagkat tayo ay mananatiling
This Court is not persuaded. As priorly pointed out, the phrase management or other forms of tagapagluwas ng hilaw na sangkap at tagaangkat ng yaring produkto. Pangalawa, naroroon pa rin ang
assistance in the 1973 Constitution was deleted in the 1987 Constitution, which allows only technical or parity rights, ang service contract, ang 60-40 equity sa natural resources. Habang naghihirap ang
financial assistance. Casus omisus pro omisso habendus est. A person, object or thing omitted from an sambayanang Pilipino, ginagalugad naman ng mga dayuhan ang ating likas na yaman. Kailan man
enumeration must be held to have been omitted intentionally. [228] As will be shown later, the management or ang Article on National Economy and Patrimony ay hindi nagpaalis sa pagkaalipin ng ating
operation of mining activities by foreign contractors, which is the primary feature of service contracts, was ekonomiya sa kamay ng mga dayuhan. Ang solusyon sa suliranin ng bansa ay dalawa lamang: ang
precisely the evil that the drafters of the 1987 Constitution sought to eradicate. pagpapatupad ng tunay na reporma sa lupa at ang national industrialization. Ito ang tinatawag naming
Respondents insist that agreements involving technical or financial assistance is just another term for pagsikat ng araw sa Silangan. Ngunit ang mga landlords and big businessmen at ang mga komprador ay
service contracts. They contend that the proceedings of the CONCOM indicate that although the terminology nagsasabi na ang free trade na ito, ang kahulugan para sa amin, ay ipinipilit sa ating sambayanan na ang
service contract was avoided [by the Constitution], the concept it represented was not. They add that [t]he araw ay sisikat sa Kanluran. Kailan man hindi puwedeng sumikat ang araw sa Kanluran. I vote
concept is embodied in the phrase agreements involving financial or technical assistance. [229] And point out no.[234] [Emphasis supplied.]
how members of the CONCOM referred to these agreements as service contracts. For instance: This Court is likewise not persuaded.
SR. TAN. Am I correct in thinking that the only difference between these future service contracts and the As earlier noted, the phrase service contracts has been deleted in the 1987 Constitutions Article on
past service contracts under Mr. Marcos is the general law to be enacted by the legislature and the National Economy and Patrimony. If the CONCOM intended to retain the concept of service contracts under
notification of Congress by the President? That is the only difference, is it not? the 1973 Constitution, it could have simply adopted the old terminology (service contracts) instead of
MR. VILLEGAS. That is right. employing new and unfamiliar terms (agreements . . . involving either technical or financial assistance). Such
SR. TAN. So those are the safeguards[?] a difference between the language of a provision in a revised constitution and that of a similar provision in
MR. VILLEGAS. Yes. There was no law at all governing service contracts before. the preceding constitution is viewed as indicative of a difference in purpose.[235] If, as respondents suggest,
SR. TAN. Thank you, Madam President.[230] [Emphasis supplied.] the concept of technical or financial assistance agreements is identical to that of service contracts, the
WMCP also cites the following statements of Commissioners Gascon, Garcia, Nolledo and Tadeo who CONCOM would not have bothered to fit the same dog with a new collar. To uphold respondents theory
alluded to service contracts as they explained their respective votes in the approval of the draft Article: would reduce the first to a mere euphemism for the second and render the change in phraseology
MR. GASCON. Mr. Presiding Officer, I vote no primarily because of two reasons: One, the provision on meaningless.
service contracts. I felt that if we would constitutionalize any provision on service contracts, this should An examination of the reason behind the change confirms that technical or financial assistance
always be with the concurrence of Congress and not guided only by a general law to be promulgated by agreements are not synonymous to service contracts.
Congress. x x x.[231] [Emphasis supplied.] [T]he Court in construing a Constitution should bear in mind the object sought to be accomplished by its
x x x. adoption, and the evils, if any, sought to be prevented or remedied. A doubtful provision will be examined in
MR. GARCIA. Thank you. light of the history of the times, and the condition and circumstances under which the Constitution was
I vote no. x x x. framed. The object is to ascertain the reason which induced the framers of the Constitution to enact the
Service contracts are given constitutional legitimization in Section 3, even when they have been particular provision and the purpose sought to be accomplished thereby, in order to construe the whole as to
proven to be inimical to the interests of the nation, providing as they do the legal loophole for the make the words consonant to that reason and calculated to effect that purpose. [236]
exploitation of our natural resources for the benefit of foreign interests. They constitute a serious As the following question of Commissioner Quesada and Commissioner Villegas answer shows the
negation of Filipino control on the use and disposition of the nations natural resources, especially with regard drafters intended to do away with service contracts which were used to circumvent the capitalization (60%-
to those which are nonrenewable.[232] [Emphasis supplied.] 40%) requirement:
xxx MS. QUESADA. The 1973 Constitution used the words service contracts. In this particular Section 3, is there
MR. NOLLEDO. While there are objectionable provisions in the Article on National Economy and Patrimony, a safeguard against the possible control of foreign interests if the Filipinos go into coproduction with them?
going over said provisions meticulously, setting aside prejudice and personalities will reveal that the article MR. VILLEGAS. Yes. In fact, the deletion of the phrase service contracts was our first attempt to avoid
contains a balanced set of provisions. I hope the forthcoming Congress will implement such provisions some of the abuses in the past regime in the use of service contracts to go around the 60-40
arrangement. The safeguard that has been introduced and this, of course can be refined is found in Section I voted in favor of the Jamir proposal because it is not really exploitation that we granted to the alien
3, lines 25 to 30, where Congress will have to concur with the President on any agreement entered into corporations but only for them to render financial or technical assistance. It is not for them to enjoy
between a foreign-owned corporation and the government involving technical or financial assistance for our natural resources. Madam President, our natural resources are depleting; our population is increasing
large-scale exploration, development and utilization of natural resources.[237] [Emphasis supplied.] by leaps and bounds. Fifty years from now, if we will allow these aliens to exploit our natural resources,
In a subsequent discussion, Commissioner Villegas allayed the fears of Commissioner Quesada there will be no more natural resources for the next generations of Filipinos. It may last long if we will begin
regarding the participation of foreign interests in Philippine natural resources, which was supposed to be now. Since 1935 the aliens have been allowed to enjoy to a certain extent the exploitation of our natural
restricted to Filipinos. resources, and we became victims of foreign dominance and control. The aliens are interested in coming to
MS. QUESADA. Another point of clarification is the phrase and utilization of natural resources shall be under the Philippines because they would like to enjoy the bounty of nature exclusively intended for Filipinos by
the full control and supervision of the State. In the 1973 Constitution, this was limited to citizens of the God.
Philippines; but it was removed and substituted by shall be under the full control and supervision of the And so I appeal to all, for the sake of the future generations, that if we have to pray in the Preamble to
State. Was the concept changed so that these particular resources would be limited to citizens of the preserve and develop the national patrimony for the sovereign Filipino people and for the generations to
Philippines? Or would these resources only be under the full control and supervision of the State; meaning, come, we must at this time decide once and for all that our natural resources must be reserved only to
noncitizens would have access to these natural resources? Is that the understanding? Filipino citizens.
MR. VILLEGAS. No, Mr. Vice-President, if the Commissioner reads the next sentence, it states: Thank you.[239] [Emphasis supplied.]
Such activities may be directly undertaken by the State, or it may enter into co-production, joint venture, The opinion of another member of the CONCOM is persuasive [240] and leaves no doubt as to the
production-sharing agreements with Filipino citizens. intention of the framers to eliminate service contracts altogether. He writes:
So we are still limiting it only to Filipino citizens. Paragraph 4 of Section 2 specifies large-scale, capital-intensive, highly technological undertakings for which
x x x. the President may enter into contracts with foreign-owned corporations, and enunciates strict conditions that
MS. QUESADA. Going back to Section 3, the section suggests that: should govern such contracts. x x x.
The exploration, development, and utilization of natural resources may be directly undertaken by the State, This provision balances the need for foreign capital and technology with the need to maintain the national
or it may enter into co-production, joint venture or production-sharing agreement with . . . corporations or sovereignty. It recognizes the fact that as long as Filipinos can formulate their own terms in their own
associations at least sixty per cent of whose voting stock or controlling interest is owned by such citizens. territory, there is no danger of relinquishing sovereignty to foreign interests.
Lines 25 to 30, on the other hand, suggest that in the large-scale exploration, development and utilization of Are service contracts allowed under the new Constitution? No. Under the new Constitution, foreign
natural resources, the President with the concurrence of Congress may enter into agreements with foreign- investors (fully alien-owned) can NOT participate in Filipino enterprises except to provide: (1)
owned corporations even for technical or financial assistance. Technical Assistance for highly technical enterprises; and (2) Financial Assistance for large-scale
I wonder if this part of Section 3 contradicts the second part. I am raising this point for fear that foreign enterprises.
investors will use their enormous capital resources to facilitate the actual exploitation or exploration, The intent of this provision, as well as other provisions on foreign investments, is to prevent the
development and effective disposition of our natural resources to the detriment of Filipino investors. I am not practice (prevalent in the Marcos government) of skirting the 60/40 equation using the cover of
saying that we should not consider borrowing money from foreign sources. What I refer to is that foreign service contracts.[241] [Emphasis supplied.]
interest should be allowed to participate only to the extent that they lend us money and give us technical Furthermore, it appears that Proposed Resolution No. 496,[242] which was the draft Article on National
assistance with the appropriate government permit. In this way, we can insure the enjoyment of our natural Economy and Patrimony, adopted the concept of agreements . . . involving either technical or financial
resources by our own people. assistance contained in the Draft of the 1986 U.P. Law Constitution Project (U.P. Law draft) which was taken
MR. VILLEGAS. Actually, the second provision about the President does not permit foreign investors into consideration during the deliberation of the CONCOM. [243] The former, as well as Article XII, as adopted,
to participate. It is only technical or financial assistance they do not own anything but on conditions employed the same terminology, as the comparative table below shows:
that have to be determined by law with the concurrence of Congress. So, it is very restrictive. PROPOSED RESOLUTION NO. ARTICLE XII OF THE 1987
If the Commissioner will remember, this removes the possibility for service contracts which we said 496 OF THE CONSTITUTIONAL CONSTITUTION
yesterday were avenues used in the previous regime to go around the 60-40 DRAFT OF THE UP LAW COMMISSION
requirement.[238][Emphasis supplied.] CONSTITUTION PROJECT
The present Chief Justice, then a member of the CONCOM, also referred to this limitation in scope in SEC. 1. All lands of the public SEC. 3. All lands of the public SEC. 2. All lands of the public
proposing an amendment to the 60-40 requirement: domain, waters, minerals, coal, domain, waters, minerals, coal, domain, waters, minerals, coal,
MR. DAVIDE. May I be allowed to explain the proposal? petroleum and other mineral oils, petroleum and other mineral oils, petroleum, and other mineral oils,
MR. MAAMBONG. Subject to the three-minute rule, Madam President. all forces of potential energy, all forces of potential energy, all forces of potential energy,
MR. DAVIDE. It will not take three minutes. fisheries, flora and fauna and fisheries, forests, flora and fauna, fisheries, forests or timber,
The Commission had just approved the Preamble. In the Preamble we clearly stated that the Filipino other natural resources of the and other natural resources are wildlife, flora and fauna, and other
people are sovereign and that one of the objectives for the creation or establishment of a Philippines are owned by the owned by the State. With the natural resources are owned by
government is to conserve and develop the national patrimony. The implication is that the national State. With the exception of exception of agricultural lands, all the State. With the exception of
patrimony or our natural resources are exclusively reserved for the Filipino people. No alien must be agricultural lands, all other natural other natural resources shall not agricultural lands, all other natural
allowed to enjoy, exploit and develop our natural resources. As a matter of fact, that principle resources shall not be be alienated. The exploration, resources shall not be
proceeds from the fact that our natural resources are gifts from God to the Filipino people and it alienated. The exploration, development, and utilization of alienated. The exploration,
would be a breach of that special blessing from God if we will allow aliens to exploit our natural development and utilization of natural resources shall be under development, and utilization of
resources. natural resources shall be under the full control and supervision of natural resources shall be under
the full control and supervision of the State. Such activities may be the full control and supervision of
the State. Such activities may be directly undertaken by the State, the State. The State may directly resources.[Emphasis supplied.]
directly undertaken by the state, or it may enter into co-production, undertake such activities or it may The President shall notify the
or it may enter into co-production, joint venture, production-sharing enter into co-production, joint Congress of every contract
joint venture, production sharing agreements with Filipino citizens venture, or production-sharing entered into in accordance with
agreements with Filipino citizens or corporations or associations at agreements with Filipino citizens, this provision, within thirty days
or corporations or associations least sixty per cent of whose or corporations or associations at from its execution.
sixty per cent of whose voting voting stock or controlling interest least sixty per centum of whose
stock or controlling interest is is owned by such citizens. Such capital is owned by such The insights of the proponents of the U.P. Law draft are, therefore, instructive in interpreting the phrase
owned by such citizens for a agreements shall be for a period citizens. Such agreements may technical or financial assistance.
period of not more than twenty- of twenty-five years, renewable be for a period not exceeding In his position paper entitled Service Contracts: Old Wine in New Bottles?, Professor Pacifico A.
five years, renewable for not more for not more than twenty-five twenty-five years, renewable for Agabin, who was a member of the working group that prepared the U.P. Law draft, criticized service
than twenty-five years and under years, and under such term and not more than twenty-five years, contracts for they lodge exclusive management and control of the enterprise to the service contractor, which
such terms and conditions as may conditions as may be provided by and under such terms and is reminiscent of the old concession regime. Thus, notwithstanding the provision of the Constitution that
be provided by law. In case as to law. In cases of water rights for conditions as may be provided by natural resources belong to the State, and that these shall not be alienated, the service contract system
water rights for irrigation, water irrigation, water supply, fisheries law. In case of water rights for renders nugatory the constitutional provisions cited.[244] He elaborates:
supply, fisheries, or industrial or industrial uses other than the irrigation, water supply, fisheries, Looking at the Philippine model, we can discern the following vestiges of the concession regime, thus:
uses other than the development development for water power, or industrial uses other than the 1. Bidding of a selected area, or leasing the choice of the area to the interested party and then negotiating
of water power, beneficial use beneficial use may be the development of water power, the terms and conditions of the contract; (Sec. 5, P.D. 87)
may be the measure and limit of measure and limit of the grant. beneficial use may be the 2. Management of the enterprise vested on the contractor, including operation of the field if
the grant. measure and limit of the grant. petroleum is discovered; (Sec. 8, P.D. 87)
3. Control of production and other matters such as expansion and development; (Sec. 8)
The State shall protect the nations 4. Responsibility for downstream operations marketing, distribution, and processing may be with the
marine wealth in its archipelagic contractor (Sec. 8);
waters, territorial sea, and 5. Ownership of equipment, machinery, fixed assets, and other properties remain with contractor (Sec. 12,
exclusive economic zone, and P.D. 87);
reserve its use and enjoyment 6. Repatriation of capital and retention of profits abroad guaranteed to the contractor (Sec. 13, P.D. 87); and
exclusively to Filipino citizens. 7. While title to the petroleum discovered may nominally be in the name of the government, the
contractor has almost unfettered control over its disposition and sale, and even the domestic
The National Assembly may by The Congress may by law allow The Congress may, by law, allow requirements of the country is relegated to a pro rata basis (Sec. 8).
law allow small scale utilization of small-scale utilization of natural small-scale utilization of natural In short, our version of the service contract is just a rehash of the old concession regime x x x. Some people
natural resources by Filipino resources by Filipino citizens, as resources by Filipino citizens, as have pulled an old rabbit out of a magicians hat, and foisted it upon us as a new and different animal.
citizens. well as cooperative fish farming in well as cooperative fish farming, The service contract as we know it here is antithetical to the principle of sovereignty over our natural
rivers, lakes, bays, and lagoons. with priority to subsistence resources restated in the same article of the [1973] Constitution containing the provision for service
fishermen and fish-workers in contracts. If the service contractor happens to be a foreign corporation, the contract would also run
rivers, lakes, bays, and lagoons. counter to the constitutional provision on nationalization or Filipinization, of the exploitation of our
natural resources.[245] [Emphasis supplied. Underscoring in the original.]
The National Assembly, may, by The President with the The President may enter into Professor Merlin M. Magallona, also a member of the working group, was harsher in his reproach of
two-thirds vote of all its members concurrence of Congress, by agreements with foreign-owned the system:
by special law provide the terms special law, shall provide the corporations involving either x x x the nationalistic phraseology of the 1935 [Constitution] was retained by the [1973] Charter, but the
and conditions under which a terms and conditions under which technical or financial essence of nationalism was reduced to hollow rhetoric. The 1973 Charter still provided that the exploitation
foreign-owned corporation may a foreign-owned corporation may assistance for large-scale or development of the countrys natural resources be limited to Filipino citizens or corporations owned or
enter into agreements with the enter into agreements with the exploration, development, and controlled by them. However, the martial-law Constitution allowed them, once these resources are in their
government involving either government involving either utilization of minerals, petroleum, name, to enter into service contracts with foreign investors for financial, technical, management, or other
technical or financial technical or financial and other mineral oils according forms of assistance. Since foreign investors have the capital resources, the actual exploitation and
assistance for large-scale assistance for large-scale to the general terms and development, as well as the effective disposition, of the countrys natural resources, would be under their
exploration, development, or exploration, development, and conditions provided by law, based direction, and control, relegating the Filipino investors to the role of second-rate partners in joint ventures.
utilization of natural utilization of natural on real contributions to the Through the instrumentality of the service contract, the 1973 Constitution had legitimized at the
resources. [Emphasis supplied.] resources. [Emphasis supplied.] economic growth and general highest level of state policy that which was prohibited under the 1973 Constitution, namely: the
welfare of the country. In such exploitation of the countrys natural resources by foreign nationals. The drastic impact of [this]
agreements, the State shall constitutional change becomes more pronounced when it is considered that the active party to any
promote the development and use service contract may be a corporation wholly owned by foreign interests. In such a case, the
of local scientific and technical
citizenship requirement is completely set aside, permitting foreign corporations to obtain actual draft itself being an admission of such scarcity. Hence, they recommended a compromise to reconcile the
possession, control, and [enjoyment] of the countrys natural resources.[246] [Emphasis supplied.] nationalistic provisions dating back to the 1935 Constitution, which reserved all natural resources exclusively
Accordingly, Professor Agabin recommends that: to Filipinos, and the more liberal 1973 Constitution, which allowed foreigners to participate in these
Recognizing the service contract for what it is, we have to expunge it from the Constitution and resources through service contracts. Such a compromise called for the adoption of a new system in the
reaffirm ownership over our natural resources. That is the only way we can exercise effective control exploration, development, and utilization of natural resources in the form of technical agreements or financial
over our natural resources. agreements which, necessarily, are distinct concepts from service contracts.
This should not mean complete isolation of the countrys natural resources from foreign investment. Other The replacement of service contracts with agreements involving either technical or financial assistance,
contract forms which are less derogatory to our sovereignty and control over natural resources like as well as the deletion of the phrase management or other forms of assistance, assumes greater
technical assistance agreements, financial assistance [agreements], co-production agreements, joint significance when note is taken that the U.P. Law draft proposed other equally crucial changes that were
ventures, production-sharing could still be utilized and adopted without violating constitutional obviously heeded by the CONCOM. These include the abrogation of the concession system and the
provisions. In other words, we can adopt contract forms which recognize and assert our sovereignty adoption of new options for the State in the exploration, development, and utilization of natural
and ownership over natural resources, and where the foreign entity is just a pure contractor instead resources. The proponents deemed these changes to be more consistent with the States ownership of, and
of the beneficial owner of our economic resources.[247] [Emphasis supplied.] its full control and supervision (a phrase also employed by the framers) over, such resources. The Project
Still another member of the working group, Professor Eduardo Labitag, proposed that: explained:
2. Service contracts as practiced under the 1973 Constitution should be discouraged, instead the 3. In line with the State ownership of natural resources, the State should take a more active role in the
government may be allowed, subject to authorization by special law passed by an extraordinary exploration, development, and utilization of natural resources, than the present practice of granting licenses,
majority to enter into either technical or financial assistance. This is justified by the fact that as concessions, or leases hence the provision that said activities shall be under the full control and supervision
presently worded in the 1973 Constitution, a service contract gives full control over the contract area to the of the State. There are three major schemes by which the State could undertake these activities: first,
service contractor, for him to work, manage and dispose of the proceeds or production. It was a subterfuge directly by itself; second, by virtue of co-production, joint venture, production sharing agreements with
to get around the nationality requirement of the constitution.[248] [Emphasis supplied.] Filipino citizens or corporations or associations sixty per cent (60%) of the voting stock or controlling
In the annotations on the proposed Article on National Economy and Patrimony, the U.P. Law draft interests of which are owned by such citizens; or third, with a foreign-owned corporation, in cases of large-
summarized the rationale therefor, thus: scale exploration, development, or utilization of natural resources through agreements involving either
5. The last paragraph is a modification of the service contract provision found in Section 9, Article XIV of the technical or financial assistance only. x x x.
1973 Constitution as amended. This 1973 provision shattered the framework of nationalism in our At present, under the licensing concession or lease schemes, the government benefits from such benefits
fundamental law (see Magallona, Nationalism and its Subversion in the Constitution). Through the service only through fees, charges, ad valorem taxes and income taxes of the exploiters of our natural
contract, the 1973 Constitution had legitimized that which was prohibited under the 1935 constitutionthe resources. Such benefits are very minimal compared with the enormous profits reaped by theses licensees,
exploitation of the countrys natural resources by foreign nationals. Through the service contract, acts grantees, concessionaires. Moreover, some of them disregard the conservation of natural resources and do
prohibited by the Anti-Dummy Law were recognized as legitimate arrangements. Service contracts lodge not protect the environment from degradation. The proposed role of the State will enable it to a greater share
exclusive management and control of the enterprise to the service contractor, not unlike the old in the profits it can also actively husband its natural resources and engage in developmental programs that
concession regime where the concessionaire had complete control over the countrys natural will be beneficial to them.
resources, having been given exclusive and plenary rights to exploit a particular resource and, in 4. Aside from the three major schemes for the exploration, development, and utilization of our natural
effect, having been assured of ownership of that resource at the point of extraction (see Agabin, resources, the State may, by law, allow Filipino citizens to explore, develop, utilize natural resources in
Service Contracts: Old Wine in New Bottles). Service contracts, hence, are antithetical to the principle of small-scale. This is in recognition of the plight of marginal fishermen, forest dwellers, gold panners, and
sovereignty over our natural resources, as well as the constitutional provision on nationalization or others similarly situated who exploit our natural resources for their daily sustenance and survival.[250]
Filipinization of the exploitation of our natural resources. Professor Agabin, in particular, after taking pains to illustrate the similarities between the two systems,
Under the proposed provision, only technical assistance or financial assistance agreements may be concluded that the service contract regime was but a rehash of the concession system. Old wine in new
entered into, and only for large-scale activities. These are contract forms which recognize and assert bottles, as he put it. The rejection of the service contract regime, therefore, is in consonance with the
our sovereignty and ownership over natural resources since the foreign entity is just a pure abolition of the concession system.
contractor and not a beneficial owner of our economic resources. The proposal recognizes the need In light of the deliberations of the CONCOM, the text of the Constitution, and the adoption of other
for capital and technology to develop our natural resources without sacrificing our sovereignty and proposed changes, there is no doubt that the framers considered and shared the intent of the U.P. Law
control over such resources by the safeguard of a special law which requires two-thirds vote of all proponents in employing the phrase agreements . . . involving either technical or financial assistance.
the members of the Legislature. This will ensure that such agreements will be debated upon exhaustively While certain commissioners may have mentioned the term service contracts during the CONCOM
and thoroughly in the National Assembly to avert prejudice to the nation.[249] [Emphasis supplied.] deliberations, they may not have been necessarily referring to the concept of service contracts under the
The U.P. Law draft proponents viewed service contracts under the 1973 Constitution as grants of 1973 Constitution. As noted earlier, service contracts is a term that assumes different meanings to different
beneficial ownership of the countrys natural resources to foreign owned corporations.While, in theory, the people.[251] The commissioners may have been using the term loosely, and not in its technical and legal
State owns these natural resources and Filipino citizens, their beneficiaries service contracts actually vested sense, to refer, in general, to agreements concerning natural resources entered into by the Government with
foreigners with the right to dispose, explore for, develop, exploit, and utilize the same. Foreigners, not foreign corporations. These loose statements do not necessarily translate to the adoption of the 1973
Filipinos, became the beneficiaries of Philippine natural resources. This arrangement is clearly incompatible Constitution provision allowing service contracts.
with the constitutional ideal of nationalization of natural resources, with the Regalian doctrine, and on a It is true that, as shown in the earlier quoted portions of the proceedings in CONCOM, in response to
broader perspective, with Philippine sovereignty. Sr. Tans question, Commissioner Villegas commented that, other than congressional notification, the only
The proponents nevertheless acknowledged the need for capital and technical know-how in the large- difference between future and past service contracts is the requirement of a general law as there were no
scale exploitation, development and utilization of natural resources the second paragraph of the proposed laws previously authorizing the same.[252]However, such remark is far outweighed by his more categorical
statement in his exchange with Commissioner Quesada that the draft article does not permit foreign A foreign-owned/-controlled corporation may likewise be granted a mineral processing
investors to participate in the nations natural resources which was exactly what service contracts did except permit.[270] Mineral processing is the milling, beneficiation or upgrading of ores or minerals and rocks or by
to provide technical or financial assistance.[253] similar means to convert the same into marketable products. [271]
In the case of the other commissioners, Commissioner Nolledo himself clarified in his work that the An FTAA contractor makes a warranty that the mining operations shall be conducted in accordance
present charter prohibits service contracts.[254] Commissioner Gascon was not totally averse to foreign with the provisions of R.A. No. 7942 and its implementing rules [272] and for work programs and minimum
participation, but favored stricter restrictions in the form of majority congressional concurrence.[255] On the expenditures and commitments.[273] And it obliges itself to furnish the Government records of geologic,
other hand, Commissioners Garcia and Tadeo may have veered to the extreme side of the spectrum and accounting, and other relevant data for its mining operation. [274]
their objections may be interpreted as votes against any foreign participation in our natural resources Mining operation, as the law defines it, means mining activities involving exploration, feasibility,
whatsoever. development, utilization, and processing.[275]
WMCP cites Opinion No. 75, s. 1987,[256] and Opinion No. 175, s. 1990[257] of the Secretary of Justice, The underlying assumption in all these provisions is that the foreign contractor manages the mineral
expressing the view that a financial or technical assistance agreement is no different in concept from the resources, just like the foreign contractor in a service contract.
service contract allowed under the 1973 Constitution. This Court is not, however, bound by this Furthermore, Chapter XII of the Act grants foreign contractors in FTAAs the same auxiliary mining
interpretation. When an administrative or executive agency renders an opinion or issues a statement of rights that it grants contractors in mineral agreements (MPSA, CA and JV). [276]Parenthetically, Sections 72 to
policy, it merely interprets a pre-existing law; and the administrative interpretation of the law is at best 75 use the term contractor, without distinguishing between FTAA and mineral agreement contractors. And
advisory, for it is the courts that finally determine what the law means. [258] so does holders of mining rights in Section 76. A foreign contractor may even convert its FTAA into a mineral
In any case, the constitutional provision allowing the President to enter into FTAAs with foreign-owned agreement if the economic viability of the contract area is found to be inadequate to justify large-scale
corporations is an exception to the rule that participation in the nations natural resources is reserved mining operations,[277]provided that it reduces its equity in the corporation, partnership, association or
exclusively to Filipinos. Accordingly, such provision must be construed strictly against their enjoyment by cooperative to forty percent (40%).[278]
non-Filipinos. As Commissioner Villegas emphasized, the provision is very restrictive. [259] Commissioner Finally, under the Act, an FTAA contractor warrants that it has or has access to all the
Nolledo also remarked that entering into service contracts is an exception to the rule on protection of natural financing, managerial, and technical expertise. . . .[279] This suggests that an FTAA contractor is bound to
resources for the interest of the nation and, therefore, being an exception, it should be subject, whenever provide some management assistance a form of assistance that has been eliminated and, therefore,
possible, to stringent rules.[260] Indeed, exceptions should be strictly but reasonably construed; they extend proscribed by the present Charter.
only so far as their language fairly warrants and all doubts should be resolved in favor of the general By allowing foreign contractors to manage or operate all the aspects of the mining operation, the
provision rather than the exception.[261] above-cited provisions of R.A. No. 7942 have in effect conveyed beneficial ownership over the nations
With the foregoing discussion in mind, this Court finds that R.A. No. 7942 is invalid insofar as said Act mineral resources to these contractors, leaving the State with nothing but bare title thereto.
authorizes service contracts. Although the statute employs the phrase financial and technical agreements in Moreover, the same provisions, whether by design or inadvertence, permit a circumvention of the
accordance with the 1987 Constitution, it actually treats these agreements as service contracts that grant constitutionally ordained 60%-40% capitalization requirement for corporations or associations engaged in
beneficial ownership to foreign contractors contrary to the fundamental law. the exploitation, development and utilization of Philippine natural resources.
Section 33, which is found under Chapter VI (Financial or Technical Assistance Agreement) of R.A. No. In sum, the Court finds the following provisions of R.A. No. 7942 to be violative of Section 2, Article XII
7942 states: of the Constitution:
SEC. 33. Eligibility.Any qualified person with technical and financial capability to undertake large-scale (1) The proviso in Section 3 (aq), which defines qualified person, to wit:
exploration, development, and utilization of mineral resources in the Philippines may enter into a Provided, That a legally organized foreign-owned corporation shall be deemed a qualified person for
financial or technical assistance agreement directly with the Government through the purposes of granting an exploration permit, financial or technical assistance agreement or mineral
Department. [Emphasis supplied.] processing permit.
Exploration, as defined by R.A. No. 7942, (2) Section 23,[280] which specifies the rights and obligations of an exploration permittee, insofar as said
means the searching or prospecting for mineral resources by geological, geochemical or geophysical section applies to a financial or technical assistance agreement,
surveys, remote sensing, test pitting, trending, drilling, shaft sinking, tunneling or any other means for the (3) Section 33, which prescribes the eligibility of a contractor in a financial or technical assistance
purpose of determining the existence, extent, quantity and quality thereof and the feasibility of mining them agreement;
for profit.[262] (4) Section 35,[281] which enumerates the terms and conditions for every financial or technical
A legally organized foreign-owned corporation may be granted an exploration permit,[263] which vests it with assistance agreement;
the right to conduct exploration for all minerals in specified areas, [264] i.e., to enter, occupy and explore the (5) Section 39,[282] which allows the contractor in a financial and technical assistance agreement to
same.[265] Eventually, the foreign-owned corporation, as such permittee, may apply for a financial and convert the same into a mineral production-sharing agreement;
technical assistance agreement.[266] (6) Section 56,[283] which authorizes the issuance of a mineral processing permit to a contractor in a
Development is financial and technical assistance agreement;
the work undertaken to explore and prepare an ore body or a mineral deposit for mining, including the The following provisions of the same Act are likewise void as they are dependent on the foregoing
construction of necessary infrastructure and related facilities. [267] provisions and cannot stand on their own:
Utilization means the extraction or disposition of minerals.[268] A stipulation that the proponent shall (1) Section 3 (g),[284] which defines the term contractor, insofar as it applies to a financial or technical
dispose of the minerals and byproducts produced at the highest price and more advantageous terms and assistance agreement.
conditions as provided for under the implementing rules and regulations is required to be incorporated in Section 34,[285] which prescribes the maximum contract area in a financial or technical assistance
every FTAA.[269] agreements;
Section 36,[286] which allows negotiations for financial or technical assistance agreements;
Section 37,[287] which prescribes the procedure for filing and evaluation of financial or technical In arguing against the annulment of the FTAA, WMCP invokes the Agreement on the Promotion and
assistance agreement proposals; Protection of Investments between the Philippine and Australian Governments, which was signed in Manila
Section 38,[288] which limits the term of financial or technical assistance agreements; on January 25, 1995 and which entered into force on December 8, 1995.
Section 40,[289] which allows the assignment or transfer of financial or technical assistance agreements; x x x. Article 2 (1) of said treaty states that it applies to investments whenever made and thus the fact that
Section 41,[290] which allows the withdrawal of the contractor in an FTAA; [WMCPs] FTAA was entered into prior to the entry into force of the treaty does not preclude the Philippine
The second and third paragraphs of Section 81,[291] which provide for the Governments share in a Government from protecting [WMCPs] investment in [that] FTAA. Likewise, Article 3 (1) of the treaty
financial and technical assistance agreement; and provides that Each Party shall encourage and promote investments in its area by investors of the
Section 90,[292] which provides for incentives to contractors in FTAAs insofar as it applies to said other Party and shall [admit] such investments in accordance with its Constitution, Laws,
contractors; regulations and investment policies and in Article 3 (2), it states that Each Party shall ensure that
When the parts of the statute are so mutually dependent and connected as conditions, considerations, investments are accorded fair and equitable treatment. The latter stipulation indicates that it was
inducements, or compensations for each other, as to warrant a belief that the legislature intended them as a intended to impose an obligation upon a Party to afford fair and equitable treatment to the investments of the
whole, and that if all could not be carried into effect, the legislature would not pass the residue other Party and that a failure to provide such treatment by or under the laws of the Party may constitute a
independently, then, if some parts are unconstitutional, all the provisions which are thus dependent, breach of the treaty. Simply stated, the Philippines could not, under said treaty, rely upon the inadequacies
conditional, or connected, must fall with them.[293] of its own laws to deprive an Australian investor (like [WMCP]) of fair and equitable treatment by invalidating
There can be little doubt that the WMCP FTAA itself is a service contract. [WMCPs] FTAA without likewise nullifying the service contracts entered into before the enactment of RA
Section 1.3 of the WMCP FTAA grants WMCP the exclusive right to explore, exploit, utilise[,] process 7942 such as those mentioned in PD 87 or EO 279.
and dispose of all Minerals products and by-products thereof that may be produced from the Contract This becomes more significant in the light of the fact that [WMCPs] FTAA was executed not by a mere
Area.[294] The FTAA also imbues WMCP with the following rights: Filipino citizen, but by the Philippine Government itself, through its President no less, which, in entering into
(b) to extract and carry away any Mineral samples from the Contract area for the purpose of conducting tests said treaty is assumed to be aware of the existing Philippine laws on service contracts over the exploration,
and studies in respect thereof; development and utilization of natural resources. The execution of the FTAA by the Philippine Government
(c) to determine the mining and treatment processes to be utilised during the Development/Operating Period assures the Australian Government that the FTAA is in accordance with existing Philippine
and the project facilities to be constructed during the Development and Construction Period; laws.[300] [Emphasis and italics by private respondents.]
(d) have the right of possession of the Contract Area, with full right of ingress and egress and the right to The invalidation of the subject FTAA, it is argued, would constitute a breach of said treaty which, in
occupy the same, subject to the provisions of Presidential Decree No. 512 (if applicable) and not be turn, would amount to a violation of Section 3, Article II of the Constitution adopting the generally accepted
prevented from entry into private ands by surface owners and/or occupants thereof when prospecting, principles of international law as part of the law of the land. One of these generally accepted principles
exploring and exploiting for minerals therein; is pacta sunt servanda, which requires the performance in good faith of treaty obligations.
xxx Even assuming arguendo that WMCP is correct in its interpretation of the treaty and its assertion that
(f) to construct roadways, mining, drainage, power generation and transmission facilities and all other types the Philippines could not . . . deprive an Australian investor (like [WMCP]) of fair and equitable treatment by
of works on the Contract Area; invalidating [WMCPs] FTAA without likewise nullifying the service contracts entered into before the
(g) to erect, install or place any type of improvements, supplies, machinery and other equipment relating to enactment of RA 7942 . . ., the annulment of the FTAA would not constitute a breach of the treaty
the Mining Operations and to use, sell or otherwise dispose of, modify, remove or diminish any and all parts invoked. For this decision herein invalidating the subject FTAA forms part of the legal system of the
thereof; Philippines.[301] The equal protection clause[302]guarantees that such decision shall apply to all contracts
(h) enjoy, subject to pertinent laws, rules and regulations and the rights of third Parties, easement rights and belonging to the same class, hence, upholding rather than violating, the fair and equitable treatment
the use of timber, sand, clay, stone, water and other natural resources in the Contract Area without cost for stipulation in said treaty.
the purposes of the Mining Operations; One other matter requires clarification. Petitioners contend that, consistent with the provisions of
xxx Section 2, Article XII of the Constitution, the President may enter into agreements
(l) have the right to mortgage, charge or encumber all or part of its interest and obligations under this involving either technical or financial assistance only. The agreement in question, however, is a
Agreement, the plant, equipment and infrastructure and the Minerals produced from the Mining Operations; technical and financial assistance agreement.
x x x. [295] Petitioners contention does not lie. To adhere to the literal language of the Constitution would lead to
All materials, equipment, plant and other installations erected or placed on the Contract Area remain absurd consequences.[303] As WMCP correctly put it:
the property of WMCP, which has the right to deal with and remove such items within twelve months from x x x such a theory of petitioners would compel the government (through the President) to enter into contract
the termination of the FTAA.[296] with two (2) foreign-owned corporations, one for financial assistance agreement and with the other, for
Pursuant to Section 1.2 of the FTAA, WMCP shall provide [all] financing, technology, management and technical assistance over one and the same mining area or land; or to execute two (2) contracts with
personnel necessary for the Mining Operations. The mining company binds itself to perform all Mining only one foreign-owned corporation which has the capability to provide both financial and technical
Operations . . . providing all necessary services, technology and financing in connection therewith, [297] and to assistance, one for financial assistance and another for technical assistance, over the same mining
furnish all materials, labour, equipment and other installations that may be required for carrying on all Mining area. Such an absurd result is definitely not sanctioned under the canons of constitutional
Operations.[298] WMCP may make expansions, improvements and replacements of the mining facilities and construction.[304] [Underscoring in the original.]
may add such new facilities as it considers necessary for the mining operations. [299] Surely, the framers of the 1987 Charter did not contemplate such an absurd result from their use of
These contractual stipulations, taken together, grant WMCP beneficial ownership over natural either/or. A constitution is not to be interpreted as demanding the impossible or the impracticable; and
resources that properly belong to the State and are intended for the benefit of its citizens.These stipulations unreasonable or absurd consequences, if possible, should be avoided. [305] Courts are not to give words a
are abhorrent to the 1987 Constitution. They are precisely the vices that the fundamental law seeks to avoid, meaning that would lead to absurd or unreasonable consequences and a literal interpretation is to be
the evils that it aims to suppress. Consequently, the contract from which they spring must be struck down.
rejected if it would be unjust or lead to absurd results. [306] That is a strong argument against its
adoption.[307] Accordingly, petitioners interpretation must be rejected.
The foregoing discussion has rendered unnecessary the resolution of the other issues raised by the
petition.
WHEREFORE, the petition is GRANTED. The Court hereby declares unconstitutional and void:
(1) The following provisions of Republic Act No. 7942:
(a) The proviso in Section 3 (aq),
(b) Section 23,
(c) Section 33 to 41,
(d) Section 56,
(e) The second and third paragraphs of Section 81, and
(f) Section 90.
(2) All provisions of Department of Environment and Natural Resources Administrative Order 96-40, s.
1996 which are not in conformity with this Decision, and
(3) The Financial and Technical Assistance Agreement between the Government of the Republic of the
Philippines and WMC Philippines, Inc.
SO ORDERED.
LA BUGAL B’LAAN TRIBAL ASSOCIATION INC., et. al. v. V. O. RAMOS, Secretary Department of service contracts was precisely the evil the drafters of the 1987 Constitution sought to avoid.
Environment and Natural Resources; H. RAMOS, Director, Mines and Geosciences Bureau (MGB- The constitutional provision allowing the President to enter into FTAAs is an exception to the rule that
DENR); R. TORRES, Executive Secretary; and WMC (PHILIPPINES) INC. participation in the nation‘s natural resources is reserved exclusively to Filipinos. Accordingly, such provision
The constitutional provision allowing the President to enter into FTAA is a exception to the rule that must be construed strictly against their enjoyment by non-Filipinos. Therefore, RA 7942 is invalid insofar as
participation in the nation’s natural resources is reserved exclusively to Filipinos. Provision must be the said act authorizes service contracts. Although the statute employs the phrase ―financial and technical
construed strictly against their enjoyment by non-Filipinos. agreements in accordance with the 1987 Constitution, its pertinent provisions actually treat these
RA 7942 (The Philippine Mining Act) took effect on April 9, 1995. Before the effectivity of RA 7942, or on agreements as service contracts that grant beneficial ownership to foreign contractors contrary to the
March 30, 1995, the President signed a Financial and Technical Assistance Agreement (FTAA) with WMCP, fundamental law.
a corporation organized under Philippine laws, covering close to 100,000 hectares of land in South The underlying assumption in the provisions of the law is that the foreign contractor manages the mineral
Cotabato, Sultan Kudarat, Davao del Sur and North Cotabato. On August 15, 1995, the Environment resources just like the foreign contractor in a service contract. By allowing foreign contractors to manage or
Secretary Victor Ramos issued DENR Administrative Order 95-23, which was later repealed by DENR operate all the aspects of the mining operation, RA 7942 has, in effect, conveyed beneficial ownership over
Administrative Order 96-40, adopted on December 20, 1996. the nation‘s mineral resources to these contractors, leaving the State with nothing but bare title thereto.
Petitioners prayed that RA 7942, its implementing rules, and the FTAA between the government and WMCP The same provisions, whether by design or inadvertence, permit a circumvention of the constitutionally
be declared unconstitutional on ground that they allow fully foreign owned corporations like WMCP to ordained 60-40% capitalizationrequirement for corporations or associations engaged in the exploitation,
exploit, explore and develop Philippine mineral resources in contravention of Article XII Section 2 development and utilization of Philippine natural resources.
paragraphs 2 and 4 of the Charter. When parts of a statute are so mutually dependent and connected as conditions, considerations,
In January 2001, WMC – a publicly listed Australian mining and exploration company – sold its whole stake inducements or compensations for each other as to warrant a belief that the legislature intended them as a
in WMCP to Sagittarius Mines, 60% of which is owned by Filipinos while 40% of which is owned by Indophil whole, then if some parts are unconstitutional, all provisions that are thus dependent, conditional or
Resources, an Australian company. DENR approved the transfer and registration of the FTAA in Sagittarius‘ connected, must fail with them.
name but Lepanto Consolidated assailed the same. The latter case is still pending before the Court of Under Article XII Section 2 of the 1987 Charter, foreign owned corporations are limited only to merely
Appeals. technical or financial assistance to the State for large scale exploration, development and utilization of
EO 279, issued by former President Aquino on July 25, 1987, authorizes the DENR to accept, consider and minerals, petroleum and other mineral oils.
evaluate proposals from foreign owned corporations or foreign investors for contracts or agreements Second Issue: RP Government-WMCP FTAA is a Service Contract
involving wither technical or financial assistance for large scale exploration, development and utilization of The FTAA between he WMCP and the Philippine government is likewise unconstitutional since the
minerals which upon appropriate recommendation of the (DENR) Secretary, the President may execute with agreement itself is a service contract.
the foreign proponent. WMCP likewise contended that the annulment of the FTAA would violate a treaty Section 1.3 of the FTAA grants WMCP a fully foreign owned corporation, the exclusive right to explore,
between the Philippines and Australia which provides for the protection of Australian investments. exploit, utilize and dispose of all minerals and by-products that may be produced from the contract area.
ISSUES: Section 1.2 of the same agreement provides that EMCP shall provide all financing, technology,
1. Whether or not the Philippine Mining Act is unconstitutional for allowing fully foreign-owned corporations management, and personnel necessary for the Mining Operations.
to exploit the Philippine mineral resources. 2. Whether or not the FTAA between the government and WMCP These contractual stipulations and related provisions in the FTAA taken together, grant WMCP beneficial
is a ―service contract that permits fully foreign owned companies to exploit the Philippine mineral ownership over natural resources that properly belong to the State and are intended for the benefit of its
resources. citizens. These stipulations are abhorrent to the 1987 Constitution. They are precisely the vices that the
HELD: fundamental law seeks to avoid, the evils that it aims to suppress. Consequently, the contract from which
First Issue: RA 7942 is Unconstitutional they spring must be struck down.
RA 7942 or the Philippine Mining Act of 1995 is unconstitutional for permitting fully foreign owned
corporations to exploit the Philippine natural resources.
Article XII Section 2 of the 1987 Constitution retained the Regalian Doctrinewhich states that ―All lands of
the public domain, waters, minerals, coal, petroleum, and other minerals, coal, petroleum, and other mineral
oils, all forces of potential energy, fisheries, forests or timber, wildlife, flora and fauna, and other natural
resources are owned by the State. The same section also states that, ―the exploration and development
and utilization of natural resources shall be under the full control and supervision of the State.
Conspicuously absent in Section 2 is the provision in the 1935 and 1973 Constitution authorizing the State to
grant licenses, concessions, or leases for the exploration, exploitation, development, or utilization of natural
resources. By such omission, the utilization of inalienable lands of the public domain through license,
concession or lease is no longer allowed under the 1987 Constitution.
Under the concession system, the concessionaire makes a direct equity investment for the purpose of
exploiting a particular natural resource within a given area. The concession amounts to complete control by
the concessionaireover the country‘s natural resource, for it is given exclusive and plenary rights to exploit a
particular resource at the point of extraction.
The 1987 Constitution, moreover, has deleted the phrase ―management or other forms of assistance in the
1973 Charter. The present Constitution now allows only ―technical and financial assistance. The
management and the operation of the mining activities by foreign contractors, the primary feature of the
[G. R. No. 107764. October 4, 2002] Reynoso, Mariano Leyva the grandson of Sesinando Leyva, the previous owner, attended to
EDNA COLLADO, BERNARDINA TAWAS, JORETO C. TORRES, JOSE AMO, SERGIO L. the farm. (Testimony of Mariano Leyva, supra). Angelina Reynoso declared the property in
MONTEALEGRE, VICENTE C. TORRES, JOSEPH L. NUEZ, GLORIA SERRANO, DANILO her name under Tax Declaration No. 7189 in 4 February 1958, under Tax Declaration No.
FABREGAS, FERNANDO T. TORRES, LUZ G. TUBUNGBANUA, CARIDAD T. TUTANA, JOSE 8775 on 3 August 1965, under Tax Declaration No. 16945 on 15 December 1975, and
C. TORRES, JR., IMELDA CAYLALUAD, ROSALIE TUTANA, NORMA ASTORIAS, MYRNA M. under Tax Declaration No. 03-06145 on 25 June 1978.
LANCION, NORBERTO CAMILOTE, CECILIA MACARANAS, PEDRO BRIONES, REMEDIOS 5. MYRNA TORRES bought the property from Angelina Reynoso on 16 October 1982 through
BANTIGUE, DANTE L. MONTEALEGRE, AIDA T. GADON, ARMANDO T. TORRES and a Deed of Sale (Exhibit G).
FIDELITO ECO, petitioners, vs. COURT OF APPEALS and REPUBLIC OF THE PHILIPPINES, 6. EDNA COLLADO bought the property from Myrna Torres in a Deed of Sale dated 28 April
thru the Director of Lands, respondents, 1984 (Exhibit P-1 to P-3).
BOCKASANJO ISF AWARDEES ASSOCIATION, INC., LITA MENDOZA, MORADO PREFIDIGNO, 7. Additional owners BERNARDINA TAWAS, JORETO TORRES, JOSE AMO, VICENTE
TERESITA CRUZ and CALOMA MOISES, respondents/intervernors. TORRES and SERGIO MONTEALEGRE who bought portions of the property from Edna
DECISION Collado through a Deed of Sale on 6 November 1985 (Exhibit Q to Q-3).
CARPIO, J.: 8. And more additional Owners JOSEPH NUNEZ, DIOSDADO ARENOS, DANILO
The Case FABREGAS, FERNANDO TORRES, LUZ TUBUNGBANUA, CARIDAD TUTANA, JOSE
This Petition[1] seeks to set aside the Decision of the Court of Appeals, [2] dated June 22, 1992, in CA- TORRES JR., RODRIGO TUTANA, ROSALIE TUTANA, NORMA ASTORIAS, MYRNA
G.R. SP No. 25597, which declared null and void the Decision [3] dated January 30, 1991 of the Regional LANCION, CHONA MARCIANO, CECILIA MACARANAS, PEDRO BRIONES, REMEDIOS
Trial Court of Antipolo, Rizal, Branch 71, in LRC No. 269-A, LRC Rec. No. N-59179, confirming the imperfect BANTIQUE, DANTE MONTEALEGRE, ARMANDO TORRES, AIDA GADON and AMELIA M.
title of petitioners over a parcel of land. MALAPAD bought portions of the property in a Deed of Sale on 12 May 1986 (Exhibit S to
The Facts S-3).
On April 25, 1985, petitioner Edna T. Collado filed with the land registration court an application for 9. Co-owners DIOSDADO ARENOS, RODRIGO TUTANA, CHONA MARCIANO and AMELIA
registration of a parcel of land with an approximate area of 1,200,766 square meters or 120.0766 hectares MALAPAD jointly sold their shares to new OWNERS GLORIA R. SERRANO, IMELDA
(Lot for brevity). The Lot is situated in Barangay San Isidro (formerly known as Boso-boso), Antipolo, Rizal, CAYLALUAD, NORBERTO CAMILOTE and FIDELITO ECO through a Deed of Sale dated
and covered by Survey Plan Psu-162620. Attached to the application was the technical description of the Lot 18 January 1987 (Exhibit T to T-9).[6]
as Lot Psu-162620 signed by Robert C. Pangyarihan, Officer-in-Charge of the Survey Division, Bureau of During the hearing on January 9, 1991, only the assistant provincial prosecutor appeared without the
Lands, which stated, [t]his survey is inside IN-12 Mariquina Watershed. On March 24, 1986, petitioner Solicitor General. For failure of the oppositors to present their evidence, the land registration court issued an
Edna T. Collado filed an Amended Application to include additional co-applicants.[4] Subsequently, more order considering the case submitted for decision based on the evidence of the petitioners. The court later
applicants joined (collectively referred to as petitioners for brevity).[5] set aside the order and reset the hearing to January 14, 1991 for the presentation of the evidence of the
The Republic of the Philippines, through the Solicitor General, and the Municipality of Antipolo, through oppositors. On this date, counsel for oppositors failed to appear again despite due notice. Hence, the court
its Municipal Attorney and the Provincial Fiscal of Rizal, filed oppositions to petitioners application. In due again issued an order submitting the case for decision based on the evidence of the petitioners.
course, the land registration court issued an order of general default against the whole world with the The Trial Courts Ruling
exception of the oppositors. After appraisal of the evidence submitted by petitioners, the land registration court held that petitioners
Petitioners alleged that they have occupied the Lot since time immemorial. Their possession has been had adduced sufficient evidence to establish their registrable rights over the Lot.Accordingly, the court
open, public, notorious and in the concept of owners. The Lot was surveyed in the name of Sesinando rendered a decision confirming the imperfect title of petitioners. We quote the pertinent portions of the courts
Leyva, one of their predecessors-in-interest, as early as March 22, 1902. Petitioners declared the Lot for decision, as follows:
taxation purposes and paid all the corresponding real estate taxes. According to them, there are now twenty- From the evidence presented, the Court finds that from the testimony of the witnesses presented by the
five co-owners in pro-indiviso shares of five hectares each. During the hearings, petitioners submitted Applicants, the property applied for is in actual, open, public and notorious possession by the applicants and
evidence to prove that there have been nine transfers of rights among them and their predecessors-in- their predecessor-in-interest since time immemorial and said possession had been testified to by witnesses
interest, as follows: Jimmy Torres, Mariano Leyva, Sergio Montealegre, Jose Amo and one Chona who were all cross-examined
1. SESINANDO LEYVA was the earliest known predecessor-in-interest of the Applicants who by Counsel for Oppositor Republic of the Philippines.
was in actual, open, notorious and continuous possession of the property in the concept of Evidence was likewise presented that said property was declared for taxation purposes in the names of the
owner. He had the property surveyed in his name on 22 March 1902 (Exhibit W and W-1 previous owners and the corresponding taxes were paid by the Applicants and the previous owners and said
testimonies of J. Torres on 16 December 1987 and Mariano Leyva on 29 December 1987). property was planted to fruit bearing trees; portions to palay and portions used for grazing purposes.
2. DIOSDADO LEYVA, is the son of Sesinando Leyva, who inherited the property. He had To the mind of the Court, Applicants have presented sufficient evidence to establish registrable title over
the property resurveyed in his name on May 21-28, 1928 (Exhibit X and X-1; testimony of said property applied for by them.
Mariano Leyva, a son of Diosdado Leyva). On the claim that the property applied for is within the Marikina Watershed, the Court can only add that all
3. GREGORIO CAMANTIQUE bought the property from Diosdado Leyva before the Japanese Presidential Proclamations like the Proclamation setting aside the Marikina Watershed are subject to private
Occupation of the Philippines during World War II. He owned and possessed the rights.
property until 1958. He declared the property for tax purposes, the latest of which In the case of Municipality of Santiago vs. Court of Appeals, 120 SCRA 734, 1983 private rights is proof of
was under Tax Declaration No. 7182 issued on 3 February 1957 (Exhibit I and testimony of acquisition through (sic) among means of acquisition of public lands.
Mariano Leyva, supra). In the case of Director of Lands vs. Reyes, 68 SCRA 193-195, by private rights means that applicant should
4. ANGELINA REYNOSO, bought the property from Gregorio Camantique by virtue of a Deed of show clear and convincing evidence that the property in question was acquired by applicants or their
Sale on 3 February 1958 (Exhibit H). During the ownership of the property by Angelina
ancestors either by composition title from the Spanish government or by Possessory Information title, or any That the land in question is within the Marikina Watershed Reservation is confirmed by the Administrator of
other means for the acquisition of public lands xxx (underscoring supplied). the National Land Titles and Deeds in a Report, dated March 2, 1988, submitted to the respondent Court in
The Court believes that from the evidence presented as above stated, Applicants have acquired private LR Case No. 269-A. These documents readily and effectively negate the allegation in private respondent
rights to which the Presidential Proclamation setting aside the Marikina Watershed should be subject to such Collados application that said parcel of land known as Psu-162620 is not covered by any form of title, nor
private rights. any public land application and are not within any government reservation (Par. 8, Application; Emphasis
At any rate, the Court notes that evidence was presented by the applicants that as per Certification issued supplied). The respondent court could not have missed the import of these vital documents which are
by the Bureau of Forest Development dated March 18, 1980, the area applied for was verified to be within binding upon the courts inasmuch as it is the exclusive prerogative of the Executive Department to classify
the area excluded from the operation of the Marikina Watershed Lands Executive Order No. 33 dated July public lands. They should have forewarned the respondent judge from assuming jurisdiction over the case.
26, 1904 per Proclamation No. 1283 promulgated on June 21, 1974 which established the Boso-boso Town x x x inasmuch as the said properties applied for by petitioners are part of the public domain, it is the
Site Reservation, amended by Proclamation No. 1637 dated April 18, 1977 known as the Lungsod Silangan Director of Lands who has jurisdiction in the disposition of the same (subject to the approval of the Secretary
Townsite Reservation. (Exhibit K).[7] of Natural Resources and Environment), and not the courts. x x x Even assuming that petitioners did have
In a motion dated April 5, 1991, received by the Solicitor General on April 6, 1991, petitioners alleged the said properties surveyed even before the same was declared to be part of the Busol Forest Reservation,
that the decision dated January 30, 1991 confirming their title had become final after the Solicitor General the fact remains that it was so converted into a forest reservation, thus it is with more reason that this action
received a copy of the decision on February 18, 1991. Petitioners prayed that the land registration court must fail. Forest lands are inalienable and possession thereof, no matter how long, cannot convert the same
order the Land Registration Authority to issue the necessary decree in their favor over the Lot. into private property. And courts are without jurisdiction to adjudicate lands within the forest zone. (Heirs of
On April 11, 1991, the Solicitor General inquired from the Provincial Prosecutor of Rizal whether the Gumangan vs. Court of Appeals. 172 SCRA 563; Emphasissupplied).
land registration court had already rendered a decision and if so, whether the Provincial Prosecutor would Needless to say, a final judgment may be annulled on the ground of lack of jurisdiction, fraud or that it is
recommend an appeal. However, the Provincial Prosecutor failed to answer the query. contrary to law (Panlilio vs. Garcia, 119 SCRA 387, 391) and a decision rendered without jurisdiction is a
According to the Solicitor General, he received on April 23, 1991 a copy of the land registration courts total nullity and may be struck down at any time (Suarez vs. Court of Appeals, 186 SCRA 339).[9]
decision dated January 30, 1991, and not on February 18, 1991 as alleged by petitioners in their motion. Hence, the instant petition.
In the meantime, on May 7, 1991, the land registration court issued an order directing the Land The Issues
Regulation Authority to issue the corresponding decree of registration in favor of the petitioners. The issues raised by petitioners are restated as follows:
On August 6, 1991, the Solicitor General filed with the Court of Appeals a Petition for Annulment of I
Judgment pursuant to Section 9(2) of BP Blg. 129 on the ground that there had been no clear showing that WHETHER THE COURT OF APPEALS ERRED OR GRAVELY ABUSED ITS DISCRETION IN
the Lot had been previously classified as alienable and disposable making it subject to private appropriation. REVERSING THE DECISION OF THE TRIAL COURT GRANTING THE APPLICATION OF THE
On November 29, 1991, Bockasanjo ISF Awardees Association, Inc., an association of holders of PETITIONERS FOR CONFIRMATION OF TITLE;
certificates of stewardship issued by the Department of Environment and Natural Resources (DENR for II
brevity) under its Integrated Social Forestry Program (ISF for brevity), filed with the Court of Appeals a WHETHER THE COURT OF APPEALS ERRED OR GRAVELY ABUSED ITS DISCRETION IN GIVING
Motion for Leave to Intervene and to Admit Petition-In-Intervention. They likewise opposed the registration DUE COURSE TO THE PETITION FOR ANNULMENT OF JUDGMENT FILED BY THE REPUBLIC LONG
and asserted that the Lot, which is situated inside the Marikina Watershed Reservation, is inalienable. They AFTER THE DECISION OF THE TRIAL COURT HAD BECOME FINAL;
claimed that they are the actual occupants of the Lot pursuant to the certificates of stewardship issued by III
the DENR under the ISF for tree planting purposes. WHETHER THE COURT OF APPEALS ERRED OR GRAVELY ABUSED ITS DISCRETION IN GIVING
The Court of Appeals granted the motion to intervene verbally during the preliminary conference held DUE COURSE TO THE INTERVENORS PETITION FOR INTERVENTION WHICH WAS FILED OUT OF
on April 6, 1992. During the preliminary conference, all the parties as represented by their respective TIME OR LONG AFTER THE DECISION OF THE TRIAL COURT HAD BECOME FINAL.
counsels agreed that the only issue for resolution was whether the Lot in question is part of the public The Courts Ruling
domain.[8] The petition is bereft of merit.
The Court of Appeals Ruling First Issue: whether petitioners have registrable title over the Lot.
In a decision dated June 22, 1992, the Court of Appeals granted the petition and declared null and void There is no dispute that Executive Order No. 33 (EO 33 for brevity) dated July 26, 1904 [10] established
the decision dated January 30, 1991 of the land registration court. The Court of Appeals explained thus: the Marikina Watershed Reservation (MWR for brevity) situated in the Municipality of Antipolo,
Under the Regalian Doctrine, which is enshrined in the 1935 (Art. XIII, Sec. 1), 1973 (Art. XIV, Sec. 8), and Rizal. Petitioners even concede that the Lot, described as Lot Psu-162620, is inside the technical, literal
1987 Constitution (Art. XII, Sec. 2), all lands of the public domain belong to the State. An applicant, like the description of the MWR. However, the main thrust of petitioners claim over the Lot is that all Presidential
private respondents herein, for registration of a parcel of land bears the burden of overcoming the proclamations like the proclamation setting aside the Marikina Watershed Reservation are subject to private
presumption that the land sought to be registered forms part of the public domain (Director of Lands vs. rights. They point out that EO 33 contains a saving clause that the reservations are subject to existing
Aquino, 192 SCRA 296). private rights, if any there be. Petitioners contend that their claim of ownership goes all the way back to
A positive Act of government is needed to declassify a public land and to convert it into alienable or 1902, when their known predecessor-in-interest, Sesinando Leyva, laid claim and ownership over the Lot.
disposable land for agricultural or other purposes (Republic vs. Bacas, 176 SCRA 376). They claim that the presumption of law then prevailing under the Philippine Bill of 1902 and Public Land
In the case at bar, the private respondents failed to present any evidence whatsoever that the land applied Act No. 926 was that the land possessed and claimed by individuals as their own are agricultural lands and
for as described in Psu-162620 has been segregated from the bulk of the public domain and declared by therefore alienable and disposable. They conclude that private rights were vested on Sesinando Leyva
competent authority to be alienable and disposable. Worse, the technical description of Psu-162620 signed before the issuance of EO 33, thus excluding the Lot from the Marikina Watershed Reservation.
by Robert C. Pangyarihan, Officer-in-Charge, Survey Division, Bureau of Lands, which was attached to the Petitioners arguments find no basis in law.
application of private respondents, categorically stated that "This survey is inside IN-12 Mariquina The Regalian Doctrine: An Overview
Watershed."
Under the Regalian Doctrine, all lands not otherwise appearing to be clearly within private ownership delegates to the Constitutional Convention very well knew that the concept of State ownership of land and
are presumed to belong to the State.[11] The Spaniards first introduced the doctrine to the Philippines through natural resources was introduced by the Spaniards, however, they were not certain whether it was continued
the Laws of the Indies and the Royal Cedulas, specifically, Law 14, Title 12, Book 4 of the Novisima and applied by the Americans. To remove all doubts, the Convention approved the provision in the
Recopilacion de Leyes de las Indias[12] which laid the foundation that all lands that were not acquired from Constitution affirming the Regalian doctrine.
the Government, either by purchase or by grant, belong to the public domain.[13] Upon the Spanish conquest Thus, Section 1, Article XIII[23] of the 1935 Constitution, on Conservation and Utilization of Natural
of the Philippines, ownership of all lands, territories and possessions in the Philippines passed to the Resources barred the alienation of all natural resources except public agricultural lands, which were the only
Spanish Crown.[14] natural resources the State could alienate. The 1973 Constitution reiterated the Regalian doctrine in Section
The Laws of the Indies were followed by the Ley Hipotecaria or the Mortgage Law of 1893. The 8, Article XIV[24] on the National Economy and the Patrimony of the Nation. The 1987 Constitution reaffirmed
Spanish Mortgage Law provided for the systematic registration of titles and deeds as well as possessory the Regalian doctrine in Section 2 of Article XII[25] on National Economy and Patrimony.
claims. The Royal Decree of 1894 or the Maura Law partly amended the Mortgage Law as well as the Law Both the 1935 and 1973 Constitutions prohibited the alienation of all natural resources except
of the Indies. The Maura Law was the last Spanish land law promulgated in the Philippines. It required the agricultural lands of the public domain. The 1987 Constitution readopted this policy.Indeed, all lands of the
adjustment or registration of all agricultural lands, otherwise the lands would revert to the state. [15] public domain as well as all natural resources enumerated in the Philippine Constitution belong to the State.
Four years later, Spain ceded to the government of the United States all rights, interests and claims Watershed Reservation is a Natural Resource
over the national territory of the Philippine Islands through the Treaty of Paris of December 10, 1898. In The term natural resource includes not only timber, gas, oil coal, minerals, lakes, and submerged
1903, the United States colonial government, through the Philippine Commission, passed Act No. 926, the lands, but also, features which supply a human need and contribute to the health, welfare, and benefit of a
first Public Land Act, which was described as follows: community, and are essential to the well-being thereof and proper enjoyment of property devoted to park
Act No. 926, the first Public Land Act, was passed in pursuance of the provisions of the Philippine Bill of and recreational purposes.[26]
1902. The law governed the disposition of lands of the public domain. It prescribed rules and regulations for In Sta. Rosa Realty Development Corp. vs. Court of Appeals, et al.,[27] the Court had occasion to
the homesteading, selling and leasing of portions of the public domain of the Philippine Islands, and discourse on watershed areas. The Court resolved the issue of whether the parcel of land which the
prescribed the terms and conditions to enable persons to perfect their titles to public lands in the Islands. It Department of Environment and Natural Resources had assessed to be a watershed area is exempt from
also provided for the issuance of patents to certain native settlers upon public lands, for the establishment of the coverage of RA No. 6657 or the Comprehensive Agrarian Reform Law (CARL for brevity). [28] The Court
town sites and sale of lots therein, for the completion of imperfect titles, and for the cancellation or defined watershed as an area drained by a river and its tributaries and enclosed by a boundary or divide
confirmation of Spanish concessions and grants in the Islands. In short, the Public Land Act operated on which separates it from adjacent watersheds. However, the Court also recognized that:
the assumption that title to public lands in the Philippine Islands remained in the government; and The definition does not exactly depict the complexities of a watershed. The most important product of a
that the governments title to public land sprung from the Treaty of Paris and other subsequent watershed is water which is one of the most important human necessit(ies). The protection of watershed
treaties between Spain and the United States. The term public land referred to all lands of the public ensures an adequate supply of water for future generations and the control of flashfloods that not only
domain whose title still remained in the government and are thrown open to private appropriation and damage property but also cause loss of lives. Protection of watersheds is an intergenerational responsibility
settlement, and excluded the patrimonial property of the government and the friar lands.[16] that needs to be answered now.
Thus, it is plain error for petitioners to argue that under the Philippine Bill of 1902 and Public Land Act No. Article 67 of the Water Code of the Philippines (PD 1067) provides:
926, mere possession by private individuals of lands creates the legal presumption that the lands are Art. 67. Any watershed or any area of land adjacent to any surface water or overlying any ground water may
alienable and disposable. be declared by the Department of Natural Resources as a protected area. Rules and Regulations may be
Act 2874, the second Public Land Act, superseded Act No. 926 in 1919. After the passage of the 1935 promulgated by such Department to prohibit or control such activities by the owners or occupants thereof
Constitution, Commonwealth Act No. 141 (CA 141 for brevity) amended Act 2874 in 1936. CA 141, as within the protected area which may damage or cause the deterioration of the surface water or ground water
amended, remains to this day as the existing general law governing the classification and disposition of or interfere with the investigation, use, control, protection, management or administration of such waters.
lands of the public domain other than timber and mineral lands. [17] The Court in Sta. Rosa Realty also recognized the need to protect watershed areas and took note of
In the meantime, in order to establish a system of registration by which recorded title becomes the report of the Ecosystems Research and Development Bureau (ERDB), a research arm of the DENR,
absolute, indefeasible and imprescriptible, the legislature passed Act 496, otherwise known as the Land regarding the environmental assessment of the Casile and Kabanga-an river watersheds involved in that
Registration Act, which took effect on February 1, 1903. Act 496 placed all registered lands in the Philippines case. That report concluded as follows:
under the Torrens system.[18] The Torrens system requires the government to issue a certificate of title The Casile barangay covered by CLOA in question is situated in the heartland of both
stating that the person named in the title is the owner of the property described therein, subject to liens and watersheds. Considering the barangays proximity to the Matangtubig waterworks, the activities of the
encumbrances annotated on the title or reserved by law. The certificate of title is indefeasible and farmers which are in conflict with proper soil and water conservation practices jeopardize and endanger the
imprescriptible and all claims to the parcel of land are quieted upon issuance of the certificate. [19] PD 1529, vital waterworks. Degradation of the land would have double edge detrimental effects. On the Casile side
known as the Property Registration Decree enacted on June 11, 1978,[20] amended and updated Act 496. this would mean direct siltation of the Mangumit river which drains to the water impounding reservoir
The 1935, 1973, 1987 Philippine Constitutions below. On the Kabanga-an side, this would mean destruction of forest covers which acts as recharged areas
The 1935, 1973 and 1987 Constitutions adopted the Regalian doctrine substituting, however, the state, of the Matangtubig springs. Considering that the people have little if no direct interest in the protection of the
in lieu of the King, as the owner of all lands and waters of the public domain. [21]Justice Reynato S. Puno, in Matangtubig structures they couldnt care less even if it would be destroyed.
his separate opinion in Cruz vs. Secretary of Environment and Natural Resources,[22] explained thus: The Casile and Kabanga-an watersheds can be considered a most vital life support system to thousands of
One of the fixed and dominating objectives of the 1935 Constitutional Convention was the nationalization inhabitants directly and indirectly affected by it. From these watersheds come the natural God-given
and conservation of the natural resources of the country. There was an overwhelming sentiment in the precious resource water. x x x
Convention in favor of the principle of state ownership of natural resources and the adoption of the Clearing and tilling of the lands are totally inconsistent with sound watershed management. More so, the
Regalian doctrine. State ownership of natural resources was seen as a necessary starting point to secure introduction of earth disturbing activities like road building and erection of permanent infrastructures.Unless
recognition of the states power to control their disposition, exploitation, development, or utilization. The the pernicious agricultural activities of the Casile farmers are immediately stopped, it would not be long
before these watersheds would cease to be of value. The impact of watershed degradation threatens the petitioners have not possessed the parcel of land in the manner and for the number of years required by law
livelihood of thousands of people dependent upon it. Toward this, we hope that an acceptable for the confirmation of imperfect title.
comprehensive watershed development policy and program be immediately formulated and implemented Second, assuming that the Lot was alienable and disposable land prior to the issuance of EO 33 in
before the irreversible damage finally happens. 1904, EO 33 reserved the Lot as a watershed. Since then, the Lot became non-disposable and inalienable
The Court remanded the case to the Department of Agriculture and Adjudication Board or DARAB to re- public land. At the time petitioners filed their application on April 25, 1985, the Lot has been reserved as a
evaluate and determine the nature of the parcels of land involved in order to resolve the issue of its watershed under EO 33 for 81 years prior to the filing of petitioners application.
coverage by the CARL. The period of occupancy after the issuance of EO 33 in 1904 could no longer be counted because as a
Sta. Rosa Realty gives us a glimpse of the dangers posed by the misuse of natural resources such as watershed reservation, the Lot was no longer susceptible of occupancy, disposition, conveyance or
watershed reservations which are akin to forest zones. Population growth and industrialization have taken a alienation. Section 48 (b) of CA 141, as amended, applies exclusively to alienable and disposable public
heavy toll on the environment. Environmental degradation from unchecked human activities could wreak agricultural land. Forest lands, including watershed reservations, are excluded. It is axiomatic that the
havoc on the lives of present and future generations.Hence, by constitutional fiat, natural resources remain possession of forest lands or other inalienable public lands cannot ripen into private
to this day inalienable properties of the State. ownership. In Municipality of Santiago, Isabela vs. Court of Appeals,[32] the Court declared that
Viewed under this legal and factual backdrop, did petitioners acquire, as they vigorously argue, private inalienable public lands -
rights over the parcel of land prior to the issuance of EO 33 segregating the same as a watershed x x x cannot be acquired by acquisitive prescription. Prescription, both acquisitive and extinctive, does not
reservation? run against the State.
The answer is in the negative. The possession of public land, however long the period may have extended, never confers title thereto upon
First. An applicant for confirmation of imperfect title bears the burden of proving that he meets the the possessor because the statute of limitations with regard to public land does not operate against the
requirements of Section 48 of CA 141, as amended. He must overcome the presumption that the land he is State, unless the occupant can prove possession and occupation of the same under claim of ownership
applying for is part of the public domain and that he has an interest therein sufficient to warrant registration for the required number of years to constitute a grant from the State.
in his name arising from an imperfect title. An imperfect title may have been derived from old Spanish grants Third, Gordula vs. Court of Appeals[33] is in point. In Gordula, petitioners did not contest the nature
such as a titulo real or royal grant, a concession especial or special grant, a composicion con el estado or of the land. They admitted that the land lies in the heart of the Caliraya-Lumot River Forest Reserve, which
adjustment title, or a titulo de compra or title through purchase.[29] Or, that he has had continuous, open and Proclamation No. 573 classified as inalienable. The petitioners in Gordula contended, however, that
notorious possession and occupation of agricultural lands of the public domain under a bona fide claim of Proclamation No. 573 itself recognizes private rights of landowners prior to the reservation. They claim to
ownership for at least thirty years preceding the filing of his application as provided by Section 48 (b) CA have established their private rights to the subject land. The Court ruled:
141. We do not agree. No public land can be acquired by private persons without any grant, express or implied
Originally, Section 48(b) of CA 141 provided for possession and occupation of lands of the public from the government; it is indispensable that there be a showing of a title from the state. The facts show that
domain since July 26, 1894. This was superseded by RA 1942 which provided for a simple thirty-year petitioner Gordula did not acquire title to the subject land prior to its reservation under Proclamation No.
prescriptive period of occupation by an applicant for judicial confirmation of an imperfect title. The same, 573. He filed his application for free patent only in January, 1973, more than three (3) years after the
however, has already been amended by Presidential Decree No. 1073, approved on January 25, 1977, the issuance of Proclamation No. 573 in June, 1969. At that time, the land, as part of the Caliraya-Lumot River
law prevailing at the time petitioners application for registration was filed on April 25, 1985. [30] As amended, Forest Reserve, was no longer open to private ownership as it has been classified as public forest reserve
Section 48 (b) now reads: for the public good.
(b) Those who by themselves or through their predecessors-in-interest have been in open, continuous, Nonetheless, petitioners insist that the term, private rights, in Proclamation No. 573, should not be
exclusive and notorious possession and occupation of agricultural lands of the public domain, under a bona interpreted as requiring a title. They opine that it suffices if the claimant had occupied and cultivated the
fide claim of acquisition or ownership, for at least thirty years immediately preceding the filing of the property for so many number of years, declared the land for taxation purposes, [paid] the corresponding real
application for confirmation of title, except when prevented by wars or force majeure. Those shall be estate taxes [which are] accepted by the government, and [his] occupancy and possession [is] continuous,
conclusively presumed to have performed all the conditions essential to a Government grant and shall be open and unmolested and recognized by the government. Prescinding from this premise, petitioners urge
entitled to a certificate of title under the provisions of this chapter. that the 25-year possession by petitioner Gordula from 1944 to 1969, albeit five (5) years short of the 30-
Interpreting Section 48 (b) of CA 141, the Court stated that the Public Land Act requires that the year possession required under Commonwealth Act (C.A.) No. 141, as amended, is enough to vest upon
applicant must prove the following: petitioner Gordula the private rights recognized and respected in Proclamation No. 573.
(a) that the land is alienable public land and (b) that his open, continuous, exclusive and notorious The case law does not support this submission. In Director of Lands vs. Reyes, we held that a settler
possession and occupation of the same must either be since time immemorial or for the period prescribed in claiming the protection of private rights to exclude his land from a military or forest reservation must show x x
the Public Land Act. When the conditions set by law are complied with, the possessor of the land, by x by clear and convincing evidence that the property in question was acquired by [any] x x x means for the
operation of law, acquires a right to a grant, a government grant, without the necessity of a certificate of title acquisition of public lands.
being issued.[31] In fine, one claiming private rights must prove that he has complied with C.A. No. 141, as amended,
Petitioners do not claim to have documentary title over the Lot. Their right to register the Lot is predicated otherwise known as the Public Land Act, which prescribes the substantive as well as the procedural
mainly upon continuous possession since 1902. requirements for acquisition of public lands. This law requires at least thirty (30) years of open, continuous,
Clearly, petitioners were unable to acquire a valid and enforceable right or title because of the failure to exclusive and notorious possession and possession of agricultural lands of the public domain, under a bona
complete the required period of possession, whether under the original Section 48 (b) of CA 141 prior to the fide claim of acquisition, immediately preceding the filing of the application for free patent. The rationale for
issuance of EO 33, or under the amendment by RA 1942 and PD 1073. the 30-year period lies in the presumption that the land applied for pertains to the State, and that the
There is no proof that prior to the issuance of EO 33 in 1904, petitioners had acquired ownership or occupants and/or possessors claim an interest therein only by virtue of their imperfect title or continuous,
title to the Lot either by deed or by any other mode of acquisition from the State, as for instance by open and notorious possession.
acquisitive prescription. As of 1904, Sesinando Leyva had only been in possession for two years. Verily,
Next, petitioners argue that assuming no private rights had attached to the Lot prior to EO 33 in 1904, 33 23 e 1575.00 m to point of beginning. Containing an area of one thousand two hundred twenty five
the President of the Philippines had subsequently segregated the Lot from the public domain and made the (1,225) Hectares, more or less.
Lot alienable and disposable when he issued Proclamation No. 1283 on June 21, 1974. Petitioners contend Note: All data are approximate and subject to change based on future survey.
that Proclamation No. 1283 expressly excluded an area of 3,780 hectares from the MWR and made the area IN WITNESS WHEREOF, I Have hereunto set my hand and caused the seal of the Republic of the
part of the Boso-boso Townsite Reservation. Petitioners assert that Lot Psu-162620 is a small part of this Philippines to be affixed.
excluded town site area. Petitioners further contend that town sites are considered alienable and disposable Done in the City of Manila, this 21st day of June, in the year of Our Lord, nineteen hundred and seventy-four.
under CA 141. (Sgd.) FERDINAND E. MARCOS
Proclamation No. 1283 reads thus: President
PROCLAMATION NO. 1283 Republic of the Philippines
EXCLUDING FROM THE OPERATION EXECUTIVE ORDER NO. 33, DATED JULY 26, 1904, AS Proclamation No. 1283 has since been amended by Proclamation No. 1637 issued on April 18,
AMENDED BY EXECUTIVE ORDERS NOS. 14 AND 16, BOTH SERIES OF 1915, WHICH ESTABLISHED 1977. Proclamation No. 1637 revised the area and location of the proposed townsite.According to then
THE WATERSHED RESERVATION SITUATED IN THE MUNICIPALITY OF ANTIPOLO, PROVINCE OF DENR Secretary Victor O. Ramos, Proclamation No. 1637 excluded Lot A (of which the Lot claimed by
RIZAL, ISLAND OF LUZON, A CERTAIN PORTION OF THE LAND EMBRACED THEREIN AND petitioners is part) for townsite purposes and reverted it to MWR coverage. [34] Proclamation No. 1637 reads:
RESERVING THE SAME, TOGETHER WITH THE ADJACENT PARCEL OF LAND OF THE PUBLIC PROCLAMATION NO. 1637
DOMAIN, FOR TOWNSITE PURPOSES UNDER THE PROVISIONS OF CHAPTER XI OF THE PUBLIC AMENDING PROCLAMATION NO. 1283, DATED JUNE 21, 1974, WHICH ESTABLISHED THE
LAND ACT. TOWNSITE RESERVATION IN THE MUNICIPALITIES OF ANTIPOLO AND SAN MATEO, PROVINCE OF
Upon recommendation of the Secretary of Agriculture and Natural Resources and pursuant to the authority RIZAL, ISLAND OF LUZON BY INCREASING THE AREA AND REVISING THE TECHNICAL
vested in me by law, I, FERDINAND E. MARCOS, President of the Philippines, do hereby, exclude from the DESCRIPTION OF THE LAND EMBRACED THEREIN, AND REVOKING PROCLAMATION NO. 765
operation of Executive Order No. 33 dated July 26, 1904, as amended by Executive Orders Nos. 14 and 16, DATED OCTOBER 26, 1970 THAT RESERVED PORTIONS OF THE AREA AS RESETTLEMENT SITE.
both series of 1915, which established the Watershed Reservation situated in the Municipality of Antipolo, Upon recommendation of the Secretary of Natural Resources and pursuant to the authority vested in me by
Province of Rizal, Island of Luzon, certain portions of land embraced therein and reserve the same, together law, I, FERDINAND E. MARCOS, President of the Philippines, do hereby amend Proclamation No. 1283,
with the adjacent parcel of land of the public domain, for townsite purposes under the provisions of Chapter dated June 21, 1974 which established the townsite reservation in the municipalities of Antipolo and San
XI of the Public Land Act, subject to private rights, if any there be, and to future subdivision survey in Mateo, Province of Rizal, Island of Luzon, by increasing the area and revising the technical descriptions of
accordance with the development plan to be prepared and approved by the Department of Local the land embraced therein, subject to private rights, if any there be, which parcel of land is more particularly
Government and Community Development, which parcels are more particularly described as follows: described as follows:
Lot A (Part of Watershed Reservation) (Proposed Lungsod Silangan Townsite)
A parcel of land (Lot A of Proposed Poor Mans Baguio, being a portion of the Marikina Watershed, IN-2), A PARCEL OF LAND (Proposed Lungsod Silangan Townsite Reservation amending the area under SWO-
situated in the municipality of Antipolo, Province of Rizal, Island of Luzon, beginning at a point marked 1 on 41762 establishing the Bagong Silangan Townsite Reservation) situated in the Municipalities of Antipolo,
sketch plan, being N-74-30 E, 8480.00 meters more or less, from BLLM 1, Antipolo, Rizal; thence N 33 28 W San Mateo, and Montalban, Province of Rizal, Island of Luzon. Bounded on the E., along lines 1-2-3-4-5-6-
1575.00 m. to point 2; thence N 40 26 W 1538.50 m. to point 3; thence N 30 50W 503.17 m. to point 4; 7-8-9-10-11-12-13-14-15-16-17-18-19-20-21-22-23 by the Marikina Watershed Reservation (IN-12); on the
thence N 75 02 W 704.33 m. to point 5; thence N 14 18 W 1399.39 m. to point 6; thence N 43 25 W 477.04 S., along lines 23-24-25 by the portion of Antipolo; on the W., along lines 25-26-27-28-29-30 by the
m. to point 7; thence N 71 38 W 458.36 m. to point 8; thence N 31 05 W 1025.00 m. to point 9; thence Due Municipalities of Montalban, San Mateo; and on the N., along lines 30-31-32-33-34-35-36-37-38-39-40-41-
North 490.38 m. to point 10; thence Due North 1075.00 m. to point 11; thence Due East 1000.00 m. to point 42-43-44 by the Angat Watershed Reservation. Beginning at a point marked 1 on the Topographic Maps
12; thence Due East 1000.00 m. to point 13; thence Due East 1000.00 m. to point 14; thence Due East with the Scale of 1:50,000 which is the identical corner 38 IN-12, Marikina Watershed Reservation.
1000.00 m. to point 15; thence Due East 1000.00 m. to point 16; thence Due East 1000.00 m. to point 17; xxx xxx xxx
thence Due East 1075.00 m. to point 18; thence Due South 1000.00 m. to point 19; thence Due South NOTE: All data are approximate and subject to change based on future survey.
1000.00 m. to point 20; thence Due South 1000.00 m. to point 21; thence Due South 1000.00 m. to point 22; Proclamation No. 765 dated October 26, 1970, which covered areas entirely within the herein Lungsod
thence Due South 1000.00 m. to point 23; thence Due South 1000.00 m. to point 24; thence Due South Silangan Townsite, is hereby revoked accordingly.
1075.00 m. to point 25; thence Due West 1000.00 m. to point 26; thence Due West 1000.00 m. to point 27; IN WITNESS WHEREOF, I have hereunto set my hand and caused the seal of the Republic of the
thence Due West 636.56 m. to point of beginning.Containing an area of three thousand seven hundred Philippines to be affixed.
eighty (3,780) Hectares, more or less. Done in the City of Manila, this 18th day of April, in the year of Our Lord, nineteen hundred and seventy-
Lot B (Alienable and Disposable Land) seven.
A parcel of land (Lot B of Proposed Poor Mans Baguio, being a portion of alienable and disposable portion (Sgd.) FERDINAND E. MARCOS
of public domain) situated in the municipality of Antipolo, Province of Rizal, Island of Luzon.Beginning at a President of the Philippines
point marked 1 on sketch plan being N 74 30 E., 8430.00 m., more or less, from BLLM 1. Antipolo, Rizal; A positive act (e.g., an official proclamation) of the Executive Department is needed to declassify land
thence Due West 363.44 m. to point 2; thence Due West 1000.00 m. to point 3; thence Due West 100.00 m. which had been earlier classified as a watershed reservation and to convert it into alienable or disposable
to point 4; thence Due West 1000.00 m. to point 5; thence Due West 1075.00 m. to point 6; thence Due land for agricultural or other purposes.[35] Unless and until the land classified as such is released in an official
North 1000.00 m. to point 7; thence Due North 1000.00 m. to point 8; thence Due North 1000.00 m. to point proclamation so that it may form part of the disposable agricultural lands of the public domain, the rules on
9; thence Due North 1000.00 m. to point 10; thence Due North 1000.00 m. to point 11; thence Due North confirmation of imperfect title do not apply.[36]
509.62 m. to point 12; thence S. 31 05 E 1025.00 m. to point 13; thence S 71 38 E 458.36 m. to point 14; The principal document presented by petitioners to prove the private character of the Lot is the
thence S 43 25 E 477.04 m. to point 15; thence S 14 18 E 1399.39 m. to point 16; thence S 75 02 E 704.33 Certification of the Bureau of Forest Development dated March 18, 1986 that the Lot is excluded from the
m. to point 17; thence S. 30 50 E 503.17 m. to point 18; thence S 40 26 E 1538.50 m. to point 19; thence s Marikina Watershed (Exh. R). The Certification reads:
Republic of the Philippines Likewise, in a letter[38] dated November 11, 1991, the Deputy Land Inspector, DENR, Region IV,
Ministry of Natural Resources Community Environment and Natural Resources Office, Antipolo, Rizal, similarly confirmed that the Lot is
BUREAU OF FOREST DEVELOPMENT within the MWR. The letter states:
REGION IV That the land sought to be registered is situated at San Isidro (Boso-boso), Antipolo, Rizal, with an area of
EL AL Building ONE HUNDRED TWENTY SIX POINT ZERO SEVEN SIXTY SIX (126.0766) hectares, more particularly
100 Quezon Avenue, Quezon City described in Psu-162620, which is within the Marikina Watershed Reservation under Executive Order No. 33
MAR 18 1986 dated July 2, 1904 which established the Marikina Watershed Reservation (IN-12) x x x.
VERIFICATION ON THE STATUS OF LAND: xxx
TO WHOM IT MAY CONCERN: That the land sought to be registered is not a private property of the Registration Applicant but part of the
This is to certify that the tract of land situated in Barangay San Isidro, Antipolo, Rizal, containing an area of public domain, not subjected to disposition and is covered by Proclamation No. 585 for Integrated Social
1,269,766 square meters, as shown and described on the reverse side hereof, surveyed by Geodetic Forestry Program hence, L.R.C. No. 269-A is recommended for rejection (Underlining supplied). Copy of the
Engineer Telesforo Cabading for Angelina C. Reynoso, is verified to be within the area excluded from the letter is attached herewith as Annex 3 and made an integral part hereof.
operation of Marikina Watershed Reservation established under Executive Order No. 33 dated July 26, 1904 Lastly, the Solicitor General pointed out that attached to petitioner Edna T. Collados [as original
per Proclamation No. 1283, promulgated on June 21, 1974, which established the Boso-Boso Townsite applicant] application is the technical description [39] of the Lot signed by Robert C. Pangyarihan, Officer-in-
Reservation, amended by proclamation No. 1637 dated April 18, 1977 known as Lungsod Silangan Townsite Charge of the Survey Division of the Bureau of Lands. This technical description categorically stated that the
Reservation. Lot is inside IN-12 Mariquina Watershed.
Subject area also falls within the bounds of Bagong Lipunan Site under P.D. 1396 dated June 2, 1978 under The evidence of record thus appears unsatisfactory and insufficient to show clearly and positively that
the sole jurisdiction of the Ministry of Human Settlements, to the exclusion of any other government the Lot had been officially released from the Marikina Watershed Reservation to form part of the alienable
agencies. and disposable lands of the public domain. We hold that once a parcel of land is included within a watershed
This verification is made upon the request of the Chief, Legal Staff, R-4 as contained in his internal reservation duly established by Executive Proclamation, as in the instant case, a presumption arises that the
memorandum dated March 18, 1986. land continues to be part of such Reservation until clear and convincing evidence of subsequent
Verified by: declassification is shown.
(Sgd) ROMEO C. PASCUBILLO It is obvious, based on the facts on record that neither petitioners nor their predecessors-in-interest
Cartographer II have been in open, continuous, exclusive and notorious possession and occupation of the Lot for at least
Checked by: thirty years immediately preceding the filing of the application for confirmation of title. Even if they submitted
(Sgd) ARMENDO R. CRUZ sufficient proof that the Lot had been excluded from the MWR upon the issuance of Proclamation No. 1283
Supervising Cartographer on June 21, 1974, petitioners possession as of the filing of their application on April 25, 1985 would have
ATTESTED: been only eleven years counted from the issuance of the proclamation in 1974. The result will not change
(Sgd) LUIS G. DACANAY even if we tack in the two years Sesinando Leyva allegedly possessed the Lot from 1902 until the issuance
Chief, Forest Engineering & of EO 33 in 1904. Petitioners case falters even more because of the issuance of Proclamation No. 1637 on
Infrastructure Section April 18, 1977. According to then DENR Secretary Victor Ramos, Proclamation No. 1637 reverted Lot A or
The above certification on which petitioners rely that a reclassification had occurred, and that the Lot is the townsite reservation, where petitioners' Lot is supposedly situated, back to the MWR.
covered by the reclassification, is contradicted by several documents submitted by the Solicitor General Finally, it is of no moment if the areas of the MWR are now fairly populated and vibrant communities as
before the land registration court. claimed by petitioners. The following ruling may be applied to this case by analogy:
The Solicitor General submitted to the land registration court a Report [37] dated March 2, 1988, signed A forested area classified as forest land of the public domain does not lose such classification simply
by Administrator Teodoro G. Bonifacio of the then National Land Titles and Deeds Registration because loggers or settlers may have stripped it of its forest cover. Parcels of land classified as forest land
Administration, confirming that the Lot described in Psu-162620 forms part of the MWR. He thus may actually be covered with grass or planted to crops by kaingin cultivators or other farmers. Forest lands
recommended the dismissal of the application for registration. The Report states: do not have to be on mountains or in out of the way places. Swampy areas covered by mangrove trees, nipa
COMES NOW the Administrator of the National Land Titles and Deeds Registration Commission and to this palms and other trees growing in brackish or sea water may also be classified as forest land. The
Honorable Court respectfully reports that: classification is descriptive of its legal nature or status and does not have to be descriptive of what the land
1. A parcel of land described in plan Psu-162620 situated in the Barrio of San Isidro, Municipality actually looks like. Unless and until the land classified as forest is released in an official proclamation to that
of Antipolo, Province of Rizal, is applied for registration of title in the case at bar. effect so that it may form part of the disposable agricultural lands of the public domain, the rules on
2. After plotting plan Psu-162620 in our Municipal Index Map it was found that a portion of the confirmation of imperfect title do not apply.[40]
SW, described as Lot 3 in plan Psu-173790 was previously the subject of registration in Land Second Issue: Whether the petition for annulment of judgment
Reg. Case No. N-9578, LRC Record No. N-55948 and was issued Decree No. N-191242 on should have been given due course.
April 4, 1986 in the name of Apolonia Garcia, et al., pursuant to the Decision and Order for Petitioners fault the Court of Appeals for giving due course to the Republics petition for annulment of
Issuance of the Decree dated February 8, 1984 and March 6, 1984, respectively, and the judgment which was filed long after the decision of the land registration court had allegedly become final and
remaining portion of plan Psu-162620 is inside IN-12, Marikina Watershed. x x x executory. The land registration court rendered its decision on January 30, 1991 and the Solicitor General
WHEREFORE, this matter is respectfully submitted to the Honorable Court for its received a copy of the decision on April 23, 1991. [41]Petitioners point out that the Solicitor General filed with
information and guidance with the recommendation that the application in the instant the Court of Appeals the petition for annulment of judgment invoking Section 9(2) of BP Blg. 129 [42] only on
proceedings be dismissed, after due hearing (Underlining supplied). August 6, 1991, after the decision had supposedly become final and executory. Moreover, petitioners further
point out that the Solicitor General filed the petition for annulment after the land registration court issued its
order of May 6, 1991 directing the Land Registration Authority to issue the corresponding decree of BARANGAYS SAN JOSEPH AND PAENAAN, MUNICIPALITY OF ANTIPOLO, PROVINCE OF RIZAL,
registration. ISLAND OF LUZON.
The Solicitor General sought the annulment of the decision on the ground that the land registration Upon recommendation of the Secretary of Environment and Natural Resources and pursuant to the authority
court had no jurisdiction over the case, specifically, over the Lot which was not alienable and vested in me by law, I, CORAZON C. AQUINO, President of the Philippines, do hereby exclude from the
disposable. The Solicitor General maintained that the decision was null and void. operation of Executive Order No. 33, which established the Marikina Watershed Reservation, certain parcel
Petitioners argue that the remedy of annulment of judgment is no longer available because it is barred of land of the public domain embraced therein situated in Sitios Bosoboso, Veterans, Kilingan and Barangay
by the principle of res judicata. They insist that the land registration court had jurisdiction over the case San Joseph and Paenaan, Municipality of Antipolo, Province of Rizal and place the same under the
which involves private land. They also argue that the Republic is estopped from questioning the land Integrated Social Forestry Program of the Department of Environment and Natural Resources in accordance
registration courts jurisdiction considering that the Republic participated in the proceedings before the court. with existing laws, rules and regulations, which parcel of land is more particularly described as follows:
It is now established that the Lot, being a watershed reservation, is not alienable and disposable public A PARCEL OF LAND, within the Marikina Watershed Reservation situated in the Municipality of Antipolo,
land. The evidence of the petitioners do not clearly and convincingly show that the Lot, described as Lot Province of Rizal, beginning at point 1 on plan, being identical to corner 1 of Marikina Watershed
Psu-162620, ceased to be a portion of the area classified as a watershed reservation of the public Reservation; thence
domain. Any title to the Lot is void ab initio. In view of this, the alleged procedural infirmities attending the xxx xxx xxx
filing of the petition for annulment of judgment are immaterial since the land registration court never acquired Containing an area of One Thousand Four Hundred Thirty (1,430) Hectares.
jurisdiction over the Lot. All proceedings of the land registration court involving the Lot are therefore null and All other lands covered and embraced under Executive Order No. 33 as amended, not otherwise affected by
void. this Proclamation, shall remain in force and effect.
We apply our ruling in Martinez vs. Court of Appeals,[43] as follows: IN WITNESS WHEREOF, I have hereunto set my hand and caused the seal of the Republic of the
The Land Registration Court has no jurisdiction over non-registrable properties, such as public navigable Philippines to be affixed.
rivers which are parts of the public domain, and cannot validly adjudge the registration of title in favor of Done in the City of Manila, this 5th day of June, in the year of Our Lord, nineteen hundred and ninety.
private applicant. Hence, the judgment of the Court of First Instance of Pampanga as regards the Lot No. 2 (Sgd.) CORAZON C. AQUINO
of certificate of Title No. 15856 in the name of petitioners may be attacked at any time, either directly or President of the Philippines
collaterally, by the State which is not bound by any prescriptive period provided for by the Statute of Pursuant to Proclamation No. 585, the chief of the ISF Unit, acting through the Regional Executive
Limitations. Director of the DENR (Region IV), issued sometime between the years 1989 to 1991 certificates of
We also hold that environmental consequences in this case override concerns over technicalities and rules stewardship contracts to bona fide residents of the barangays mentioned in the proclamation as qualified
of procedure. recipients of the ISF programs. Among those awarded were intervenors. The certificates of stewardship are
In Republic vs. De los Angeles,[44] which involved the registration of public lands, specifically parts of actually contracts of lease granted by the DENR to actual occupants of parcels of land under its ISF
the sea, the Court rejected the principle of res judicata and estoppel to silence the Republics claim over programs for a period of twenty-five (25) years, renewable for another twenty-five (25) years.[45] The DENR
public lands. The Court said: awarded contracts of stewardship to ISF participants in Barangay San Isidro (or Boso-boso) and the other
It should be noted further that the doctrine of estoppel or laches does not apply when the Government barangays based on the Inventory of Forest Occupants the DENR had conducted. [46]
sues as a sovereign or asserts governmental rights, nor does estoppel or laches validate an act that According to intervenors, they learned only on July 31, 1991 about the pendency of LRC Case No.
contravenes law or public policy, and that res judicata is to be disregarded if its application would involve the 269-A before the Regional Trial Court of Antipolo, Rizal. On August 8, 1991, they filed a Motion for Leave to
sacrifice of justice to technicality. Intervene and to Admit Opposition in Intervention before the land registration court to assert their rights and
The Court further held that the right of reversion or reconveyance to the State of the public properties to protect their interests.
registered and which are not capable of private appropriation or private acquisition does not prescribe. However, shortly after the filing of their opposition, intervenors learned that the land registration court
Third issue: Whether the petition-in-intervention is proper. had already rendered a decision on January 30, 1991 confirming petitioners imperfect title. Intervenors
The Bockasanjo ISF Awardees Association, Inc., an association of holders of certificates of counsel received a copy of the decision on August 9, 1991.
stewardship issued by the DENR under its Integrated Social Forestry Program, filed with the Court of On August 14, 1991, intervenors filed a motion to vacate judgment and for new trial before the land
Appeals on November 29, 1991 a Motion for Leave to Intervene and to Admit Petition-In-Intervention. registration court. According to intervenors, the land registration court could not act on its motions due to the
According to intervenors, they are the actual occupants of the Lot which petitioners sought to restraining order issued by the Court of Appeals on August 8, 1991, enjoining the land registration court from
register. Aware that the parcels of land which their forefathers had occupied, developed and tilled belong to executing its decision, as prayed for by the Solicitor General in its petition for annulment of judgment. The
the Government, they filed a petition with then President Corazon C. Aquino and then DENR Secretary intervenors were thus constrained to file a petition for intervention before the Court of Appeals which allowed
Fulgencio S. Factoran, to award the parcels of land to them. the same.
Secretary Factoran directed the Director of Forest Management Bureau to take steps for the Rule 19 of the 1997 Rules of Civil Procedure[47] provides in pertinent parts:
segregation of the aforementioned area from the MWR for development under the DENRs ISF Section 1. Who may intervene. A person who has a legal interest in the matter in litigation, or in the success
Programs. Subsequently, then President Aquino issued Proclamation No. 585 dated June 5, 1990 of either of the parties, or an interest against both, or is so situated as to be adversely affected by a
excluding 1,430 hectares from the operation of EO 33 and placed the same under the DENRs Integrated distribution or other disposition of property in the custody of the court, or an officer thereof may, with leave of
Social Forestry Program. Proclamation No. 585 reads: court, be allowed to intervene in the action. The Court shall consider whether or not the intervention will
PROCLAMATION NO. 585 unduly delay or prejudice the adjudication of the rights of the original parties, and whether or not the
AMENDING FURTHER EXECUTIVE ORDER NO. 33, DATED JULY 26, 1904 WHICH ESTABLISHED THE inertvenors rights may be fully protected in a separate proceeding.
MARIKINA WATERSHED RESERVATION (IN-12) AS AMENDED, BY EXCLUDING CERTAIN PORTIONS Sec. 2. Time to intervene. The motion to intervene may be filed at any time before rendition of judgment by
OF LANDS EMBRACED THEREIN SITUATED AT SITIOS BOSOBOSO, KILINGAN, VETERANS, the trial court. A copy of the pleading-in-intervention shall be attached to the motion and served on the
original parties.
As a rule, intervention is allowed before rendition of judgment by the trial court, as Section 2, Rule 19
expressly provides. However, the Court has recognized exceptions to this rule in the interest of substantial
justice. Mago vs. Court of Appeals[48] reiterated the ruling in Director of Lands vs. Court of Appeals,
where the Court allowed the motions for intervention even when the case had already reached this
Court. Thus, in Mago the Court held that:
It is quite clear and patent that the motions for intervention filed by the movants at this stage of the
proceedings where trial had already been concluded x x x and on appeal x x x the same affirmed by the
Court of Appeals and the instant petition for certiorari to review said judgment is already submitted for
decision by the Supreme Court, are obviously and, manifestly late, beyond the period prescribed under x x x
Section 2, Rule 12 of the rules of Court.
But Rule 12 of the Rules of Court, like all other Rules therein promulgated, is simply a rule of procedure, the
whole purpose and object of which is to make the powers of the Court fully and completely available for
justice. The purpose of procedure is not to thwart justice. Its proper aim is to facilitate the application of
justice to the rival claims of contending parties. It was created not to hinder and delay but to facilitate and
promote the administration of justice. It does not constitute the thing itself which courts are always striving to
secure to litigants. It is designed as the means best adopted to obtain that thing. In other words, it is a
means to an end.
To be sure, the Court of Appeals did not pass upon the actual status of intervenors in relation to the Lot
as this was not in issue. Neither was the validity of the certificates of stewardship contracts which
intervenors allegedly possessed inquired into considering this too was not in issue. In fact, intervenors did
not specifically seek any relief apart from a declaration that the Lot in question remains inalienable land of
the public domain. We cannot fault the Court of Appeals for allowing the intervention, if only to provide the
rival groups a peaceful venue for ventilating their sides. This case has already claimed at least five lives due
to the raging dispute between the rival camps of the petitioners on one side and those of the DENR
awardees on the other. It also spawned a number of criminal cases between the two rival groups including
malicious mischief, robbery and arson. A strict application of the rules would blur this bigger, far more
important picture.
WHEREFORE, the Petition is DENIED. The Decision of the Court of Appeals dated June 22, 1992
declaring null and void the Decision dated January 30, 1991 of Branch 71, Regional Trial Court of Antipolo,
Rizal, in LRC No. 269-A, LRC Rec. No. N-59179 is AFFIRMED.
SO ORDERED.
Collado, et al. v. CA, et al., G.R. No. 107764, Oct.4,2002

Facts: Edna T. Collado (Collado) filed with the land registration court an application for registration of a Second, assuming that the Lot was alienable and disposable land prior to the issuance of EO 33 in 1904,
parcel of land with an approximate area of 1,200,766 square meters or 120.0766 hectares. The Lot is EO 33 reserved the Lot as a watershed. Since then, the Lot became nondisposable and inalienable public
situated in Barangay San Isidro, Antipolo, Rizal. Attached to the applicationwas the technical description of land. At the time Collado, et al. filed their application on April25,1985, the Lot has been reserved as a
the Lotwhich stated that, "[tJhis survey is inside IN-12 Mariquina Watershed." Collado filed an Amended watershed under EO 33 for 81 years prior to the filing of petitioners' application. The period of occupancy
Application to include additional co-applicants and subsequently, more applicants joined. after the issuance of EO 33 in 1904 could no longer be counted because as a watershed reservation, the Lot
was no longer susceptible of occupancy, disposition, conveyance or alienation. CA 141, applies exclusively
The Republic of the Philippines, through the Solicitor General, and the Municipality of Antipolo, through its to alienable and disposable public agricultural land. Forestlands, including watershed reservations, are
Municipal Attorney and the Provincial Fiscal of Rizal, filed oppositions to Collado, et al.'s application. In due excluded. It is axiomatic that the possession of forest lands or other inalienable public lands cannot ripen
course, the land registration court issued an order of general default against the whole world with the into private ownership.
exception of the oppositors.
Third, Based on the facts on record that neither Collado, et al. nor their predecessors-ininterest have been in
During the hearing, only the assistant provincial prosecutor appeared without the Solicitor General. For open, continuous, exclusive and notorious possession and occupation of the Lot for at least thirty years
failure of the oppositors to present their evidence, the land registration court issued an order considering the immediately preceding the filing of the application for confirmation of title.
case submitted for decision based on the evidence of the petitioners. The court set aside the order and reset
the hearing for the presentation of the evidence. Counsel for oppositors failed to appear again despite due
notice. Hence, the court issued an order submitting the case for decision based on the evidence of the
petitioners.

Lower Court's Ruling: The RTC held that Collado, et al. had adduced sufficient evidence to establish their
registrable rights over the Lot. Accordingly, the RTC rendered a decision confirming the imperfect title of
Collado, et al.

Appellate Court's Ruling: The CA declared null and void the decision of the RTC.

Issues: Whether Collado, et al. have registrable title over the lot

The Supreme Court's decision: Collado, et al. did not acquire private rights over the parcel of land prior to
the issuance of EO 33 segregating the same as a watershed reservation.

First. An applicant for confirmation of imperfect title bears the burden of proving that he meets the
requirements of CA 141. He must overcome the presumption that the land he is applying for is part of the
public domain and that he has an interest therein sufficient to warrant registration in his name arising from
an imperfect title. An imperfect title may have been derived from old Spanish grants such as a titulo real or
royal grant, a concession especial or special gtant, a composition con el estado or adjustment title, or a titulo
de compra or title through purchase. Or, that he has had continuous, open and notorious possession and
occupation of agricultural lands of the public domain under a bona fide claim of ownership for at least thirty
years preceding the filing of his application as provided by CA 14r.

Clearly, Collado, et al. were unable to acquire a valid and enforceable right or title because of the failure to
complete the required period of possession, whether under CA 141 prior to the issuance of EO 33, or under
the amendment by RA 1942 and PD 1073. There is no proof that prior to the issuance of EO 33 in 1904,
Collado, et al. had acquired ownership or title to the MDG-F 1919: Enhancing Access to and Provision of
Water Seruices with the Active Participation of the Poor for the Compilation and Analysis of Jurisprudence
on Water Supply Case Digests with of Development lmplications Lot either by deed or by any other mode of
acquisition from the State, as for instance by acquisitive prescription. As of 1904, Sesinando Leyva had only
been in possession for two years. Verily, Collado, et al. have not possessed the parcel of land in the manner
and for the number of years required by law for the confirmation of imperfect title.
G.R. No. 167707
Boracay Island in the Municipality of Malay, Aklan, with its powdery white sand beaches and warm
crystalline waters, is reputedly a premier Philippine tourist destination.The island is also home to 12,003
THE SECRETARY OF THE DEPARTMENT OF ENVIRONMENT inhabitants[4] who live in the bone-shaped islands three barangays.[5]
AND NATURAL RESOURCES, THE
REGIONAL EXECUTIVE Present: On April 14, 1976, the Department of Environment and Natural Resources (DENR)
DIRECTOR, DENR-REGION VI, approved the National Reservation Survey of Boracay
REGIONAL TECHNICAL PUNO, C.J., Island,[6] which identified several lots as being occupied or claimed by named persons. [7]
DIRECTOR FOR LANDS, QUISUMBING,
LANDS MANAGEMENT BUREAU, YNARES-SANTIAGO, On November 10, 1978, then President Ferdinand Marcos issued Proclamation
REGION VI PROVINCIAL CARPIO, No. 1801[8] declaring Boracay Island, among other islands, caves and peninsulas in the Philippines,
ENVIRONMENT AND NATURAL AUSTRIA-MARTINEZ, as tourist zones and marine reserves under the administration of the Philippine Tourism Authority
RESOURCES OFFICER OF KALIBO, CORONA,* (PTA). President Marcos later approved the issuance of PTACircular 3-82[9] dated September 3, 1982, to
AKLAN, REGISTER OF DEEDS, CARPIO MORALES, implement Proclamation No. 1801.
DIRECTOR OF LAND AZCUNA,
REGISTRATION AUTHORITY, TINGA, Claiming that Proclamation No. 1801 and PTA Circular No 3-82 precluded them from filing an
DEPARTMENT OF TOURISM CHICO-NAZARIO, application for judicial confirmation of imperfect title or survey of land for titling purposes, respondents-
SECRETARY, DIRECTOR OF VELASCO, JR., claimants
PHILIPPINE TOURISM NACHURA,** Mayor Jose S. Yap, Jr., Libertad Talapian, Mila Y. Sumndad, and Aniceto Yap filed a petition for declaratory
AUTHORITY, REYES, relief with the RTC in Kalibo, Aklan.
Petitioners, LEONARDO-DE CASTRO, and
BRION, JJ. In their petition, respondents-claimants alleged that Proclamation No. 1801 and PTA Circular No. 3-
- versus - 82 raised doubts on their right to secure titles over their occupied lands. They declared that they themselves,
or through their predecessors-in-interest, had been in open, continuous, exclusive, and notorious possession
and occupation in Boracay since June 12, 1945, or earlier since time immemorial. They declared their lands
MAYOR JOSE S. YAP, LIBERTAD for tax purposes and paid realty taxes on them. [10]
TALAPIAN, MILA Y. SUMNDAD, and
ANICETO YAP, in their behalf and Promulgated: Respondents-claimants posited that Proclamation No. 1801 and its implementing Circular did not
in behalf of all those similarly situated, place Boracay beyond the commerce of man. Since the Island was classified as a tourist zone, it was
Respondents. October 8, 2008 susceptible of private ownership. Under Section 48(b) of Commonwealth Act (CA) No. 141, otherwise known
as the Public Land Act, they had the right to have the lots registered in their names through judicial
x--------------------------------------------------x confirmation of imperfect titles.

The Republic, through the Office of the Solicitor General (OSG), opposed the petition for
DECISION declaratory relief. The OSG countered that Boracay Island was an unclassified land of the public domain. It
formed part of the mass of lands classified as public forest, which was not available for disposition pursuant
to Section 3(a) of Presidential Decree (PD) No. 705 or the Revised Forestry Code, [11] as amended.
REYES, R.T., J.:
The OSG maintained that respondents-claimants reliance on PD No. 1801 and PTA Circular No. 3-
AT stake in these consolidated cases is the right of the present occupants of Boracay Island to 82 was misplaced. Their right to judicial confirmation of title was governed by CA No. 141 and PD No.
secure titles over their occupied lands. 705. Since Boracay Island had not been classified as alienable and disposable, whatever possession they
had cannot ripen into ownership.
There are two consolidated petitions. The first is G.R. No. 167707, a petition for review
on certiorari of the Decision[1] of the Court of Appeals (CA) affirming that[2] of the Regional Trial Court (RTC) During pre-trial, respondents-claimants and the OSG stipulated on the following facts: (1)
in Kalibo, Aklan, which granted the petition for declaratory relief filed by respondents-claimants Mayor Jose respondents-claimants were presently in possession of parcels of land in Boracay Island; (2) these parcels
Yap, et al. and ordered the survey of Boracay for titling purposes. The second is G.R. No. 173775, a petition of land were planted with coconut trees and other natural growing trees; (3) the coconut trees had heights of
for prohibition, mandamus, and nullification of Proclamation No. 1064 [3] issued by President Gloria more or less twenty (20) meters and were planted more or less fifty (50) years ago; and (4) respondents-
Macapagal-Arroyo classifying Boracay into reserved forest and agricultural land. claimants declared the land they were occupying for tax purposes. [12]

The Antecedents The parties also agreed that the principal issue for resolution was purely legal: whether
Proclamation No. 1801 posed any legal hindrance or impediment to the titling of the lands in Boracay. They
G.R. No. 167707
decided to forego with the trial and to submit the case for resolution upon submission of their respective of the centerline of roads and trails, reserved for right-of-way and which shall form part of the area reserved
memoranda.[13] for forest land protection purposes.

The RTC took judicial notice[14] that certain parcels of land in Boracay Island, more particularly Lots On August 10, 2006, petitioners-claimants Dr. Orlando Sacay,[27] Wilfredo Gelito,[28] and other
1 and 30, Plan PSU-5344, were covered by Original Certificate of Title No. 19502 (RO 2222) in the name of landowners[29] in Boracay filed with this Court an original petition for prohibition, mandamus, and nullification
the Heirs of Ciriaco S. Tirol. These lots were involved in Civil Case Nos. 5222 and 5262 filed before of Proclamation No. 1064.[30] They allege that the Proclamation infringed on their prior vested rights over
the RTC of Kalibo, Aklan.[15]The titles were issued on portions of Boracay. They have been in continued possession of their respective lots in Boracay since time
August 7, 1933.[16] immemorial. They have also invested billions of pesos in developing their lands and building internationally
renowned first class resorts on their lots.[31]
RTC and CA Dispositions

On July 14, 1999, the RTC rendered a decision in favor of respondents-claimants, with Petitioners-claimants contended that there is no need for a proclamation reclassifying Boracay into
a fallo reading: agricultural land. Being classified as neither mineral nor timber land, the island is deemed agricultural
pursuant to the Philippine Bill of 1902 and Act No. 926, known as the first Public Land Act.[32] Thus, their
WHEREFORE, in view of the foregoing, the Court declares that Proclamation No. possession in the concept of owner for the required period entitled them to judicial confirmation of imperfect
1801 and PTA Circular No. 3-82 pose no legal obstacle to the petitioners and those title.
similarly situated to acquire title to their lands in Boracay, in accordance with the
applicable laws and in the manner prescribed therein; and to have their lands surveyed Opposing the petition, the OSG argued that petitioners-claimants do not have a vested right over
and approved by respondent Regional Technical Director of Lands as the approved their occupied portions in the island. Boracay is an unclassified public forest land pursuant to Section 3(a) of
survey does not in itself constitute a title to the land. PD No. 705. Being public forest, the claimed portions of the island are inalienable and cannot be the subject
of judicial confirmation of imperfect title. It is only the executive department, not the courts, which has
SO ORDERED.[17] authority to reclassify lands of the public domain into alienable and disposable lands. There is a need for a
positive government act in order to release the lots for disposition.
The RTC upheld respondents-claimants right to have their occupied lands titled in their name. It
ruled that neither Proclamation No. 1801 nor PTA Circular No. 3-82 mentioned that lands in Boracay were On November 21, 2006, this Court ordered the consolidation of the two petitions as they principally
inalienable or could not be the subject of disposition. [18] The Circular itself recognized private ownership of involve the same issues on the land classification of Boracay Island.[33]
lands.[19] The trial court cited Sections 87[20] and 53[21] of the Public Land Act as basis for acknowledging
private ownership of lands in Boracay and that only those forested areas in public lands were declared as Issues
part of the forest reserve.[22]
G.R. No. 167707
The OSG moved for reconsideration but its motion was denied. [23] The Republic then appealed to
the CA. The OSG raises the lone issue of whether Proclamation No. 1801 and PTA Circular No. 3-82 pose
any legal obstacle for respondents, and all those similarly situated, to acquire title to their occupied lands
On December 9, 2004, the appellate court affirmed in toto the RTC decision, disposing as follows: in Boracay Island.[34]

WHEREFORE, in view of the foregoing premises, judgment is hereby rendered


by us DENYING the appeal filed in this case and AFFIRMING the decision of the lower G.R. No. 173775
court.[24]
Petitioners-claimants hoist five (5) issues, namely:

The CA held that respondents-claimants could not be prejudiced by a declaration that the lands I.
they occupied since time immemorial were part of a forest reserve. AT THE TIME OF THE ESTABLISHED POSSESSION OF PETITIONERS IN CONCEPT
OF OWNER OVER THEIR RESPECTIVE AREAS IN BORACAY, SINCE TIME
Again, the OSG sought reconsideration but it was similarly denied. [25] Hence, the present petition IMMEMORIAL OR AT THE LATEST SINCE 30 YRS. PRIOR TO THE FILING OF THE
under Rule 45. PETITION FOR DECLARATORY RELIEF ON NOV. 19, 1997, WERE THE AREAS
OCCUPIED BY THEM PUBLIC AGRICULTURAL LANDS AS DEFINED
G.R. No. 173775 BY LAWS THEN ON JUDICIAL CONFIRMATION OF IMPERFECT TITLES OR PUBLIC
FOREST AS DEFINED BY SEC. 3a, PD 705?
On May 22, 2006, during the pendency of G.R. No. 167707, President Gloria Macapagal-Arroyo issued
Proclamation No. 1064[26] classifying Boracay Island into four hundred (400) hectares of reserved forest land II.
(protection purposes) and six hundred twenty-eight and 96/100 (628.96) hectares of agricultural land HAVE PETITIONERS OCCUPANTS ACQUIRED PRIOR VESTED RIGHT OF PRIVATE
(alienable and disposable). The Proclamation likewise provided for a fifteen-meter buffer zone on each side OWNERSHIP OVER THEIR OCCUPIED PORTIONS OF BORACAY LAND, DESPITE
THE FACT THAT THEY HAVE NOT APPLIED YET FOR JUDICIAL CONFIRMATION OF The Regalian Doctrine dictates that all lands of the public domain belong to the State, that the State
IMPERFECT TITLE? is the source of any asserted right to ownership of land and charged with the conservation of such
patrimony.[45] The doctrine has been consistently adopted under the 1935, 1973, and 1987 Constitutions. [46]
III.
IS THE EXECUTIVE DECLARATION OF THEIR AREAS AS All lands not otherwise appearing to be clearly within private ownership are presumed to belong to
ALIENABLE AND DISPOSABLE UNDER SEC 6, CA 141 [AN] INDISPENSABLE PRE- the State.[47] Thus, all lands that have not been acquired from the government, either by purchase or by
REQUISITE FOR PETITIONERS TO OBTAIN TITLE UNDER THE TORRENS SYSTEM? grant, belong to the State as part of the inalienable public domain. [48] Necessarily, it is up to the State to
determine if lands of the public domain will be disposed of for private ownership. The government, as the
IV. agent of the state, is possessed of the plenary power as the persona in law to determine who shall be the
IS THE ISSUANCE OF PROCLAMATION 1064 ON MAY 22, 2006, VIOLATIVE OF THE favored recipients of public lands, as well as under what terms they may be granted such privilege, not
PRIOR VESTED RIGHTS TO PRIVATE OWNERSHIP OF PETITIONERS OVER THEIR excluding the placing of obstacles in the way of their exercise of what otherwise would be ordinary acts of
LANDS IN BORACAY, PROTECTED BY THE DUE PROCESS CLAUSE OF THE ownership.[49]
CONSTITUTION OR IS PROCLAMATION 1064 CONTRARY TO SEC. 8, CA 141,
OR SEC. 4(a) OF RA 6657. Our present land law traces its roots to the Regalian Doctrine. Upon the Spanish conquest of
the Philippines, ownership of all lands, territories and possessions in the Philippines passed to the Spanish
V. Crown.[50] The Regalian doctrine was first introduced in the Philippines through the Laws of the Indies and
CAN RESPONDENTS BE COMPELLED BY MANDAMUS TO ALLOW THE the Royal Cedulas, which laid the foundation that all lands that were not acquired from the Government,
SURVEY AND TO APPROVE THE SURVEY PLANS FOR PURPOSES OF THE either by purchase or by grant, belong to the public domain. [51]
APPLICATION FOR TITLING OF THE LANDS OF PETITIONERS IN
BORACAY?[35] (Underscoring supplied) The Laws of the Indies was followed by the Ley Hipotecaria or the Mortgage Law of 1893. The
Spanish Mortgage Law provided for the systematic registration of titles and deeds as well as possessory
In capsule, the main issue is whether private claimants (respondents-claimants in G.R. No. 167707 claims.[52]
and petitioners-claimants in G.R. No. 173775) have a right to secure titles over their occupied portions in
Boracay. The twin petitions pertain to their right, if any, to judicial confirmation of imperfect title under CA No. The Royal Decree of 1894 or the Maura Law[53] partly amended the Spanish Mortgage Law and
141, as amended. They do not involve their right to secure title under other pertinent laws. the Laws of the Indies. It established possessory information as the method of legalizing possession of
vacant Crown land, under certain conditions which were set forth in said decree. [54] Under Section 393 of the
Our Ruling Maura Law, an informacion posesoria or possessory information title,[55] when duly inscribed in the Registry
of Property, is converted into a title of ownership only after the lapse of twenty (20) years of uninterrupted
Regalian Doctrine and power of the executive possession which must be actual, public, and adverse, [56] from the date of its inscription.[57] However,
to reclassify lands of the public domain possessory information title had to be perfected one year after the promulgation of the Maura Law, or
until April 17, 1895. Otherwise, the lands would revert to the State.[58]
Private claimants rely on three (3) laws and executive acts in their bid for judicial confirmation of
imperfect title, namely: (a) Philippine Bill of 1902 [36] in relation to Act No. 926, later amended and/or In sum, private ownership of land under the Spanish regime could only be founded on royal
superseded by Act No. 2874 and CA No. 141;[37] (b) Proclamation No. 1801[38] issued by then President concessions which took various forms, namely: (1) titulo real or royal grant; (2) concesion especial or special
Marcos; and (c) Proclamation No. 1064[39] issued by President Gloria Macapagal-Arroyo. We shall proceed grant; (3) composicion con el estado or adjustment title; (4) titulo de compra or title by purchase; and
to determine their rights to apply for judicial confirmation of imperfect title under these laws and executive (5) informacion posesoria or possessory information title.[59]
acts.
The first law governing the disposition of public lands in the Philippines under American rule was
But first, a peek at the Regalian principle and the power of the executive to reclassify lands of the embodied in the Philippine Bill of 1902.[60] By this law, lands of the public domain in the Philippine Islands
public domain. were classified into three (3) grand divisions, to wit: agricultural, mineral, and timber or forest lands.[61] The
act provided for, among others, the disposal of mineral lands by means of absolute grant (freehold system)
The 1935 Constitution classified lands of the public domain into agricultural, forest or and by lease (leasehold system).[62] It also provided the definition by exclusion of agricultural public
timber.[40] Meanwhile, the 1973 Constitution provided the following divisions: agricultural, industrial or lands.[63] Interpreting the meaning of agricultural lands under the Philippine Bill of 1902, the Court declared
commercial, residential, resettlement, mineral, timber or forest and grazing lands, and such other classes as in Mapa v. Insular Government:[64]
may be provided by law,[41] giving the government great leeway for classification.[42] Then the 1987
Constitution reverted to the 1935 Constitution classification with one addition: national parks.[43] Of
these, onlyagricultural lands may be alienated.[44] Prior to Proclamation No. 1064 of May 22, x x x In other words, that the phrase agricultural land as used in Act No. 926
2006, Boracay Island had never been expressly and administratively classified under any of these grand means those public lands acquired from Spain which are not timber or mineral
divisions. Boracay was an unclassified land of the public domain. lands. x x x[65](Emphasis Ours)
On February 1, 1903, the Philippine Legislature passed Act No. 496, otherwise known as the Land disposable. To prove that the land subject of an application for registration is alienable, the applicant must
Registration Act. The act established a system of registration by which recorded title becomes absolute, establish the existence of a positive act of the government such as a presidential proclamation or an
indefeasible, and imprescriptible. This is known as the Torrens system.[66] executive order; an administrative action; investigation reports of Bureau of Lands investigators; and a
legislative act or a statute.[85] The applicant may also secure a certification from the government that the land
Concurrently, on October 7, 1903, the Philippine Commission passed Act No. 926, which was the claimed to have been possessed for the required number of years is alienable and disposable. [86]
first Public Land Act. The Act introduced the homestead system and made provisions for judicial and
administrative confirmation of imperfect titles and for the sale or lease of public lands. It permitted In the case at bar, no such proclamation, executive order, administrative action, report, statute, or
corporations regardless of the nationality of persons owning the controlling stock to lease or purchase lands certification was presented to the Court. The records are bereft of evidence showing that, prior to 2006, the
of the public domain.[67] Under the Act, open, continuous, exclusive, and notorious possession and portions of Boracay occupied by private claimants were subject of a government proclamation that the land
occupation of agricultural lands for the next ten (10) years preceding July 26, 1904 was sufficient for judicial is alienable and disposable.Absent such well-nigh incontrovertible evidence, the Court cannot accept the
confirmation of imperfect title.[68] submission that lands occupied by private claimants were already open to disposition before 2006. Matters
of land classification or reclassification cannot be assumed. They call for proof.[87]
On November 29, 1919, Act No. 926 was superseded by Act No. 2874, otherwise known as the
second Public Land Act. This new, more comprehensive law limited the exploitation of agricultural lands to Ankron and De Aldecoa did not make the whole of Boracay Island, or portions of it,
Filipinos and Americans and citizens of other countries which gave Filipinos the same privileges. For judicial agricultural lands. Private claimants posit that Boracay was already an agricultural land pursuant to the old
confirmation of title, possession and occupation en concepto dueo since time immemorial, or since July 26, cases Ankron v. Government of the Philippine Islands (1919)[88] and De Aldecoa v. The Insular Government
1894, was required.[69] (1909).[89] These cases were decided under the provisions of the Philippine Bill of 1902 and Act No.
926. There is a statement in these old cases that in the absence of evidence to the contrary, that in each
After the passage of the 1935 Constitution, CA No. 141 amended Act No. 2874 on December 1, case the lands are agricultural lands until the contrary is shown. [90]
1936. To this day, CA No. 141, as amended, remains as the existing general law governing the
classification and disposition of lands of the public domain other than timber and mineral lands, [70] and Private claimants reliance on Ankron and De Aldecoa is misplaced. These cases did not have the
privately owned lands which reverted to the State.[71] effect of converting the whole of Boracay Island or portions of it into agricultural lands. It should be stressed
that the Philippine Bill of 1902 and Act No. 926 merely provided the manner through which land registration
Section 48(b) of CA No. 141 retained the requirement under Act No. 2874 of possession and courts would classify lands of the public domain. Whether the land would be classified as timber, mineral, or
occupation of lands of the public domain since time immemorial or since July 26, 1894. However, this agricultural depended on proof presented in each case.
provision was superseded by Republic Act (RA) No. 1942, [72] which provided for a simple thirty-year
prescriptive period for judicial confirmation of imperfect title. The provision was last amended by PD No. Ankron and De Aldecoa were decided at a time when the President of the Philippines had no power
1073,[73] which now provides for possession and occupation of the land applied for since June 12, 1945, or to classify lands of the public domain into mineral, timber, and agricultural. At that time, the courts were free
earlier.[74] to make corresponding classifications in justiciable cases, or were vested with implicit power to do so,
depending upon the preponderance of the evidence. [91] This was the Courts ruling in Heirs of the Late
The issuance of PD No. 892[75] on February 16, 1976 discontinued the use of Spanish titles as Spouses Pedro S. Palanca and Soterranea Rafols Vda. De Palanca v. Republic,[92] in which it stated,
evidence in land registration proceedings.[76] Under the decree, all holders of Spanish titles or grants should through Justice Adolfo Azcuna, viz.:
apply for registration of their lands under Act No. 496 within six (6) months from the effectivity of the decree
on February 16, 1976. Thereafter, the recording of all unregistered lands[77] shall be governed by Section x x x Petitioners furthermore insist that a particular land need not be formally
194 of the Revised Administrative Code, as amended by Act No. 3344. released by an act of the Executive before it can be deemed open to private ownership,
citing the cases of Ramos v. Director of Lands and Ankron v. Government of the
On June 11, 1978, Act No. 496 was amended and updated by PD No. 1529, known as the Property Philippine Islands.
Registration Decree. It was enacted to codify the various laws relative to registration of property. [78] It
governs registration of lands under the Torrens system as well as unregistered lands, including chattel xxxx
mortgages.[79]
Petitioners reliance upon Ramos v. Director of Lands and Ankron v.
A positive act declaring land as alienable and disposable is required. In keeping with the Government is misplaced. These cases were decided under the Philippine Bill of 1902 and
presumption of State ownership, the Court has time and again emphasized that there must be a positive the first Public Land Act No. 926 enacted by the Philippine Commission on October 7,
act of the government, such as an official proclamation,[80] declassifying inalienable public land into 1926, under which there was no legal provision vesting in the Chief Executive or President
disposable land for agricultural or other purposes.[81] In fact, Section 8 of CA No. 141 limits alienable or of the Philippines the power to classify lands of the public domain into mineral, timber and
disposable lands only to those lands which have been officially delimited and classified. [82] agricultural so that the courts then were free to make corresponding classifications in
justiciable cases, or were vested with implicit power to do so, depending upon the
The burden of proof in overcoming the presumption of State ownership of the lands of the public preponderance of the evidence.[93]
domain is on the person applying for registration (or claiming ownership), who must prove that the land
subject of the application is alienable or disposable. [83] To overcome this presumption, incontrovertible To aid the courts in resolving land registration cases under Act No. 926, it was then necessary to
evidence must be established that the land subject of the application (or claim) is alienable or devise a presumption on land classification. Thus evolved the dictum in Ankron that the courts have a right
disposable.[84] There must still be a positive act declaring land of the public domain as alienable and
to presume, in the absence of evidence to the contrary, that in each case the lands are agricultural lands land. The Government, in the first instance, under the provisions of Act No. 1148, may, by
until the contrary is shown.[94] reservation, decide for itself what portions of public land shall be considered forestry land,
unless private interests have intervened before such reservation is made. In the latter
case, whether the land is agricultural, forestry, or mineral, is a question of proof. Until
private interests have intervened, the Government, by virtue of the terms of said Act (No.
But We cannot unduly expand the presumption in Ankron and De Aldecoa to an argument that all 1148), may decide for itself what portions of the public domain shall be set aside and
lands of the public domain had been automatically reclassified as disposable and alienable agricultural reserved as forestry or mineral land. (Ramos vs. Director of Lands, 39 Phil. 175; Jocson
lands. By no stretch of imagination did the presumption convert all lands of the public domain into vs. Director of Forestry, supra)[95] (Emphasis ours)
agricultural lands.
Since 1919, courts were no longer free to determine the classification of lands from the facts of
If We accept the position of private claimants, the Philippine Bill of 1902 and Act No. 926 would each case, except those that have already became private lands. [96] Act No. 2874, promulgated in 1919 and
have automatically made all lands in the Philippines, except those already classified as timber or mineral reproduced in Section 6 of CA No. 141, gave the Executive Department, through the President,
land, alienable and disposable lands. That would take these lands out of State ownership and worse, would the exclusive prerogative to classify or reclassify public lands into alienable or disposable, mineral or
be utterly inconsistent with and totally repugnant to the long-entrenched Regalian doctrine. forest.96-a Since then, courts no longer had the authority, whether express or implied, to determine the
classification of lands of the public domain.[97]
The presumption in Ankron and De Aldecoa attaches only to land registration cases brought under
the provisions of Act No. 926, or more specifically those cases dealing with judicial and administrative Here, private claimants, unlike the Heirs of Ciriaco Tirol who were issued their title in 1933, [98] did
confirmation of imperfect titles. The presumption applies to an applicant for judicial or administrative not present a justiciable case for determination by the land registration court of the propertys land
conformation of imperfect title under Act No. 926. It certainly cannot apply to landowners, such as private classification. Simply put, there was no opportunity for the courts then to resolve if the land the Boracay
claimants or their predecessors-in-interest, who failed to avail themselves of the benefits of Act No. 926. As occupants are now claiming were agricultural lands. When Act No. 926 was supplanted by Act No. 2874 in
to them, their land remained unclassified and, by virtue of the Regalian doctrine, continued to be owned by 1919, without an application for judicial confirmation having been filed by private claimants or their
the State. predecessors-in-interest, the courts were no longer authorized to determine the propertys land
classification. Hence, private claimants cannot bank on Act No. 926.
In any case, the assumption in Ankron and De Aldecoa was not absolute. Land classification was,
in the end, dependent on proof. If there was proof that the land was better suited for non-agricultural uses, We note that the RTC decision[99] in G.R. No. 167707 mentioned Krivenko v. Register of Deeds of
[100]
the courts could adjudge it as a mineral or timber land despite the presumption. In Ankron, this Court stated: Manila, which was decided in 1947 when CA No. 141, vesting the Executive with the sole power to
classify lands of the public domain was already in effect. Krivenko cited the old cases Mapa v. Insular
In the case of Jocson vs. Director of Forestry (supra), the Attorney-General Government,[101] De Aldecoa v. The Insular Government,[102] and Ankron v. Government of the
admitted in effect that whether the particular land in question belongs to one class or Philippine Islands.[103]
another is a question of fact. The mere fact that a tract of land has trees upon it or has
mineral within it is not of itself sufficient to declare that one is forestry land and the other, Krivenko, however, is not controlling here because it involved a totally different issue. The pertinent
mineral land. There must be some proof of the extent and present or future value of the issue in Krivenko was whether residential lots were included in the general classification of agricultural
forestry and of the minerals. While, as we have just said, many definitions have been lands; and if so, whether an alien could acquire a residential lot. This Court ruled that as an alien, Krivenko
given for agriculture, forestry, and mineral lands, and that in each case it is a question of was prohibited by the 1935 Constitution[104] from acquiring agricultural land, which included residential
fact, we think it is safe to say that in order to be forestry or mineral land the proof must lots. Here, the issue is whether unclassified lands of the public domain are automatically deemed
show that it is more valuable for the forestry or the mineral which it contains than it is for agricultural.
agricultural purposes. (Sec. 7, Act No. 1148.) It is not sufficient to show that there exists
some trees upon the land or that it bears some mineral. Land may be classified as forestry
or mineral today, and, by reason of the exhaustion of the timber or mineral, be classified Notably, the definition of agricultural public lands mentioned in Krivenko relied on the old cases
as agricultural land tomorrow. And vice-versa, by reason of the rapid growth of timber or decided prior to the enactment of Act No. 2874, including Ankron and De Aldecoa.[105] As We have already
the discovery of valuable minerals, lands classified as agricultural today may be differently stated, those cases cannot apply here, since they were decided when the Executive did not have the
classified tomorrow. Each case must be decided upon the proof in that particular authority to classify lands as agricultural, timber, or mineral.
case, having regard for its present or future value for one or the other purposes. We
believe, however, considering the fact that it is a matter of public knowledge that a majority Private claimants continued possession under Act No. 926 does not create a presumption
of the lands in the Philippine Islands are agricultural lands that the courts have a right to that the land is alienable. Private claimants also contend that their continued possession of portions
presume, in the absence of evidence to the contrary, that in each case the lands are of Boracay Island for the requisite period of ten (10) years under Act No. 926 [106] ipso facto converted the
agricultural lands until the contrary is shown. Whatever the land involved in a particular island into private ownership. Hence, they may apply for a title in their name.
land registration case is forestry or mineral land must, therefore, be a matter of
proof. Its superior value for one purpose or the other is a question of fact to be A similar argument was squarely rejected by the Court in Collado v. Court of
settled by the proof in each particular case. The fact that the land is a manglar Appeals.[107] Collado, citing the separate opinion of now Chief Justice Reynato S. Puno in Cruz v. Secretary
[mangrove swamp] is not sufficient for the courts to decide whether it is agricultural, of Environment and Natural Resources,107-a ruled:
forestry, or mineral land. It may perchance belong to one or the other of said classes of
Act No. 926, the first Public Land Act, was passed in cover. Parcels of land classified as forest land may actually be covered with grass or
pursuance of the provisions of the Philippine Bill of 1902. The law planted to crops by kaingin cultivators or other farmers. Forest lands do not have to be on
governed the disposition of lands of the public domain. It prescribed mountains or in out of the way places. Swampy areas covered by mangrove trees, nipa
rules and regulations for the homesteading, selling and leasing of palms, and other trees growing in brackish or sea water may also be classified as forest
portions of the public domain of the Philippine Islands, and prescribed land. The classification is descriptive of its legal nature or status and does not have
the terms and conditions to enable persons to perfect their titles to to be descriptive of what the land actually looks like. Unless and until the land
public lands in the Islands. It also provided for the issuance of patents to classified as forest is released in an official proclamation to that effect so that it may form
certain native settlers upon public lands, for the establishment of town part of the disposable agricultural lands of the public domain, the rules on confirmation of
sites and sale of lots therein, for the completion of imperfect titles, and imperfect title do not apply.[115] (Emphasis supplied)
for the cancellation or confirmation of Spanish concessions and grants
in the Islands. In short, the Public Land Act operated on the assumption There is a big difference between forest as defined in a dictionary and forest or timber land as a
that title to public lands in the Philippine Islands remained in the classification of lands of the public domain as appearing in our statutes. One is descriptive of what appears
government; and that the governments title to public land sprung from on the land while the other is a legal status, a classification for legal purposes. [116] At any rate, the Court is
the Treaty of Paris and other subsequent treaties between Spain and tasked to determine the legal status of Boracay Island, and not look into its physical layout. Hence, even if
the United States. The term public land referred to all lands of the public its forest cover has been replaced by beach resorts, restaurants and other commercial establishments, it has
domain whose title still remained in the government and are thrown not been automatically converted from public forest to alienable agricultural land.
open to private appropriation and settlement, and excluded the
patrimonial property of the government and the friar lands. Private claimants cannot rely on Proclamation No. 1801 as basis for judicial confirmation of
imperfect title. The proclamation did not convert Boracay into an agricultural land. However, private
Thus, it is plain error for petitioners to argue that under the Philippine Bill of 1902 claimants argue that Proclamation No. 1801 issued by then President Marcos in 1978 entitles them to
and Public Land Act No. 926, mere possession by private individuals of lands judicial confirmation of imperfect title. The Proclamation classified Boracay, among other islands, as a tourist
creates the legal presumption that the lands are alienable and zone. Private claimants assert that, as a tourist spot, the island is susceptible of private ownership.
disposable.[108] (Emphasis Ours)
Proclamation No. 1801 or PTA Circular No. 3-82 did not convert the whole of Boracay into an
Except for lands already covered by existing titles, Boracay was an unclassified land of the agricultural land. There is nothing in the law or the Circular which made Boracay Island an agricultural
public domain prior to Proclamation No. 1064. Such unclassified lands are considered public forest land. The reference in Circular No. 3-82 to private lands[117] and areas declared as alienable and
under PD No. 705. The DENR[109] and the National Mapping and Resource Information Authority[110] certify disposable[118] does not by itself classify the entire island as agricultural. Notably, Circular No. 3-82 makes
that Boracay Island is an unclassified land of the public domain. reference not only to private lands and areas but also to public forested lands. Rule VIII, Section 3 provides:

PD No. 705 issued by President Marcos categorized all unclassified lands of the public domain as No trees in forested private lands may be cut without prior authority from
public forest. Section 3(a) of PD No. 705 defines a public forest as a mass of lands of the public domain the PTA. All forested areas in public lands are declared forest reserves. (Emphasis
which has not been the subject of the present system of classification for the determination of which lands supplied)
are needed for forest purpose and which are not. Applying PD No. 705, all unclassified lands, including
those in Boracay Island, are ipso facto considered public forests. PD No. 705, however, respects titles Clearly, the reference in the Circular to both private and public lands merely recognizes that the
already existing prior to its effectivity. island can be classified by the Executive department pursuant to its powers under CA No. 141. In fact,
Section 5 of the Circular recognizes the then Bureau of Forest Developments authority to declare areas in
The Court notes that the classification of Boracay as a forest land under PD No. 705 may seem to the island as alienable and disposable when it provides:
be out of touch with the present realities in the island. Boracay, no doubt, has been partly stripped of its
forest cover to pave the way for commercial developments. As a premier tourist destination for local and Subsistence farming, in areas declared as alienable and disposable by the
foreign tourists, Boracay appears more of a commercial island resort, rather than a forest land. Bureau of Forest Development.

Nevertheless, that the occupants of Boracay have built multi-million peso beach resorts on the Therefore, Proclamation No. 1801 cannot be deemed the positive act needed to
island;[111] that the island has already been stripped of its forest cover; or that the implementation of classify Boracay Island as alienable and disposable land. If President Marcos intended to classify the island
Proclamation No. 1064 will destroy the islands tourism industry, do not negate its character as public forest. as alienable and disposable or forest, or both, he would have identified the specific limits of each, as
President Arroyo did in Proclamation No. 1064. This was not done in Proclamation No. 1801.
Forests, in the context of both the Public Land Act and the Constitution[112] classifying lands of the
public domain into agricultural, forest or timber, mineral lands, and national parks, do not necessarily refer to The Whereas clauses of Proclamation No. 1801 also explain the rationale behind the declaration of
large tracts of wooded land or expanses covered by dense growths of trees and underbrushes. [113] The Boracay Island, together with other islands, caves and peninsulas in the Philippines, as a tourist zone and
discussion in Heirs of Amunategui v. Director of Forestry[114] is particularly instructive: marine reserve to be administered by the PTA to ensure the concentrated efforts of the public and private
sectors in the development of the areas tourism potential with due regard for ecological balance in the
A forested area classified as forest land of the public domain does not lose such marine environment. Simply put, the proclamation is aimed at administering the islands for tourism and
classification simply because loggers or settlers may have stripped it of its forest ecological purposes. It does not address the areas alienability.[119]
More importantly, Proclamation No. 1801 covers not only Boracay Island, but sixty-four (64) other In Heirs of the Late Spouses Pedro S. Palanca and Soterranea Rafols v. Republic,[124] the Court
islands, coves, and peninsulas in the Philippines, such as Fortune and Verde Islands in Batangas, Port stated that unclassified lands are public forests.
Galera in Oriental Mindoro, Panglao and Balicasag Islands in Bohol, Coron Island, Puerto Princesa and
surrounding areas in Palawan, Camiguin Island in Cagayan de Oro, and Misamis Oriental, to name a few. If
the designation of Boracay Island as tourist zone makes it alienable and disposable by virtue of While it is true that the land classification map does not categorically state
Proclamation No. 1801, all the other areas mentioned would likewise be declared wide open for private that the islands are public forests, the fact that they were unclassified lands leads
disposition. That could not have been, and is clearly beyond, the intent of the proclamation. to the same result.In the absence of the classification as mineral or timber land, the land
remains unclassified land until released and rendered open to disposition.[125] (Emphasis
It was Proclamation No. 1064 of 2006 which positively declared part of Boracay as alienable supplied)
and opened the same to private ownership. Sections 6 and 7 of CA No. 141[120] provide that it is only the
President, upon the recommendation of the proper department head, who has the authority to classify the Moreover, the prohibition under the CARL applies only to a reclassification of land. If the land had
lands of the public domain into alienable or disposable, timber and mineral lands. [121] never been previously classified, as in the case of Boracay, there can be no prohibited reclassification under
the agrarian law. We agree with the opinion of the Department of Justice[126] on this point:
In issuing Proclamation No. 1064, President Gloria Macapagal-Arroyo merely exercised the
authority granted to her to classify lands of the public domain, presumably subject to existing vested Indeed, the key word to the correct application of the prohibition in Section 4(a) is
rights. Classification of public lands is the exclusive prerogative of the Executive Department, through the the word reclassification. Where there has been no previous classification of public forest
Office of the President. Courts have no authority to do so.[122] Absent such classification, the land remains [referring, we repeat, to the mass of the public domain which has not been the subject of
unclassified until released and rendered open to disposition. [123] the present system of classification for purposes of determining which are needed for
forest purposes and which are not] into permanent forest or forest reserves or some other
Proclamation No. 1064 classifies Boracay into 400 hectares of reserved forest land and 628.96 forest uses under the Revised Forestry Code, there can be no reclassification of forest
hectares of agricultural land. The Proclamation likewise provides for a 15-meter buffer zone on each side of lands to speak of within the meaning of Section 4(a).
the center line of roads and trails, which are reserved for right of way and which shall form part of the area
reserved for forest land protection purposes. Thus, obviously, the prohibition in Section 4(a) of the CARL against the
Contrary to private claimants argument, there was nothing invalid or irregular, much less reclassification of forest lands to agricultural lands without a prior law delimiting the limits
unconstitutional, about the classification of Boracay Island made by the President through Proclamation No. of the public domain, does not, and cannot, apply to those lands of the public domain,
1064. It was within her authority to make such classification, subject to existing vested rights. denominated as public forest under the Revised Forestry Code, which have not been
previously determined, or classified, as needed for forest purposes in accordance with the
Proclamation No. 1064 does not violate the Comprehensive Agrarian Reform Law. Private provisions of the Revised Forestry Code.[127]
claimants further assert that Proclamation No. 1064 violates the provision of the Comprehensive Agrarian
Reform Law (CARL) or RA No. 6657 barring conversion of public forests into agricultural lands. They claim Private claimants are not entitled to apply for judicial confirmation of imperfect title under
that since Boracay is a public forest under PD No. 705, President Arroyo can no longer convert it into an CA No. 141. Neither do they have vested rights over the occupied lands under the said law. There are
agricultural land without running afoul of Section 4(a) of RA No. 6657, thus: two requisites for judicial confirmation of imperfect or incomplete title under CA No. 141, namely: (1) open,
continuous, exclusive, and notorious possession and occupation of the subject land by himself or through his
SEC. 4. Scope. The Comprehensive Agrarian Reform Law of 1988 shall cover, predecessors-in-interest under a bona fide claim of ownership since time immemorial or from June 12, 1945;
regardless of tenurial arrangement and commodity produced, all public and private and (2) the classification of the land as alienable and disposable land of the public domain.[128]
agricultural lands as provided in Proclamation No. 131 and Executive Order No. 229,
including other lands of the public domain suitable for agriculture. As discussed, the Philippine Bill of 1902, Act No. 926, and Proclamation No. 1801 did not convert
portions of Boracay Island into an agricultural land. The island remained an unclassified land of the public
More specifically, the following lands are covered by the Comprehensive domain and, applying the Regalian doctrine, is considered State property.
Agrarian Reform Program:
Private claimants bid for judicial confirmation of imperfect title, relying on the Philippine Bill of 1902,
(a) All alienable and disposable lands of the public domain devoted to or Act No. 926, and Proclamation No. 1801, must fail because of the absence of the second element of
suitable for agriculture. No reclassification of forest or mineral alienable and disposable land. Their entitlement to a government grant under our present Public Land Act
lands to agricultural lands shall be undertaken after the approval of presupposes that the land possessed and applied for is already alienable and disposable. This is clear from
this Act until Congress, taking into account ecological, the wording of the law itself.[129] Where the land is not alienable and disposable, possession of the land, no
developmental and equity considerations, shall have determined matter how long, cannot confer ownership or possessory rights. [130]
by law, the specific limits of the public domain.
Neither may private claimants apply for judicial confirmation of imperfect title under Proclamation
That Boracay Island was classified as a public forest under PD No. 705 did not bar the Executive No. 1064, with respect to those lands which were classified as agricultural lands. Private claimants failed to
from later converting it into agricultural land. Boracay Island still remained an unclassified land of the public prove the first element of open, continuous, exclusive, and notorious possession of their lands in Boracay
domain despite PD No. 705. since June 12, 1945.
To be sure, forest lands are fundamental to our nations survival. Their promotion and protection are
We cannot sustain the CA and RTC conclusion in the petition for declaratory relief that private not just fancy rhetoric for politicians and activists. These are needs that become more urgent as destruction
claimants complied with the requisite period of possession. of our environment gets prevalent and difficult to control. As aptly observed by Justice Conrado Sanchez in
1968 in Director of Forestry v. Munoz:[134]
The tax declarations in the name of private claimants are insufficient to prove the first element of
possession. We note that the earliest of the tax declarations in the name of private claimants were issued in The view this Court takes of the cases at bar is but in adherence to public policy
1993. Being of recent dates, the tax declarations are not sufficient to convince this Court that the period of that should be followed with respect to forest lands. Many have written much, and many
possession and occupation commenced on June 12, 1945. more have spoken, and quite often, about the pressing need for forest preservation,
conservation, protection, development and reforestation. Not without justification. For,
Private claimants insist that they have a vested right in Boracay, having been in possession of the forests constitute a vital segment of any country's natural resources. It is of common
island for a long time. They have invested millions of pesos in developing the island into a tourist spot. They knowledge by now that absence of the necessary green cover on our lands produces a
say their continued possession and investments give them a vested right which cannot be unilaterally number of adverse or ill effects of serious proportions. Without the trees, watersheds dry
rescinded by Proclamation No. 1064. up; rivers and lakes which they supply are emptied of their contents. The fish disappear.
Denuded areas become dust bowls. As waterfalls cease to function, so will hydroelectric
The continued possession and considerable investment of private claimants do not automatically plants. With the rains, the fertile topsoil is washed away; geological erosion results. With
give them a vested right in Boracay. Nor do these give them a right to apply for a title to the land they are erosion come the dreaded floods that wreak havoc and destruction to property crops,
presently occupying. This Court is constitutionally bound to decide cases based on the evidence presented livestock, houses, and highways not to mention precious human lives. Indeed, the
and the laws applicable. As the law and jurisprudence stand, private claimants are ineligible to apply for a foregoing observations should be written down in a lumbermans decalogue. [135]
judicial confirmation of title over their occupied portions in Boracay even with their continued possession and
considerable investment in the island. WHEREFORE, judgment is rendered as follows:

One Last Note 1. The petition for certiorari in G.R. No. 167707 is GRANTED and the Court of Appeals Decision
in CA-G.R. CV No. 71118 REVERSED AND SET ASIDE.
The Court is aware that millions of pesos have been invested for the development
of Boracay Island, making it a by-word in the local and international tourism industry. The Court also notes 2. The petition for certiorari in G.R. No. 173775 is DISMISSED for lack of merit.
that for a number of years, thousands of people have called the island their home. While the Court
commiserates with private claimants plight, We are bound to apply the law strictly and judiciously. This is the SO ORDERED.
law and it should prevail. Ito ang batas at ito ang dapat umiral.

All is not lost, however, for private claimants. While they may not be eligible to apply for judicial
confirmation of imperfect title under Section 48(b) of CA No. 141, as amended, this does not denote their
automatic ouster from the residential, commercial, and other areas they possess now classified as
agricultural. Neither will this mean the loss of their substantial investments on their occupied alienable
lands. Lack of title does not necessarily mean lack of right to possess.

For one thing, those with lawful possession may claim good faith as builders of
improvements. They can take steps to preserve or protect their possession. For another, they may look into
other modes of applying for original registration of title, such as by homestead [131] or sales patent,[132] subject
to the conditions imposed by law.

More realistically, Congress may enact a law to entitle private claimants to acquire title to their
occupied lots or to exempt them from certain requirements under the present land laws. There is one such
bill[133] now pending in the House of Representatives. Whether that bill or a similar bill will become a law is
for Congress to decide.

In issuing Proclamation No. 1064, the government has taken the step necessary to open up the
island to private ownership. This gesture may not be sufficient to appease some sectors which view the
classification of the island partially into a forest reserve as absurd. That the island is no longer overrun by
trees, however, does not becloud the vision to protect its remaining forest cover and to strike a healthy
balance between progress and ecology. Ecological conservation is as important as economic progress.
Secretary of DENR vs Yap
Natural Resources and Environmental Laws: Regalian Doctrine
GR No. 167707; Oct 8, 2008
FACTS: This petition is for a review on certiorari of the decision of the Court of Appeals (CA) affirming that
of the Regional Trial Court (RTC) in Kalibo Aklan, which granted the petition for declaratory relief filed by
respondents-claimants Mayor Jose Yap et al, and ordered the survey of Boracay for titling purposes.
On Nov. 10, 1978, President Marcos issued Proclamation No. 1801 declaring Boracay Island as a tourist
zone and marine reserve. Claiming that Proc. No. 1801 precluded them from filing an application for a
judicial confirmation of imperfect title or survey of land for titling purposes, respondents-claimants filed a
petition for declaratory relief with the RTC in Kalibo, Aklan.
The Republic, through the Office of the Solicitor General (OSG) opposed the petition countering that
Boracay Island was an unclassified land of the public domain. It formed part of the mass of lands classified
as “public forest,” which was not available for disposition pursuant to section 3(a) of PD No. 705 or the
Revised Forestry Code.
ISSUE: Whether unclassified lands of the public domain are automatically deemed agricultural land,
therefore making these lands alienable.
HELD: No. To prove that the land subject of an application for registration is alienable, the applicant must
establish the existence of a positive act of the government such as a presidential proclamation or an
executive order, an administrative action, investigative reports of the Bureau of Lands investigators, and a
legislative act or statute.
A positive act declaring land as alienable and disposable is required. In keeping with the presumption of
state ownership, the Court has time and again emphasized that there must be a positive act of the
government, such as an official proclamation, declassifying inalienable public land into disposable land for
agricultural or other purposes.
The Regalian Doctrine dictates that all lands of the public domain belong to the State, that the State is the
source of any asserted right to ownership of land and charged with the conservation of such patrimony.

All lands not otherwise appearing to be clearly within private ownership are presumed to belong to the State.
Thus, all lands that have not been acquired from the government, either by purchase or by grant, belong to
the State as part of the inalienable public domain.
REPUBLIC OF THE PHILIPPINES, G.R. No. 159589 1. Certificate of Death proving the fact of death of Juan Fabio;[6]
Petitioner, 2. Special Power of Attorney showing that the heirs authorized Esteibar to file the application; [7]
Present: 3. Order dated 25 November 1994 of Sydicious F. Panoy, Regional Technical Director, Regional
- versus - Office No. IV-A, Department of Environment and Natural Resources (DENR), giving authority to
PUNO, C.J., Chairperson, survey the Lot, which survey was numbered SWO-042121-003369-D;[8]
HEIRS OF JUAN FABIO, CARPIO, 4. Surveyors Certificate and Transmittal of Survey Returns signed by Geodetic Engineer Susipatro
namely: DOMINGA C. FABIO, SOCORRO D. FABIO, CHICO-NAZARIO,* Mancha proving that the Lot was surveyed;[9]
LYDIA D. FABIO, ROLANDO D. FABIO, NORMA D. VELASCO, JR.,** and 5. Sepia copies of the survey plan establishing that the land area is more or less 109 hectares and
FABIO, NORMA L. FABIO, ANGELITA FABIO, LEONARDO-DE CASTRO, JJ. that the Lot was already surveyed and the boundaries determined; [10]
ROSALIE FABIO, DANILO 6. Letter of Authority dated 30 June 1997 authorizing Engineer Roberto C. Pangyarihan
FABIO, RENATO FABIO, (Pangyarihan) to represent the Land Management Sector, DENR-Region IV, and to testify on plan
LEVITA FABIO, IRENE FABIO, TERESITA MOLERA, SWO-042121-003369-D covering the Lot;[11]
ROSEMARIE C. PAKAY, 7. Technical Description signed by Pangyarihan proving the boundaries of the Lot as surveyed;[12]
LIGAYA C. MASANGKAY, ALFREDO F. CASTILLO, 8. Letter dated 22 April 1991 of Arnaldo Conlu (Conlu), Land Management Inspector, DENR-Region
MELINDA F. CASTILLO, MERCEDITA F. CASTILLO, IV establishing that the Lot is alienable and disposable;[13]
ESTELA DE JESUS AQUINO, FELECITO FABIO, 9. First Indorsement dated 22 April 1991 of Rufo F. Lorenzo, Community Environment and Natural
and ALEXANDER FABIO, represented herein by Resources Officer, forwarding to the Regional Technical Director, Land Management Division,
ANGELITA F. ESTEIBAR as their Attorney-in-Fact, through the Chief, Surveys Division, DENR-Region IV the investigation report of Land Management
Respondents. Inspector, Conlu;[14]
10. Certifications dated 4 July 1995 and 23 November 1995 of Conrado C. Lindo,
Municipal Mayor, and Flordeliza C. Soberano, Municipal Assessor of Ternate, Cavite, respectively,
establishing that Juan Fabio was the declared owner of the Lot under Tax Declaration No. 1385
Promulgated: having an area of 200 hectares and situated in Calumpang and Caybangat, Zapang, Ternate,
Cavite;[15]
December 23, 2008 11. Tax Declarations corresponding to different years showing that the Lot has been
x- - - - - - - - - - - - - - - - - - - - - - - - - - - - - - - - - - - - - - - - - - - - - - - - - - - x declared under the name of Juan Fabio for tax purposes: Tax Declaration No. 428 for the year
1947, Tax Declaration No. 302 for the year 1961; Tax Declaration No. 227 for the year 1969, Tax
D E C I S I O N Declaration No. 210 for the year 1974, Tax Declaration No. 173 for the year 1980, Tax Declaration
No. 1543 for the year 1985, and Tax Declaration No. 1385 for the year 1994;[16] and
CARPIO, J.: 12. Certifications of the Assistant Municipal Treasurer of Ternate, Cavite stating that the real estate
taxes for the years 1994 to 1997 were paid.[17]
The Case
After the presentation of exhibits establishing the jurisdictional facts, the trial prosecutor assigned to the case
Before the Court is a petition for review on certiorari [1] assailing the Decision[2] dated 29 August 2003 of the interposed no objection. Thus, the trial court ordered a general default against the public except the
Court of Appeals in CA-G.R. CV No. 66522, which affirmed the judgment of the Regional Trial Court of government.
Naic, Cavite, Branch 15, in LRC Case No. NC-96-782 granting respondents application for registration of
title to Lot No. 233 (Lot), Cad-617-D, Ternate Cadastre. On 1 July 1997, respondents presented their evidence consisting of documentary exhibits and the
The Facts testimonies of witnesses Esteibar, Pangyarihan, Dominga Fabio Lozano, Mariano Huerto, and Raymundo
On 21 November 1996, respondents, who are the heirs of Juan Fabio, represented by Angelita F. Esteibar Pakay.
(Esteibar) as their Attorney-in-Fact, filed with the Regional Trial Court of Naic, Cavite, Branch 15, an Esteibar, the duly appointed representative of the heirs of Juan Fabio, testified that her grandfather, Juan,
application for registration of title[3] to the Lot with an approximate area of 1,096,866 square meters or 109.6 died in 1959 when she was only 13 years old. She attested that she was born on the Lot and knows that her
hectares. The Lot is situated in Barangay Sapang, Ternate, Cavite. The respondents sought the registration grandfather owned, possessed and occupied the Lot until his death. Esteibar claimed that they and their
of title under the provisions of Act No. 496 or the Land Registration Act, as amended by Presidential Decree predecessors-in-interest have possessed and occupied the Lot openly, publicly, continuously, peacefully,
No. 1529 (PD 1529).[4] without interruption in the concept of an owner and adverse to the public since time immemorial up to the
present or for more than 100 years. They had paid real estate taxes; planted trees, vegetables, rice, and
In the application, respondents alleged that they are the owners of the Lot, including all the improvements, banana plants; and raised animals on the Lot. Further, she stated that the Lot is neither mortgaged nor
having acquired the same through a bona fide claim of ownership.They declared that they and their encumbered and that no other person other than her and her co-heirs are in possession of the Lot.
predecessors-in-interest were in open, continuous, exclusive and notorious possession of the Lot in the
concept of an owner for more than 100 years.[5] The next witness, Pangyarihan of the Land Management Sector, DENR-Region IV, testified that he had
been connected with DENR-Region IV since 1956. He was formerly the Chief of the Survey Division of
Together with the application for registration, respondents submitted the following documents: DENR-Region IV from 1991 until his designation as Special Assistant to the Regional Director in
1995. Pangyarihan affirmed that the Lot is 1,096,866 square meters or 109.6 hectares and that he
recommended the approval of the survey plan, SWO-042121-003369-D, which includes the Lot, on the basis FURTHER, upon the finality of this DECISION, the Administrator, Land Registration
of submission of certain requirements like tax declarations, report of investigation by the land investigator Authority, is hereby ordered to issue the corresponding decree of registration and the
and survey returns prepared by the geodetic engineer. He verified that the survey plan and the technical Original Certificate of Title in favor of, and in the name of JUAN FABIO, of Barangay
descriptions matched with each other and stated that there is no overlap or encroachment on other Sapang, Municipality of Ternate, Province of Cavite, over the parcel of land described,
surrounding claims on adjacent or adjoining lots. Further, he confirmed that there is a notation at the left identified and bounded as hereinabove-mentioned and subject matter of this Decision
hand footnote of the approved survey plan which reads this survey falls within the Calumpang Point which decreed and adjudged the registration of its title in his name.
Naval Reservation and disposition hereof shall be subject to the final delimitation thereof as per
Proc. No. 1582-A dated September 6, 1976. SO ORDERED.[19]
Dominga Fabio Lozano, the only living and youngest child of Juan Fabio and who was then 63 years of age, The Republic of the Philippines (petitioner), through the Office of the Solicitor General, filed an appeal with
testified that she was born in 1934 in Calumpang, Ternate, Cavite.She alleged that she was born and has the Court of Appeals. Petitioner claimed that the trial court erred in ruling that respondents have acquired a
lived on the Lot, owned by her father Juan Fabio, who in turn inherited the land from his father Ignacio vested right over the Lot which falls within the Calumpang Point Naval Reservation. Petitioner asserted that
Fabio. She narrated that her father was born in 1887 and died in 1959 at the age of 72 as evidenced by his the trial court disregarded the testimony of Pangyarihan who recommended the approval of the survey plan
death certificate. She stated further that no one has ever questioned their ownership or disturbed their with the following notation:
peaceful possession and occupation of the Lot. As a result, their possession of the Lot covers more than 100
years of continuous, uninterrupted, public, open and peaceful possession. This survey falls within the Calumpang Point Naval Reservation and disposition hereof
Mariano Huerto, a helper of the late Juan Fabio, testified that since 1935, when he was only 12 years old, he shall be subject to the final delimitation thereof as per Proc. No. 1582-A dated September
had helped cultivate the Lot until he left the place in 1955. He stated that at the time he served as helper, 6, 1976. x x x
Juan Fabio and his family were the ones who possessed and occupied the Lot. He helped plant vegetables,
banana plants, papaya trees and upland rice and was familiar with the boundaries of the Lot. In essence, petitioner argued that the trial courts grant of registration is contrary to the provisions of Section
Raymundo Pakay, 70 years of age at the time and a resident of Ternate, Cavite, testified that he knew Juan 88 of Commonwealth Act No. 141[20] and Proclamation No. 1582-A.[21]
Fabio as the owner of the Lot, which has an area of 200 hectares, more or less. He stated that Juan built a
house there and could not recall of anyone else who claimed ownership of the Lot.
On 7 August 1997, the Assistant City Prosecutor of Tagaytay City filed his Manifestation and Comment The Ruling of the Court of Appeals
dated 28 July 1997:

COMES NOW the government, through the undersigned Assistant City Prosecutor of On 29 August 2003, the Court of Appeals affirmed the ruling of the trial court. [22] The appellate court ruled
Tagaytay City, assisting the Office of the Provincial Prosecutor of the Province of Cavite, that the mode of appeal filed by petitioner was wrong. Since the lone question involved was one of law,
by way of comment to petitioners formal offer of evidence dated July 3, 1997 hereby petitioner should have filed a petition for review with this Court under Rule 45 of the 1997 Rules of Civil
manifest that the government interposes no objection to Exhibit A up to PP together with Procedure instead of filing an appeal under Rule 41. Nevertheless, the appellate court looked into the merits
its sub markings, the same being material and relevant to the instant petition. of the case and sustained the findings of the trial court:

The government further manifests that considering the fact that it has no controverting On the merits of the case, it may be true that the General Order 56 of the United States
evidence in its possession to refute the material allegations of the herein petitioner, the War Department dated 25 March 1904 reserved the subject property as a military
government is submitting the instant case for the immediate resolution of this Honorable reservation, however, President Ferdinand Marcos issued Proclamation 307 on 20
Court on the basis of the evidence adduced by the petitioner and the cross examination November 1967 which provides x x x.
propounded by the Trial Prosecutor.[18]
In other words, Presidential Proclamation 307 provides for an exception those properties
subject to private rights or those on which private individuals can prove ownership by any
mode acceptable under our laws and Torrens system.
On 29 September 1997, the trial court rendered a Decision ordering the registration of the Lot in the name of
Juan Fabio. The dispositive portion states: Proclamation 1582-A issued by President Marcos on 6 September 1976 again provided
the following x x x.
WHEREFORE, PREMISES CONSIDERED, finding the application for registration and
grant of title under Act 496, as amended by Presidential Decree No. 1529 to be Without doubt, this complements and recognizes the rights acquired by private individuals
meritorious and fully substantiated by evidence sufficient and requisite under the law, this under Proclamation 307, over the portion of the properties reserved under General Order
Court, confirming its previous Order of general default as against the general public, 56 of the United States War Department dated 25 March 1904.
hereby decrees and adjudges and hereby orders the registration of the parcel of land as
hereinabove described, identified, and bounded and now the subject matter of the present Considering that the annotation appearing in the survey plan merely provides that the
application for registration of title in the above-entitled case, in favor of, and in the name of controversial portion shall be subject to final delimitation as per Proclamation 1582-A, the
JUAN FABIO, of Barangay Sapang, Municipality of Ternate, Province of Cavite. same is consistent with the provisions of Proclamation 307.
For wrong remedy and for lack of merit, the Court holds and so rules that the trial court In its appellants brief filed with the Court of Appeals, petitioner interposed a lone assignment of error:
erred not in granting petitioners application for registration of title.
The trial court erred in ruling that appellees have acquired a vested right over the subject
WHEREFORE, premises considered, the appeal is DISMISSED and the challenged 29 property despite the fact that it falls within the Calumpang Point Naval Reservation.[25]
September 1997 Decision of the court a quo is hereby AFFIRMED in toto. No costs.

SO ORDERED.[23] Clearly, the issue stated by petitioner provides no confusion with regard to the truth or falsity of the given
facts pertaining to the Lot and its location as established during the trial.It had been duly established that the
Lot falls within the Calumpang Point Naval Reservation as shown in the survey conducted and attested to by
the DENR. Here, the only issue involved is the interpretation of a relevant order and proclamations
Hence, the instant petition. denominating the Lot as part of a military reservation subject to the limitation that private rights should be
The Issues respected. Undoubtedly, this is a pure question of law.
The issues for our resolution are (1) whether petitioner correctly appealed the ruling of the trial court to the
Court of Appeals, and (2) whether the respondents have acquired a right over the Lot. Thus, petitioners appeal under Rule 41 having been improperly brought before the Court of Appeals, it
The Courts Ruling should have been dismissed by the appellate court pursuant to Section 2, Rule 50 of the 1997 Rules of Civil
The petition has merit. Procedure, as amended, which provides:
First Issue: Mode of Appeal
Sec. 2. Dismissal of improper appeal to the Court of Appeals. An appeal under Rule
Petitioner contends that the jurisdiction of the Court of Appeals over the appeal is determined on the basis of 41 taken from the Regional Trial Court to the Court of Appeals raising only questions of
the averments in the notice of appeal. Since the appeal involves questions of fact and law, petitioner law shall be dismissed, issues purely of law not being reviewable by the said court. x x x
correctly appealed the ruling of the trial court to the Court of Appeals and not directly to this Court. (Emphasis supplied)
Respondents, on the other hand, maintain that the remedy resorted to by petitioner before the Court of
Appeals was not correct. Respondents contend that the issues actually raised in the appellants brief Nonetheless, the appellate court in resolving that petitioners appeal constituted a wrong remedy, looked into
determine the appropriate mode of appeal, not the averments in the notice of appeal. Since the appellate the merits of the case and found that the laws involved recognize the rights of respondents. As such, equity
court found that petitioner only raised questions of law, the appeal is dismissible under the Rules. considerations require that we take a similar course of action in order to put a rest to this case.

Section 2, Rule 41 of the 1997 Rules of Civil Procedure, as amended, which governs appeals from
judgments and final orders of the Regional Trial Court to the Court of Appeals, provides:
Second Issue: Validity of Respondents Title
Section 2. Modes of appeal.

(a) Ordinary appeal. The appeal to the Court of Appeals in cases decided by the Regional Petitioner asserts that both the trial and appellate courts failed to recognize the import of the notation in the
Trial Court in the exercise of its original jurisdiction shall be taken by filing a notice of appeal survey plan stating that the Lot falls within the Calumpang Point Naval Reservation. At the time the
with the court which rendered the judgment or final order appealed from and serving a copy application for registration of title was filed, the Lot was no longer open to private ownership as it had been
thereof upon the adverse party. No record on appeal shall be required except in special classified as a military reservation for public service. Thus, respondents are not entitled to have the Lot
proceedings and other cases of multiple or separate appeals where the law or these Rules so registered under the Torrens system.
require. In such cases, the record on appeal shall be filed and served in like manner. Respondents, on the other hand, maintain that they have acquired a vested right over the Lot. The
Presidential Proclamations, which declared the Lot part of a naval reservation, provided for an exception that
(b) Petition for review. The appeal to the Court of Appeals in cases decided by the Regional private rights shall be respected, taking the portion covered by private rights out of the reservation. Thus,
Trial Court in the exercise of its appellate jurisdiction shall be by petition for review in respondents claim they are entitled to have the Lot registered under their names.
accordance with Rule 42. The three proclamations cited reserving the Calumpang Point Naval Reservation for the exclusive use of the
military are the following: (1) U.S. War Department Order No. 56 issued on 25 March 1904, (2) Proclamation
(c) Appeal by certiorari. In all cases where only questions of law are raised or involved, No. 307[26] issued on 20 November 1967, and (3) Proclamation No. 1582-A issued on 6 September
the appeal shall be to the Supreme Court by petition for review on certiorari in 1976. Such proclamations state:
accordance with Rule 45. (Emphasis supplied)
U.S. War Department General Order No. 56[27]
A question of law arises when there is doubt as to what the law is on a certain state of facts, while there is a
question of fact when the doubt arises as to the truth or falsity of the alleged facts. For questions to be one U.S. War Department General Order No. 56
of law, the same must not involve an examination of the probative value of the evidence presented by the Washington, March 25, 1904.
litigants. The resolution of the issue must rest solely on what the law provides on the given set of
circumstances.[24] For the knowledge and governance of all interested parties, the following is hereby
announced:
The President of the United States, by the Order dated March 14, 1904, which provides By Order of the Secretary of War:
that the reservations made by Executive Order of April 11, 1902 (General Order No. 38,
Army Headquarters, Office of the Adjutant General, April 17, 1902), at the entrance of GEORGE L. GILLESPIE,
Manila Bay, Luzon, Philippine Islands, are arranged in such a way that will include only General Commander, Chief of Internal General Staff,
these lands as later described, whose lands were reserved by the Order of March 14, Official copy.
1904 for military purposes, by virtue of Article 12 of the Act of Congress approved on July
1, 1902, entitled Act providing for the Temporary Administration of Civil Affairs of the W.P. HALL, Internal Adjutant General. (Emphasis supplied)
Government of the Philippine Islands and for Other Purposes (32 Stat. L., 691); namely:
Proclamation No. 307
1. In the northern side of the entrance to Manila Bay, in the province of Bataan,
Luzon (Mariveles Reservation), all public lands within the limits that are described as x x x do hereby withdraw from sale or settlement and reserve for military purposes under
follows: the administration of the Chief of Staff, Armed Forces of the Philippines, subject to private
rights, if any there be, a certain parcel of land of the public domain situated in the
Starting from the mouth of the Mariveles River in the eastern border and from here straight municipality of Ternate, province of Cavite, Island of Luzon, more particularly described as
North to a distance of 5,280 feet; from this point straight to the East to intercept a line, in a follows:
straight direction to the South from a stone monument marked U.S. (Station 4); from there
straight from the North until the aforementioned Station 4; from here straight to the East to Proposed Naval Reservation
a distance of 6,600 feet until a stone monument marked U.S. (Station 5); from here Calumpang Point
straight South to a distance of 6,600 feet until a stone monument marked U.S. (Station 6);
from here straight to the East to a distance of 8,910 feet until a stone monument marked A parcel of land (the proposed Calumpang Point Naval Reservation), situated in the
U.S. (Station 7); from here straight to the South to a distance of 7,730 feet until a stone municipality of Ternate, province of Cavite. Bounded on the NW., N. and E., by Manila
monument marked U.S. (Station 8), situated at the northwest corner of the second creek Bay; on the SE. and S., by municipality of Ternate; and on the W., by Manila
to the east of Lasisi Point, 30 feet North of the high-tide mark; from there in the same Bay. Beginning at a point marked 1 on the attached Sketch Plan traced from Coastal
direction until the high-tide mark; from here towards the East following the shoreline up to Hydrography of Limbones Island.
the starting point. thence N. 54 deg. 30 E., 750.00 m. to point 2;
thence N. 89 deg. 15E., 1780.00 m. to point 3;
2. In the southern side of the Manila Bay entrance, in the province of thence N. 15 deg. 10 E., 6860.00 m. to point 4;
Cavite, Luzon (Calumpan Point Reservation), all public lands within the limits that thence N. 12 deg. 40 W., 930.00 m. to point 5;
are described as follows: thence S. 77 deg. 20 W., 2336.00 m. to point 6;
thence S. 49 deg. 30 W., 4450.00 m. to point 7;
Starting from a stone monument marked U.S. (Station 1) situated in the cliff on the thence S. 12 deg. 40 E., 2875.00 m. to point 8;
Eastern side of Asubig Point, 20 feet above the high-tide mark and about 50 feet thence S. 30 deg. 30 E., 2075.00 m. to the point of beginning; containing
from the edge of the cliff and continuing from there to the South 28 10 West, a an approximate area of twenty eight million nine hundred seventy three
distance of up to 22,000 feet until a stone monument marked U.S. (Station 2); from thousand one hundred twelve (28, 973,112) square meters.
here to North 54 10' West at a distance of 5,146 feet until a stone monument marked
U.S. (Station 3); from here towards South 85 35 ' 30 West, at a distance of 2,455 feet NOTE: All data are approximate and subject to change based on future
until a stone monument marked U.S. (Station 4), situated on the beach near the surveys.
Northeast corner of Limbones Bay, about 50 feet from the high-tide mark and
following in the same direction until the high-tide mark; from here towards North
and East following the shoreline until North 28 10 ' East from the starting point and Proclamation No. 1582-A
from there encompassing more or less 5,200 acres. The markers are exact.

3. The islands of Corregidor, Pulo Caballo, La Monja, El Fraile, and Carabao, WHEREAS, Proclamation No. 307 dated November 20, 1967 and U.S. War
and all other islands and detached rocks lying between Mariveles Reservation on the Department Order No. 56 dated March 25, 1904 reserved for military purposes, and
north side of the entrance to Manila Bay and Calumpan Point Reservation on the south withdrew from sale or settlement, a parcel of land of the public domain situated in
side of said entrance. the Municipality of Ternate, Province of Cavite, more particularly described as follows:
xxx
4. The jurisdiction of the military authorities in the case of reservations in the
northern and southern beaches of the entrance to Manila Bay and all the islands referred WHEREAS, the Philippine Navy and the Philippine Marines now need that portion of this
to in paragraph 3, are extended from the high-tide marker towards the sea until a distance area reserved under Proclamation No. 307, particularly, Cayladme Cove, Caynipa Cove,
of 1,000 yards. Calumpang Cove and Sinalam Cove, for their use as official station, not only to guard and
protect the mouth of Manila Bay and the shorelines of the Province of Cavite, Batangas
and Bataan, but also to maintain peace and order in the Corregidor area, which is now have performed all the conditions essential to a Government grant and shall be entitled to
one of the leading tourist attractions in the country; x x x a certificate of title under the provisions of this chapter.[32]

x x x containing an approximate area of EIGHT MILLION EIGHTY NINE THOUSAND Similarly, Section 14 of PD 1529 or the Property Registration Decree, governing original registration through
NINE HUNDRED NINETY (8,089,990) SQUARE METERS, more or less. registration proceedings, provides:

The portion that remains after the segregation which are occupied shall be released SECTION 14. Who may apply. - The following persons may file in the proper Court of First
to bona fide occupants pursuant to existing laws/policies regarding the disposition of lands Instance an application for registration of title to land, whether personally or through their
of the public domain and the unoccupied portions shall be considered as alienable or duly authorized representatives:
disposable lands. (Emphasis supplied)
(1) Those who by themselves or through their predecessors-in-interest have been in
open, continuous, exclusive and notorious possession and occupation of
The proclamations established that as early as 1904 a certain parcel of land was placed under the exclusive alienable and disposable lands of the public domain under a bona fide claim of
use of the government for military purposes by the then colonial American government. In 1904, the U.S. ownership since June 12, 1945, or earlier.
War Department segregated the area, including the Lot, for military purposes through General Order No. xxx
56. Subsequently, after the Philippines regained its independence in 1946, the American government
transferred all control and sovereignty to the Philippine government, including all the lands appropriated for a To put it simply, Section 14(1) of PD 1529 states that there are three requisites for the filing of an application
public purpose. Twenty years later, two other presidential proclamations followed, both issued by former for registration of title: (1) that the property in question is alienable and disposable land of the public domain;
President Ferdinand E. Marcos, restating that the same property is a naval reservation for the use of the (2) that the applicants by themselves or through their predecessors-in-interest have been in open,
Republic. continuous, exclusive and notorious possession and occupation; and (3) that such possession is under
There is no question that the Lot is situated within a military reservation. The only issue to be resolved is a bona fide claim of ownership since 12 June 1945 or earlier.
whether the respondents are entitled to have the Lot registered under the Torrens systems based on the
limitation clause cited in the proclamations: (1) subject to private rights, if any there be in Proclamation No. To prove that the Lot is alienable and disposable land of the public domain, respondents presented in
307, and (2) the portion that remains after the segregation which are occupied shall be released to bona evidence a letter[33] dated 22 April 1991 of Conlu, a Land Management Inspector of the DENR-Region IV.
fide occupants pursuant to existing laws/policies regarding the disposition of lands of the public domain and The relevant portion of the letter states:
the unoccupied portions shall be considered as alienable or disposable lands in Proclamation No. 1582-
A. This proviso means that persons claiming rights over the reserved land are not precluded from proving In examination [of] the above-noted subject, please be [informed] that I have examined the
their claims. In effect, the State gives respect and recognizes the rights of private persons who may have land x x x and the following findings [were] ascertained;
acquired any vested interest to the Lot before the issuance of the General Order or proclamations.
That the land covers a portion of 3 (three) barangays, namely: Calumpang, Cabangat and
Commonwealth Act No. 141 (CA 141), also known as the Public Land Act, remains to this day the existing Zapang, all within the municipality of Ternate, Cavite;
general law governing the classification and disposition of lands of the public domain, other than timber and
mineral lands.[28] Under the Regalian doctrine embodied in our Constitution, land that has not been acquired That the land is within alienable and disposable zone under Project No. 22-B, L.C. Map
from the government, either by purchase, grant or any other mode recognized by law, belongs to the State No. 3091;
as part of the public domain.[29] No public land can be acquired by private persons through any other means,
and it is indispensable that the person claiming title to public land should show that his title was acquired That the land was declared for taxation purposes since 1945, the latest of which is Tax
through purchase or grant from the State, or through any other mode of acquisition recognized by law.[30] Declaration No. 1543 with a market value of P1,250,000.00 in favor of Juan Fabio x x
x (Emphasis supplied)
Section 48(b) of CA 141, as amended by Presidential Decree No. 1073 (PD 1073), [31] provides:
This letter-certification is insufficient. Conlu is merely a land investigator of the DENR. It is not enough that
Sec. 48. The following described citizens of the Philippines, occupying lands of the public he alone should certify that the Lot is within the alienable and disposable zone. Under Section 6 of the Public
domain or claiming to own any such land or an interest therein, but whose titles have not Land Act, the prerogative of classifying or reclassifying lands of the public domain belongs to the
been perfected or completed, may apply to the Court of First Instance of the province President.[34] The President, through a presidential proclamation or executive order, can classify or reclassify
where the land is located for confirmation of their claims and the issuance of a certificate of a land to be included or excluded from the public domain. The DENR Secretary is the only other public
title therefor, under the Land Registration Act, to wit: official empowered by law to approve a land classification and declare such land as alienable and
xxx disposable.[35]
(b) Those who by themselves or through their predecessors in interest have been in open,
continuous, exclusive, and notorious possession and occupation of agricultural lands of From the records, this letter was the only evidence presented by respondents to prove that the Lot is
the public domain, under a bona fide claim of acquisition of ownership, since June 12, alienable and disposable. In fact, not even the Community Environment and Natural Resources Office
1945, or earlier, immediately preceding the filing of the application for confirmation of title, (CENRO) certified as correct the investigation report of the Land Management Inspector. The most that the
except when prevented by war or force majeure. These shall be conclusively presumed to CENRO officer did was to indorse the report to the Regional Technical Director of the DENR. [36] In Republic
v. T.A.N. Properties, Inc.,[37] we ruled that it is not enough for the Provincial Environment and Natural
Resources Office (PENRO) or CENRO to certify that a land is alienable and disposable. The applicant for
land registration must prove that the DENR Secretary had approved the land classification and released the While it is an acknowledged policy of the State to promote the distribution of alienable public lands to spur
land of the public domain as alienable and disposable, and that the land subject of the application for economic growth and in line with the ideal of social justice, the law imposes stringent safeguards upon the
registration falls within the approved area per verification through survey by the PENRO or CENRO. In grant of such resources lest they fall into the wrong hands to the prejudice of the national patrimony. [42] We
addition, the applicant must present a copy of the original classification of the land into alienable and must not, therefore, relax the stringent safeguards relative to the registration of imperfect titles.[43]
disposable, as declared by the DENR Secretary, or as proclaimed by the President. Such copy of the DENR
Secretarys declaration or the Presidents proclamation must be certified as a true copy by the legal custodian In Republic v. Estonilo,[44] we ruled that persons claiming the protection of private rights in order to exclude
of such official record. These facts must be established to prove that the land is alienable and disposable. their lands from military reservations must show by clear and convincing evidence that the properties in
question have been acquired by a legal method of acquiring public lands. Here, respondents failed to do so,
Respondents have failed to present any of these documents. No document was presented to show that the and are thus not entitled to have the Lot registered in their names. Clearly, both the trial and appellate courts
DENR Secretary or the President has classified the Lot as alienable and disposable. No CENRO or PENRO gravely erred in granting respondents application for registration of title.
certification was presented that the Lot, per verification through survey, falls within the alienable and
disposable zone. The 22 April 1991 letter of Land Management Inspector Conlu is not proof that the DENR WHEREFORE, we GRANT the petition. We SET ASIDE the 29 August 2003 Decision of the Court of
Secretary or the President has classified the Lot as alienable and disposable, or that the Lot falls within the Appeals in CA-G.R. CV No. 66522. We DISMISS respondents application for registration and issuance of
alienable and disposable zone. The mere issuance of the letter does not prove the facts stated in such title to Lot No. 233, Cad-617-D, Ternate Cadastre in LRC Case No. NC-96-782 filed with the Regional Trial
letter.[38] Court of Naic, Cavite, Branch 15.

Further, the burden is on respondents to prove that the Lot ceased to have the status of a military SO ORDERED.
reservation or other inalienable land of the public domain. No proof was ever submitted by respondents that
the Calumpang Point Naval Reservation, or the Lot, ceased as a military reservation. Even if its ownership
and control had been transferred by the Americans to the Philippine government, the Calumpang Point
Naval Reservation remained as an official military reservation. Thus, being a military reservation at the time,
the Calumpang Point Naval Reservation, to which the Lot is a part of, can not be subject to occupation, entry
or settlement.[39] This is clear from Sections 83 and 88 of CA 141, which provide:

SECTION 83. Upon the recommendation of the Secretary of Agriculture and Commerce,
the President may designate by proclamation any tract or tracts of land of the public
domain as reservations for the use of the Commonwealth of the Philippines or of any of its
branches, or of the inhabitants thereof, in accordance with regulations prescribed for this
purpose, or for quasi-public uses or purposes when the public interest requires it, including
reservations for highways, rights of way for railroads, hydraulic power sites, irrigation
systems, communal pastures or leguas comunales, public parks, public quarries, public
fishponds, working-mens village and other improvements for the public benefit.
SECTION 88. The tract or tracts of land reserved under the provisions of section eighty-
three shall be non-alienable and shall not be subject to occupation, entry, sale, lease,
or other disposition until again declared alienable under the provision of this Act or
by proclamation of the President. (Emphasis supplied)

Well-entrenched is the rule that unless a land is reclassified and declared alienable and disposable,
occupation in the concept of an owner, no matter how long, cannot ripen into ownership and be registered
as a title.[40] Consequently, respondents could not have occupied the Lot in the concept of an owner in 1947
and subsequent years when respondents declared the Lot for taxation purposes, or even earlier when
respondents predecessors-in-interest possessed the Lot, because the Lot was considered inalienable from
the time of its declaration as a military reservation in 1904. Therefore, respondents failed to prove, by clear
and convincing evidence, that the Lot is alienable and disposable.

Public lands not shown to have been classified as alienable and disposable land remain part of the
inalienable public domain.[41] In view of the lack of sufficient evidence showing that the Lot was already
classified as alienable and disposable, the Lot applied for by respondents is inalienable land of the public
domain, not subject to registration under Section 14(1) of PD 1529 and Section 48(b) of CA 141, as
amended by PD 1073. Hence, there is no need to discuss the other requisites dealing with respondents
occupation and possession of the Lot in the concept of an owner.

You might also like